Anda di halaman 1dari 128

Our Google Groups contain Banking and Financial Study Materials.

From the xam point of view they are more than sufficient and will also help U'll in the interview. Keep the discussions rolling , here a bit from my side : 1. What is a Repo Rate? A: Repo rate is the rate at which our banks borrow rupees from RBI. Whenever the banks have any shortage of funds they can borrow it from RBI. A reduction in the repo rate will help banks to get money at a cheaper rate. When the repo rate increases, borrowing from RBI becomes more expensive. 2. What is Reverse Repo Rate? A: This is exact opposite of Repo rate. Reverse Repo rate is the rate at which Reserve Bank of India (RBI) borrows money from banks. RBI uses this tool when it feels there is too much money floating in the banking system. Banks are always happy to lend money to RBI since their money is in safe hands with a good interest. An increase in Reverse repo rate can cause the banks to transfer more funds to RBI due to this attractive interest rates. 3. What is CRR Rate? A: Cash reserve Ratio (CRR) is the amount of funds that the banks have to keep with RBI. If RBI decides to increase the percent of this, the available amount with the banks comes down. RBI is using this method (increase of CRR rate), to drain out the excessive money from the banks.3 4. What is SLR Rate? A: SLR (Statutory Liquidity Ratio) is the amount a commercial bank needs to maintain in the form of cash, or gold or govt. approved securities (Bonds) before providing credit to its customers. SLR rate is determined and maintained by the RBI (Reserve Bank of India) in order to control the expansion of bank credit. SLR is determined as the percentage of total demand and percentage of time liabilities. Time Liabilities are the liabilities a commercial bank liable to pay to the customers on their anytime demand. SLR is used to control inflation and propel growth. Through SLR rate tuning the money supply in the system can be controlled efficiently. 5. What is Bank Rate? A: Bank rate, also referred to as the discount rate, is the rate of interest which a central bank charges on the loans and advances that it extends to commercial banks and other financial intermediaries. Changes in the bank rate are often used by central banks to control the money supply. 6. What is Inflation? A: Inflation is as an increase in the price of bunch of Goods and services that projects the Indian economy. An increase in inflation figures occurs when there is an increase in the average level of prices in Goods and services. Inflation happens when there are fewer Goods and more buyers; this will result in increase in the price of Goods, since there is more demand and less supply of the goods. 7. What is Deflation? A: Deflation is the continuous decrease in prices of goods and services. Deflation occurs when the inflation rate becomes negative (below zero) and stays there for a longer period. 8. What is PLR? A: The Prime Interest Rate is the interest rate charged by banks to their most creditworthy customers (usually the most prominent and stable business customers). The rate is almost always the same amongst major banks. Adjustments to the prime rate are made by banks at the same time; although, the prime rate does not adjust on any regular basis. The Prime Rate is usually adjusted at the same time and in correlation to the adjustments of the Fed Funds Rate. The rates reported below are based upon the prime rates on the first day of each respective month. Some banks use the name "Reference Rate" or "Base Lending Rate" to

refer to their Prime Lending Rate. 9. What is Deposit Rate? A: Interest Rates paid by a depository institution on the cash on deposit. 10. What is FII? A: FII (Foreign Institutional Investor) used to denote an investor, mostly in the form of an institution. An institution established outside India, which proposes to invest in Indian market, in other words buying Indian stocks. FII's generally buy in large volumes which has an impact on the stock markets. Institutional Investors includes pension funds, mutual funds, Insurance Companies, Banks, etc. 11. What is FDI? A: FDI (Foreign Direct Investment) occurs with the purchase of the physical assets or a significant amount of ownership (stock) of a company in another country in order to gain a measure of management control (Or) A foreign company having a stake in a Indian Company. 12. What is IPO? A: IPO is Initial Public Offering. This is the first offering of shares to the general public from a company wishes to list on the stock exchanges. 13. What is Disinvestment? A: The Selling of the government stake in public sector undertakings. 14. What is Fiscal Deficit? A: It is the difference between the governments total receipts (excluding borrowings) and total expenditure. Fiscal deficit in 2009-10 is proposed at 6.8% of GDP. 15. What is Revenue deficit? A: It defines that, where the net amount received (by taxes & other forms) fails to meet the predicted net amount to be received by the government. Revenue deficit in 2009-10 is proposed at 4.8% of GDP. 16. What is GDP? A: The Gross Domestic Product or GDP is a measure of all of the services and goods produced in a country over a specific period; classically a year. GDP during 2008-09 is 6.7%. 17. What is GNP? A: Gross National Product is measured as GDP plus income of residents from investments made abroad minus income earned by foreigners in domestic market. 18. What is National Income? A: National Income is the money value of all goods and services produced in a country during the year. 19. What is Per Capita Income? A: The national income of a country, or region, divided by its population. Per capita income is often used to measure a country's standard of living.Per capita income during 2008-09 estimated by CSO: Rs.25, 494. 20. What is Vote on Account? A: A vote-on account is basically a statement ,where the government presents an estimate of a sum required to meet the expenditure that it incurs during the first three to four months of an election financial year until a new government is in place, to keep the machinery running.

21. Difference between Vote on Account and Interim Budget? A: Vote-on-account deals only with the expenditure side of the government's budget, an interim Budget is a complete set of accounts, including both expenditure and receipts. 22. What is SDR? A: The SDR (Special Drawing Rights) is an artificial currency created by the IMF in 1969. SDRs are allocated to member countries and can be fully converted into international currencies so they serve as a supplement to the official foreign reserves of member countries. Its value is based on a basket of key international currencies (U.S. dollar, euro, yen and pound sterling). 23. What is SEZ? A: SEZ means Special Economic Zone is the one of the part of governments policies in India. A special Economic zone is a geographical region that economic laws which are more liberal than the usual economic laws in the country. The basic motto behind this is to increase foreign investment, development of infrastructure, job opportunities and increase the income level of the people. What is corporate governance? The way in which a company is governed and how it deals with the various interests of its customers, shareholders, employees and society at large. Corporate governance is the set of processes, customs, policies, laws, and institutions affecting the way a corporation (or company) is directed, administered or controlled.Is defined as the general set of customs, regulations, habits, and laws that determine to what end a firm should be run. Functions of RBI? The Reserve Bank of India is the central bank of India, was established on April 1, 1935 in accordance with the provisions of the Reserve Bank of India Act, 1934. The Reserve Bank of India was set up on the recommendations of the Hilton Young Commission. The commission submitted its report in the year 1926, though the bank was not set up for nine years.To regulate the issue of Bank Notes and keeping of reserves with a view to securing monetary stability in India and generally to operate the currency and credit system of the country to its advantage." Banker to the Government: performs merchant banking function for the central and the state governments; also acts as their banker.Banker to banks: maintains banking accounts of all scheduled banks. What is monetary policy? A Monetary policy is the process by which the government, central bank, of a country controls (i) the supply of money, (ii) availability of money, and (iii) cost of money or rate of interest, in order to attain a set of objectives oriented towards the growth and stability of the economy. What is Fiscal Policy? Fiscal policy is the use of government spending and revenue collection to influence the economy. These policies affect tax rates, interest rates and government spending, in an effort to control the economy. Fiscal policy is an additional method to determine public revenue and public expenditure. What is Core Banking Solutions? Core banking is a general term used to describe the services provided by a group of networked bank branches. Bank customers may access their funds and other simple transactions from any of the member branch offices. It will cut down time, working simultaneously on different issues and increasing efficiency. The platform where communication technology and information technology are merged to suit core needs of banking is known as Core Banking Solutions. What is bank and its features and types? A bank is a financial organization where people deposit their money to keep it safe.Banks play an important

role in the financial system and the economy. As a key component of the financial system, banks allocate funds from savers to borrowers in an efficient manner. Regional Rural Banks were established with an objective to ensure sufficientinstitutional credit for agriculture and other rural sectors. The RRBs mobilizefinancial resources from rural / semi-urban areas and grant loans and advancesmostly to small and marginal farmers, agricultural labourers and rural artisans.The area of operation of RRBs is limited to the area as notified by GoI coveringone or more districts in the State.Banking services for individual customers is known as retail banking.A bank that deals mostly in but international finance, long-term loans forcompanies and underwriting. Merchant banks do not provide regular banking services to the general publicOnline banking (or Internet banking) allows customers to conduct financialtransactions on a secure website operated by their retail or virtual bank.Mobile Banking is a service that allows you to do banking transactions on yourmobile phone without making a call , using the SMS facility. Is a term used for performing balance checks, account transactions, payments etc. via a mobile device such as a mobile phone. Traditional banking is the normal bank accounts we have. Like, put your money in the bank and they act as a security and you will get only the normal interests (decided by RBI in our case, FED bank in US).Investment banking is entirely different. Here, people who are having so much money (money in excess which will yield only less interest if in Banks) will invest their money and get higher returns. For example, If i have more money instead of taking the pain of investing in share market, buying properties etc. I will give to investment banks and they will do the money management and give me higher returns when compared to traditional banks. What is E-Governance? E-Governance is the public sectors use of information and communication technologies with the aim of improving information and service delivery, encouraging citizen participation in the decision-making process and making government more accountable,transparent and effective. What is Right to information Act? The Right to Information act is a law enacted by the Parliament of India giving citizens of India access to records of the Central Government and State overnments.The Act applies to all States and Union Territories of India, except the State of Jammu and Kashmir - which is covered under a State-level law. This law was passed by Parliament on 15 June 2005 and came fully into force on 13 October 2005. Credit Rating Agencies in India? The credit rating agencies in India mainly include ICRA and CRISIL. ICRA wasformerly referred to the Investment Information and Credit Rating Agency of India Limited. Their main function is to grade the different sector and companies in terms of performance and offer solutions for up gradation. The credit rating agencies in India mainly include ICRA and CRISIL(Credit Rating Information Services of India Limited) What is Cheque? Cheque is a negotiable instrument instructing a Bank to pay a specific amount from a specified account held in the maker/depositor's name with that Bank.A bill of exchange drawn on a specified banker and payable on demand.Written order directing a bank to pay money. What is demand Draft? A demand draft is an instrument used for effecting transfer of money. It is a Negotiable Instrument. Cheque and Demand-Draft both are used for Transfer of money. You can 100% trust a DD. It is a banker's check. A check may be dishonored for lack of funds a DD can not. Cheque is written by an individual and Demand draft is issued by a bank. People believe banks more than individuals. What is a NBFC? A non-banking financial company (NBFC) is a company registered under the Companies Act, 1956 and is engaged in the business of loans and advances, acquisition of shares/stock/bonds/debentures/securities issued by government, but does not include any institution whose principal business is that of agriculture activity, industrial activity, sale/purchase/construction of immovable property.NBFCs are doing functions akin to that of banks; however there are a few differences:

(i)A NBFC cannot accept demand deposits (demand deposits are funds deposited at a depository institution that are payable on demand -- immediately or within a very short period -- like your current or savings accounts.) (ii) it is not a part of the payment and settlement system and as such cannot issue cheques to its customers; and (iii) Deposit insurance facility of DICGC is not available for NBFC depositors unlike in case of banks. Diff between banking & Finance? Finance is generally related to all types of financial, this could be accounting, insurances and policies. Whereas banking is everything that happens in a bank only.The term Banking and Finance are two very different terms but are often associated together. These two terms are often used to denote services that a bank and other financial institutions provide to its customers. What is NASSCOM ? The National Association of Software and Services Companies (NASSCOM), the Indian chamber of commerce is a consortium that serves as an interface to the Indian software industry and Indian BPO industry. Maintaining close interaction with the Government of India in formulating National IT policies with specific focus on IT software and services maintaining a state of the art information database of IT software and services related activities for use of both the software developers as well as interested companies overseas. What is ASSOCHAM? The Associated Chambers of Commerce and Industry of India (ASSOCHAM), India's premier apex chamber covers a membership of over 2 lakh companies and professionals across the country. It was established in 1920 by promoter chambers, representing all regions of India. As an apex industry body, ASSOCHAM represents the interests of industry and trade, interfaces with Government on policy issues and interacts with counterpart international organizations to promote bilateral economic issues. What is NABARD? NABARD was established by an act of Parliament on 12 July 1982 to implement the National Bank for Agriculture and Rural Development Act 1981. It replaced the Agricultural Credit Department (ACD) and Rural Planning and Credit Cell (RPCC) of Reserve Bank of India, and Agricultural Refinance and Development Corporation (ARDC). It is one of the premiere agency to provide credit in rural areas. NABARD is set up as an apex Development Bank with a mandate for facilitating credit flow for promotion and development of agriculture, small-scale industries, cottage and village industries, handicrafts and other rural crafts. What is SIDBI? The Small Industries Development Bank of India is a state-run bank aimed to aid the growth and development of micro, small and medium scale industries in India. Set up in 1990 through an act of parliament, it was incorporated initially as a wholly owned subsidiary of Industrial Development Bank of India. What is SENSEX and NIFTY? SENSEX is the short term for the words "Sensitive Index" and is associated with the Bombay (Mumbai) Stock Exchange (BSE). The SENSEX was first formed on 1-1-1986 and used the market capitalization of the 30 most traded stocks of BSE. Where as NSE has 50 most traded stocks of NSE.SENSEX IS THE INDEX OF BSE. AND NIFTY IS THE INDEX OF NSE.BOTH WILL SHOW DAILY TRADING MARKS. Sensex and Nifty both are an "index. An index is basically an indicator it indicates whether most of the stocks have gone up or most of the stocks have gone down. What is SEBI? SEBI is the regulator for the Securities Market in India. Originally set up by the Government of India in 1988,

it acquired statutory form in 1992 with SEBI Act 1992 being passed by the Indian Parliament. Chaired by C B Bhave. What is Mutual funds? Mutual funds are investment companies that pool money from investors at large and offer to sell and buy back its shares on a continuous basis and use the capital thus raised to invest in securities of different companies. The mutual fund will have a fund manager that trades the pooled money on a regular basis. The net proceeds or losses are then typically distributed to the investors annually. What is Asset Management Companies? A company that invests its clients' pooled fund into securities that match its declared financial objectives. Asset management companies provide investors with more diversification and investing options than they would have by themselves. Mutual funds, hedge funds and pension plans are all run by asset management companies. These companies earn income by charging service fees to their clients. What are non-perfoming assets? Non-performing assets, also called non-performing loans, are loans,made by a bank or finance company, on which repayments or interest payments are not being made on time. A debt obligation where the borrower has not paid any previously agreed upon interest and principal repayments to the designated lender for an extended period of time. The nonperforming asset is therefore not yielding any income to the lender in the form of principal and interest payments. What is Recession? A true economic recession can only be confirmed if GDP (Gross Domestic Product)growth is negative for a period of two or more consecutive quarters. What is foreign exchange reservers? Foreign exchange reserves (also called Forex reserves) in a strict sense are only the foreign currency deposits and bonds held by central banks and monetary authorities.However, the term in popular usage commonly includes foreign exchange and gold,SDRs and IMF reserve positions. What is Open Market operations(OMO)? The buying and selling of government securities in the open market in order to expand or contract the amount of money in the banking system by RBI. Open market operations are the principal tools of monetary policy. What is Micro Credit? It is a term used to extend small loans to very poor people for self-employment projects that generate income, allowing them to care for themselves and their families. What is Liquidity Adjustment Facility(LAF)? A tool used in monetary policy that allows banks to borrow money through repurchase agreements. This arrangement allows banks to respond to liquidity pressures and is used by governments to assure basic stability in the financial markets. What is RTGS System? The acronym 'RTGS' stands for Real Time Gross Settlement. RTGS system is a funds transfer mechanism where transfer of money takes place from one bank to another on a 'real time' and on 'gross' basis. This is the fastest possible money transfer system through the banking channel. Settlement in 'real time' means payment transaction is not subjected to any waiting period. The transactions are settled as soon as they are processed. 'Gross settlement' means the transaction is settled on one to one basis without bunching with any other transaction.

What is Bancassurance? It is the term used to describe the partnership or relationship between a bank and an insurance company whereby the insurance company uses the bank sales channel in order to sell insurance products. What is Wholesale Price Index ? The Wholesale Price Index (WPI) is the index used to measure the changes in the average price level of goods traded in wholesalemarket. A total of 435 commodity prices make up the index. It is available on a weekly basis. It is generally taken as an indicator of the inflation rate in the Indian economy. The Indian Wholesale Price Index (WPI) was first published in 1902, and was used by policy makers until it was replaced by the Producer Price Index (PPI) in 1978. What is Consumer price Index(CPI)? It is a measure estimating the average price of consumer goods and services purchased by households. What is Venture Capital? Venture capital is money provided by an outside investor to finance a new, growing, or troubled business. The venture capitalist provides the funding knowing that theres a significant risk associated with the companys future profits and cash flow. Capital is invested in exchange for an equity stake in the business rather than given as a loan, and the investor hopes the investment will yield a better-than-average return. What is a Treasury Bills? Treasury Bills (T-Bills) are short term, Rupee denominated obligations issued by the Reserve Bank of India (RBI) on behalf of the Government of India. They are thus useful in managing short-term liquidity. At present, the Government of India issues three types of treasury bills through auctions, namely, 91-day, 182day and 364-day. There are no treasury bills issued by State Governments. What is Banking Ombudsmen Scheme? The Banking Ombudsman Scheme enables an expeditious and inexpensive forum to bank customers for resolution of complaints relating to certain services rendered by banks.The Banking Ombudsman is a senior official appointed by the Reserve Bank of India to redress customer complaints against deficiency in certain banking services.The Banking Ombudsman Scheme was first introduced in India in 1995, and was revised in 2002. The current scheme became operative from the 1 January 2006, and replaced and superseded the banking Ombudsman Scheme 2002. What is Subsidy? A subsidy is a form of financial assistance paid to a business or economic sector. Most subsidies are made by the government to producers or distributors in an industry to prevent the decline of that industry or an increase in the prices of its products or to encourage it to hire more labor. What is a Debenture? How many types of debentures are there? What are they? A debenture is basically an unsecured loan to a corporation. A type of debt instrument that is not secured by physical asset. Debentures are backed only by the general creditworthiness and reputation of the issuer. i)Convertible Debentures: Any type of debenture that can be converted into some other security or it can be converted into stock.. ii)Non-Convertibility Debentures(NCB): Non Convertible Debentures are those that cannot be converted into equity shares of the issuing company, as opposed to Convertible debentures. Non-convertible debentures normally earn a higher interest rate than convertible debentures do. What is a hedge fund? Hedge means to reduce financial risk. A hedge fund is an investment fund open to a limited range of investors and requires a very large initial minimum investment. It is important to note that hedging is actually the practice of

attempting to reduce risk, but the goal of most hedge funds is to maximize return on investment. What is FCCB? A Foreign Currency Convertible Bond (FCCB) is a type of convertible bond issued in a currency different than the issuers domestic currency. In other words, the money being raised by the issuing company is in the form of a foreign currency. A company may issue an FCCB if it intends to make a large investment in a country using that foreign currency. What is Capital Account Convertibility(CAC)? It is the freedom to convert local financial assets into foreign financial assets and vice versa at market determined rates of exchange. This means that capital account convertibility allows anyone to freely move from local currency into foreign currency and back.The Reserve Bank of India has appointed a committee to set out the framework for fuller Capital Account Convertibility.Capital account convertibility is considered to be one of the major features of a developed economy. It helps attract foreign investment. capital account convertibility makes it easier for domestic companies to tap foreign markets. What is Current Account Convertibility? It defines at one can import and export goods or receive or make payments for services rendered. However, investments and borrowings are restricted. What is Arbitrage? The opportunity to buy an asset at a low price then immediately selling it on a different market for a higher price. What is Capitalism? Capitalism as an economy is based on a democratic political ideology and produces a free market economy, where businesses are privately owned and operated for profit; in capitalism, all of the capital investments and decisions about production, distribution, and the prices of goods, services, and labor, are determined in the free market and affected by the forces of supply and demand. What is Socialism? Socialism as an economy is based on a collectivist type of political ideology and involves the running of businesses to benefit the common good of a vast majority of people rather than of a small upper class segment of society.

ome Economic , Banking and Financial Terms for all non - commerce backgound candidates :

Active Market This is a term used by stock exchange which specifies the particular stock or share which deals in frequent and regular transactions. It helps the buyers to obtain reasonably large amounts at any time. Administered Price The administrative body e.g., the government a marketing board or a trading group determines this price. The competitive market force are not entitled to determine this price. The government fixes a price in accordance with demand supply portion in the market.

Ad-valorem TaxAd-valorem tax is a kind of indirect tax in which goods are taxed by their values. In the case of ad-volorem tax, the tax amount is calculated as the proportion of the price of the goods. Value added Tax (VAT) is an ad-volorem Tax. Advanced Countries Advanced countries are countries which are industrially advanced, having high national and per capita income and ensure high rate of capital formation. These countries possess highly developed infrastructure and apply most updated and advanced technical know-how in their productive activities. A strong and well organised financial structure is found in these advanced countries. Amalgamation It means merger. As and when necessity arises two or more companies are merged into a large organisation. This merger takes place in order to effect economies, reduce competition and capture market. The old firms completely lose their identity when the merger takes place. Appreciation Appreciation means an increase in the value of something e.g., stock of raw materials or manufactured goods. It also includes an increase in the traded value of a currency. It is the antonym of Depreciation. When the prices rise due to inflation, appreciation may occur. It causes scarcity or increase in earning power. Arbitrage When a person performs functions of middle man and buys and sells goods at a particular time to cash the price differences of two markets, this action is termed as arbitrage. Purchases are made in the market where price is low and at the same time, goods are sold in other market where the price are high. Thus the middleman earns profit due to price difference in two markets. Arbitration Where there is an industrial dispute, the Arbitration comes to the force. The judgement is given by the Arbitrator. Both the parties have to accept and honour the Arbitration. Arbitration is the settlement of labour disputes that takes place between employer and the employees. Auction When a commodity is sold by auction, the bids are made by the buyers. Whose ever makes the highest bid, gets the commodity which is being sold. The buyers make the bid taking into consideration the quality and quantity of the commodity. Autarchy If a country is self-sufficient, it does not require the imports for the country. Autarchy is an indicator of self-sufficiency. It means that the country itself can satisfy the needs of its population without making imports from other countries. Automation Automation means the use of machinery & technology to replace the labours work. Automation increases the demand of skilled workers. Unskilled and semiskilled workers are reduced as a result of automation. Balanced Budget

When the total revenue of the government exactly equals the total expenditure incurred by the government, the budget becomes a balanced budget. But it is a conservative view point. In present days, the welfare government has to regulate a number of economic and social activities which increase the expenditure burden on the government and results in deficit budget. Balance of Payment Balance of payment of a country is a systematic record of all economic transactions completed between its residents and the residents of remaining world during a year. In other words, the balance of payment shows the relationship between the one country's total payment to all other countries and its total receipts from them. Balance of payment is a comprehensive term which includes both visible and invisible items. Balance of payment not only include visible export and imports but also invisible trade like shipping, banking, insurance, tourism, royalty, payments of interest on foreign debts. Balance of Trade Balance of trade refers to the total value of a country's export commodities and total value of imports commodities. Thus balance of trade includes only visible trade i.e., movement of goods (exports and imports of goods). Balance of trade is a part of Balance of payment statement. Balance Sheet Balance sheet is a statement showing the assets and liabilities of a business at a certain date. Balance sheet helps in estimating the real financial situation of a firm. Bank Bank is a financial institution. It accepts funds on current and deposit accounts. It also lends money. The bank pays the cheques drawn by customers against current and deposits accounts. The bank is a trader that deals in money and credit. Bank Draft Banker's draft is a negotiable claim drawn upon a bank. Drafts are as good as cash. The drafts cannot be returned and unpaid. Draft is issued when a customer shows his unwillingness to accept cheque in payment for his services or mercantile goods. Bank Draft is safer than a cheque. Bank Rate Bank Rate is the rate of discount at which the central bank of the country discounts first class bills. It is the rate of interest at which the central bank lends money to the lower banking institutions. Bank rate is a direct quantitative method of credit control in the economy. Bilateralism It implies an agreement between two countries to extend to each other specific privileges in their international trade which are not extended to others. Birth Rate Birth Rate (or Crude Birth Rate) is number of the births per thousand of the population during a period, usually a year. Only live births are included in the calculation of birth rate. Black Money

It is unaccounted money which is concealed from tax authorities. All illegal economic activities are dealt with this black Money. Hawala market has deep roots with this black money. Black money creates parallel economy. It puts an adverse pressure on equitable distribution of wealth and income in the economy. Blue Chip It is concerned with such equity shares whose purchase is extremely safe. It is a safe investment. It does not involve any risk. Blue Collar Jobs These Jobs are concerned with factory. Persons who are unskilled and depend upon manual jobs that require physical strain on human muscle are said to be engaged in Blue Collar Jobs. In the age of machinery, such Jobs are on the decline these days. Brain-Drain It means the drift of intellectuals of a country to another country. Scientists, doctors and technology experts generally go to other prominent countries of the world to better their lot and earn huge sums of money. This Brain-Drain deprives a country of its genius and capabilities. Bridge Loan A loan made by a bank for a short period to make up for a temporary shortage of cash. On the part of borrower, mostly the companies for example, a business organization wants to install a new company with new equipments etc. while his present installed company / equipments etc. are not yet disposed off. Bridge loan covers this period between the buying the new and disposing of the old one. Budget It is a document containing a preliminary approved plan of public revenue and public expenditure. It is a statement of the estimated receipt and expenses during a fixed period, it is a comparative table giving the accounts of the receipts to be realized and of the expenses to be incurred. Budget Deficit Budget may take a shape of deficit when the public revenue falls short to public expenditure. Budget deficit is the difference between the estimated public expenditure and public revenue. The government meets this deficit by way of printing new currency or by borrowing. Bull Bull is that type of speculator who gains with the rise in prices of shares and stocks. He buys share or commodities in anticipation of rising prices and sells them later at a profit. Bull Market It is a market where the speculators buy shares or commodities in anticipation of rising prices. This market enables the speculators to resale such shares and make a profit. Buoyancy When the government fails to check inflation, it raises income tax and the corporate tax. Such a tax is called Buoyancy. It concerns with the revenue from taxation in the period of inflation.

Business Cycle Business cycle (also known as trade cycle) are species of fluctuations in the economic activity of organised communities. It is composed of period of good trade characterised by rising prices and low unemployment, alternating with period of bad trade characterised by falling prices and high unemployment. Every trade cycle have five different subphases depression, recovery, full employment, prosperity (boom) and recession. Call Money Call money is in the form of loans and advances which are payable on demand or within the number of days specified for the purpose. Capital Budgeting Capital budgeting represents the process of preparing budget for a period of a year or even for several years allocating capital outlays for the various investment projects. In other words, it is the process of budgeting capital expenditure by means of an annual or longer period capital budget. Capital-labour Ratio Latest models of machinery and equipment raise the labour efficiency and the output is maximized. Capitallabour ratio is the amount of capital against the given labours that a firm employs. Capital-labour ratio is the ratio of capital to labour. Capital Market Capital market is the market which gives medium term and long term loans. It is different from money market which deals only in short term loans. Capitalism Capitalism is an economic system in which all means of production are owned by private individuals Selfprofit motive is the guiding feature for all the economic activates under capitalism. Under pure capitalism system economic conditions are regulated solely by free market forces. This system is based on Laissez-faire system i.e., no state intervention. Sovereignty of consumer prevails in this system. Consumer behaves like a king under capitalism. Cash Reserve Ratio (CRR) The commercial banks are required to keep a certain amount of cash reserves at the central bank. This percentage amount is called CRR. It influences the commercial banks volume of credit because variation in CRR affects the liquidity position of the banks and hence their ability to lend. Census Census gives us estimates of population. Census is of great economic importance for the country. It tells us the rate at which the total population is increasing among different age groups. In India census is done after every 10 years. The latest census in India has been done in 2001. Central Bank Central Bank may be defined as the apex barking and monetary institution whose main function is to control, regulate and stabilize the banking and the monetary system of the

country in the national interest. Cheque Cheque is an order in writing issued by the drawer to a bank. If the customer has sufficient amount in his account, the cheque is paid by the bank. Cheques are used in place of cash money. Clearing Bank Clearing bank is one which settles the debits and credits of the commercial banks. Even of the cash balances are lesser, clearing bank facilitates banking operation of the commercial bank. Clearing House Clearing house is an institution which helps to settle the mutual indebtedness that occurs among the members of its organisation. Closed Economy Closed economy refers to the economy having no foreign trade (i.e., export and import). Such economies depend exclusively on their own internal domestic resources and have no dependence on outside world. Collusion Producers of an industry reduce competition among themselves to raise their profits. They fix the price themselves with a clear understanding in this regard. This understanding among different firms is called collusion. Coinage Art and practice of making coins is called coinage. The metal is melted and moulded to shape into a coin. The coinage is a medium of exchange (money). Collectivism Collectivism is a belief that nation's interest is superior to individual interest. This is the collective thinking of the society and polity national leaders and also communist opine the theory of collection. Commercial Bank Commercial Bank is an institution of finance. It deals with the banking services through its branches in whole of the country. Operation of current accounts, deposits, granting of loans to individuals and companies etc. are various functions of the commercial bank. CommunismCommunism is a political and economic system in which the state makes the major economic decision State owns the bulk of capital assets. Responsibility for production and distribution lies with the state in this system. Core SectorEconomy needs basic infrastructure for accelerating development. Development of infrastructure industries like cement, iron and steel, petroleum, heavy machinery etc. can only ensure the development of the economy as a whole. Such industries are core sector industries. Corporation Tax It is a tax on company's profit. It is a direct tax which is calculated on profits after interest

payments and allowance (i.e., Capital allowance) have been deducted but before dividends are allowed for. Cost-push InflationIt arises due to an increase in production cost. Such type of inflation is caused by three factors : (i) an increase in wages, (ii) an increase in the profit margin and (iii) imposition of heavy taxation. Credit RationingCredit rationing takes place when the banks discriminates between the borrowers. Credit rationing empowers the bank to lend to some and to refuse to lend to others. In this way credit rationing restricts lending on the part of bank. Credit SqueezeMonetary authorities restrict credit as and when required. This credit restriction is called credit squeeze. Monetary authorities adopt the policy of credit squeeze to control inflationary pressure in the economy. Custom DutyCustom duty is a duty that is imposed on the products received from exporting nations of the world. It is also called protective duty as it protects the home industries. Cyclical UnemploymentIt is that phase of unemployment which appears due to the occurrence of the downward phase of the trade cycle. Such an employment is reduced or eliminated when the business cycle turns up again. Dear Money Dear money is that money which can only be borrowed at a high rate of interest. In dear money policy, bank rate and other rates of interest are high and as a result borrowing becomes expensive. Dear money policy is deliberate policy which is adopted by the monetary authorities to check inflation in the economy. Death Duty It is a direct tax which is imposed on the estate of deceased person. Death duty or Death Tax is a form of personal tax on property which is levied when property passes from one person to other at the time of death of the former. Death Rate Death rate signifies the number of deaths in a year per thousand of the population. It is mostly known as crude death rate. Life expectancy is important determinant of death rate. A country having high life expectancy will have a high crude death rate. DecentralisationDecentralisation means the establishment of various unit of the same industry at different places. Large scale organisation or industry can not be run at one particular place or territory. In order to increase the efficiency of the industry, various units at different places are located. Debt Service (Total) The sum of principal repayments and interest actually paid in foreign currency, goods and services on longterm debt (having maturity of more than one year), interest paid on shortterm debt and repayments to IMF. Deficit Financing It is a practice resorted to by modern government of spending more money than it receives in revenue. It is a policy of bridging a deficit between governments expenditure and revenue. Deliberately budgeting for a deficit is called deficit financing. This practice was popularised by Prof. J. M. Keynes to deal with the depression and unemployment situations and to stimulate

economic activity. Deficit financing, though having inflationary effects, has now become a common practice in all countries. Deflation Deflation is the reverse case of inflation. Deflation is that state of falling prices which occurs at that time when the output of goods and services increases more rapidly than the volume of money in the economy. In the deflation the general price level falls and the value of money rises. Devaluation The loss of value of currency of a country relative to other foreign currency is known as devaluation. Devaluation is a process in which the government deliberately cheapens the exchange value of its own currency in terms of other currency by giving it a lower exchange value. Devaluation is used for improving, the balance of payment situation in the country. Direct Tax A tax is said to be a direct tax when it is not intended to be shifted to anybody else. The person who pays it in the first instance is also excepted to bear it. Thus the impact and incidence of direct tax fall on the same person shifting of direct tax is not possible Income Tax is a example of direct tax. Disinflation It refers to a process of bringing down prices moderately from their high level without any adverse impact on production and employment. Thus, disinflation is an anti-inflationary measure. DissavingDissaving occurs when expenditure exceeds income. Raising of loans or utilization of past accumulated savings takes place in such eventuality. Dividend Dividend is the amount which the company distributes to shareholders when the profits of the company are calculated by the board of directors. Economic Integration Economic integration appears when two or more nations coordinate themselves and their economies are linked up. It may exhibit itself in the form of free trade area or a full economic union. EEC is an example of economic integration. Engel's Law This law was formulated by Ernst Engel. This law states that, with given taste and preference, the portion of income spend on food diminishes as income increases. According to this law, smaller a person's income, the greater the proportion of it that he will spend on food and vice versa. Estate DutyIt is a tax which is levied on the estate of a decreased person. It is also known as death duty. The ownership of state changes hands only after the payments of the estate duty. It is an progressive tax in nature. Excise Duty It is a tax which is imposed on certain indigenous production (e.g., petroleum products, cigarettes etc.) of the country. Excise duty may be imposed either to raise revenue or to check

the consumption of the commodities on which they are imposed. Excise duty is progressive in nature. Face ValueIt refers to that normal value of coin at which the coin circulates and is accepted in the discharge of debit or obligation. Broadly speaking, the face value refers to domination stamped on a coin / or documents when it is issued. In securities, it refers to par value. Fascism It is a form of political system. In it every economic consideration rests on one criterionthe increase in the people's standard of living. It also lays emphasis on military strength and prestige of the country. It is the extreme nationalism and the ultimate goal is self-sufficiency. Federal EconomyIt refers to a federation which is an association of two and more states. A federal state is a union of state in which authority is divided between the federal (or central) government and the state governments. In a federal economy both the centre and the states are independent in the exercise of this authority. Fiduciary IssueGenerally bank-note are backed by gold. But when they are not backed by gold and government securities replace gold, it is called fiduciary issue. Such fiduciary issue results in inflation. Fertility Rate The term fertility refers to the actual bearing of children or occurrence of births. Fertility rate measures the average number of the live births per 1000 women. This rate is one of the most important and useful aids to population projection. It helps in assessing population trends in the economy. Fiscal PolicyFiscal policy is that part of government economic policy which deals with taxation, expenditure, borrowing, and the management of public debt in the economy. Fiscal policy primarily concerns itself with the flow of funds in the economy. Fiscal policy primarily concerns itself with the flow of funds in the economy. It exerts a very powerful influence on the working of economy as a whole. GEM GEM (Gender Empowerment Measure) is a composite index measuring gender inequality in three basic dimensions of empowermenteconomic participation and decision making, political participation and decision making, and power over economic resources. GDI GDI (Gender Related Development Index) is a composite index measuring average achievement in the three basic dimensions captured in the human development indexa long and healthy life, knowledge and a decent standard of livingadjusted to account for inequalities between men and women. Gini-coefficientIt represents the measurement of inequality derived from the Lorenz Curve, with every increase in the degree of inequality, the curvature of the Lorenz Curve also increases and the area between the curve and 45 line becomes larger. The Gini-coefficient is measured as G =Area between Lorenz Curve & 45 Line/Area above the 45 Line Giffin Goods Giffin goods have the positive relationship between price and quantity demanded and as a

result demand curve of Giffin goods slopes upward from left to right. This phenomenon was first observed by Sir Robert Giffin in relation to the demand for bread by poor labours. Gresham's Law Bad money (if not limited in quantity) drives good money out of circulationThis statement was given by Sir Thomas Gresham, the economic Adviser of Queen Elizabeth. This law states that people always want to hoard good money and spend bad money when two forms of money are in circulation at the same time. Gross Domestic Product (GDP)It is the money value of all final goods and services produced within the geographical boundaries of the country during a given period of time (usually a year). GDP can be calculated both at current prices and at constant prices. If we add net factor income from abroad to the GDP, we get Gross National Product (GNP). Gross National Product (GNP) It refers to the money value of total output or production of final goods and services produced by the nationals of a country during a given period of time, generally a year. Gross National Product Deflator It is a Price Index Number used to correct the money value of Gross National Product (GNP) for price changes so as to isolate the changes which have taken place in the physical output of goods and services. Guild Socialism This form of socialism accepts the leadership of artisans. The operation of the whole economy specially the management and control of industries lies in the hands of artisans Socialism established by artisans is termed a Guild Socialism. HDI HDI (Human Development Index) is a composite index measuring average achievement in three basic dimensions of human lifea long and healthy life, knowledge and a decent standard of living. Import Duty Import duty is a tax on imports imposed on an ad-valorem basis i.e., fixed in the form of a percentage on the value of the commodity imported. Indirect Tax Indirect tax is that tax which is levied on goods or services produced or purchased. Indirect taxes are those which are demanded from one person in the expectation and intention that he shall indemnify himself at the expense to another. Inflation A situation of a steady and sustained rise in general prices is usually known as inflation. Inflation is a state in which the value of money is falling i.e., prices are rising. Joint Demand Joint demand appears in case of complementary goods. When two commodities are complementary to one another and cannot be used separately, they have joint demand. Bread and butter, sugar and tea, pen and ink are a few examples of joint demand. In joint demand a

change in demand of one commodity bring about the proportionate change in demand for the other. Joint Sector When a sector is jointly owned, managed and run by both public and private sector, it is called joint sector. This sector indicates the partnership between the two i.e., public and private sector. Labour Union Labour union represents that organisation of workers which works for improving working condition of labours and also for raising their wage by adopting collective bargaining measures with the management of the industry in particular. Laffer Curve This curve is given by American economist Prof. Arthur Laffer. It represents relationship between total tax revenue and corresponding tax rates. Laissez Faire It is a French word meaning non-interference. This doctrine was popularised by classical economists who gave the view that government should interfere as little as possible in the economic activities of the individuals. Life Expectancy at Birth The number of years a newborn infant would live if prevailing pattern of age specific mortality rates at the time of birth were to stay the same throughout the childs life. Liquidation It refers to the termination (or winding up) of a registered company. Liquidation takes place because of company's insolvency. In liquidation, assets are turned into cash for settling outstanding debts and for apportioning the balance, if any, amongst the owners. Liquidity Assets which can easily be converted into cash money are said to have liquidity. Land does not possess liquidity at it takes longer time to get converted into cash. Liquidity Ratio The commercial banks under banking regulations have to maintain a certain specified proportion of their total deposits of various categories in liquid assets. This maintainable proportion is called liquidity ratio. Lock-out Lock-out refers to such a situation when the management does not permit the workers to work unless they agree to accept the employer's term. Lock-out is the closing of work by the management for an uncertain period of time to put pressure on the labour union. It is an action by the employer equivalent to a strike by employees. Lorentz Curve This curve shows the degree of inequalities of a frequency distribution in a graphical manner. It is a curve on a graph which shows the cumulative proportion of a statistical population

against this cumulative share of some characteristic. This curve is commonly used to depict income distribution showing the cumulative percentage of people from the poorest up and their cumulative share of national income. Lump Sum Tax Lump sum tax is a fixed amount which has imperative nature irrespective of the income level. This tax is not equitable in nature. Merit Goods Merit goods refer those goods that are very essential to the society as a whole and hence the government ensures their availability to all consumers, regardless of their ability to pay to reasonable price. Mixed EconomyIt refers to that economic system in which both private and public sector coexists. Indian economy is an example of a mixed economy. Monetary Policy Monetary policy comprises all measures applied by the monetary authorities with a view to produce a deliberate impact on the nature and volume of money so as to achieve the objectives of general economic policy. It aims at regulating the flow of currency, credit and other money substitutes in an economy with a view to affect the total stock of such assets as well as to influence the demand of the community for such assets. Monetary Reforms When a new currency is introduced in a country due to hyperinflation or due to a deliberate policy measure (such as decimalization) it is termed as monetary reform. Monopoly Monopoly refers to that market structure where there is only one seller in the market who controls the entire market supply and no substitute of the product is available in the market. Monopsony Monopsony is that market situation in which there is only one single buyer of the product in the market. In other word, buyer's monopoly is termed as monopsony. Multinational Company It is a large scale company which has its production base in several countries and the bulk of the production is produced in outside nations. This company produces more overseas than they do in its parent country. Increased trade and economies of scale have encouraged such type of companies in the recent years. National Income In the simplest way it can be defined as factor income accruing to the national residents of a country. It is the sum of domestic factor income and net factor income earned from abroad. Net national product at factor cost is called national income. Net National Product (NNP) When depreciation is deducted from GNP i.e., Gross National Product, we get Net National Product (NNP).

Oligopoly Oligopoly is that form of imperfect competition in which there are only a few firms in the industry (or group) producing either homogeneous products or may be having product differentiation in a given line of production. Open Economy Open economy is that economy which is left free and the government imposes no restrictions on trade with areas outside that economy. Okuns Law Arthur Okun presented an empirical relationship between cyclical movements in GNP and unemployment. Okun found that an annual 25% increase in the rate of real growth above the trend growth results in a 1% decrease in the rate of unemployment. This relationship is known as Okuns Law. Perfect Competition Perfect competition is the market in which there are many firms selling identical products with no firm large enough relative to the entire market to be able to influence market price. Poverty Line Poverty line is a virtual line demarcating persons living below and above it. In India all those persons are treated living below poverty line who are not able to earn that much of income which is not sufficient to acquire food equivalent to 2100 calories per person per day in urban areas and 2400 calories per person per day in rural areas. As per UNDP, one US dollar (1993 PPP US $) per person per day is treated as poverty line. PQLI PQLI is known as Physical Quality of Life Index which is used to assess the level of social development. This index was developed by Jim Grant for The Overseas Development Council PQLI is calculated by using indices of (i) Adult literacy rate, (ii) IMR, (iii) Life Expectancy. Price Mechanism Price mechanism signifies the working of those market forces which establishes equilibrium in the economy. Laissez faire policy is the basis for the working of price mechanism. Price Ring It is an unofficial syndicate by which the prices are controlled with the prior understanding among the traders. These dealers under a price ring decide not to over-bid one another at the public auction to keep the prices low. This price ring may discourage outsiders from coming to the auctions. Private Sector Private Sector is that part of the economy which is not owned by the government and is under the hands of private enterprise. In other words, private sector is not under direct government control. Private sector includes the personal as well as the corporate sector. Privatisation Privatisation is the antithesis of nationalisation. When the government owned public industries are denationalised and the disinvestment process is initiated, it is called privatisation.

Public Debt Public debt represents borrowing by the state and public authorities. All loans taken by the public authorities constitute public debt. Public Goods Public goods are those goods which belong to the entire community. None of the individual of the society can be made deprived of using these public goods. National defence, Police, Street lighting etc. are examples of public goods. Public SectorPublic sector signifies those undertakings which are owned, managed and run by public authorities. Public sector includes direct government enterprise, the nationalized industries and public corporations. In this sector of the economy the government acts itself as an entrepreneur. Peril Point It indicates that point beyond which tariff reductions would threaten the existence of domestic industry. Quick Asset Those assets are quick assets which are liquid or nearly liquid in nature and easily be turned into cash. Quoted Company That company is called quoted company whose share prices are quoted on a stock exchange. Reflation It signifies general increase in the level of business activity in the economy. Reflation generally involves greater government expenditure and the easing of credit to encourage increased production. Regressive Tax It is a tax in which rate of taxation falls with an increase in income. In regressive taxation incidence falls more on people having lower incomes than that of those having higher incomes. Repressed Inflation It is a state in which aggregate demand is greater than the total supply of goods and services in an economy, but prices are prevented from rising to eliminate excess demand. The holding down of price is sometimes done by government as a means of suppressing inflation. Reserve Asset Ratio It is the ratio of a banks reserve assets to its eligible liabilities. Revolving Credit It is a bank credit that is renewed automatically until notice of cancellation is received. Revolving credits may be sanctioned for an unlimited amount in total but with a limit on the amount that may be drawn at any one time or within a specified period, e.g., one month. Seasonal UnemploymentIt is that unemployment which is caused by seasonal variation in

demand for labour by various industries, such as agriculture, construction and tourism. Seasonal unemployment normally declines in spring as more outdoor work can be undertaken. Security Security refers to a share, bond or government stock that can be bought and sold, usually on the stock exchange or on a secondary market, and carries a right to some form of income, either in the form of a fixed rate of interest or dividends. Shadow PriceIt is an imputed value for a good based on the opportunity costs of the resources used to produce it such values are of particular significance in resolving problems of resource allocating with respect to the effect on welfare. Share Capital It is the amount of money raised by a company by issuing shares. The authorized share capital is the amount that a company is allowed to issue as laid down in its Articles of Association. The issued share capital is the amount actually issued i.e., the number of issued shares multiplied by their par value. Fully paid share capital is the amount raised by payment of the full par value of the issued shares. Single Tax System It is a system in which all tax revenues are raised from one form of taxation. Socialism The political doctrine that the means of production (machines, materials and output) should be owned by society and specifically either by the state, as in the case of nationalized industries or by the workers directly, as in the case of producer co-operatives. Social SecurityProvision by the state out of taxation of welfare assistance to those in need as a result of illness, unemployment, or old age compare national insurance refers to social security. Soft CurrencyA currency with limited convertibility into gold and other currencies, either because it is depreciating due to balance of payments difficulties or because controls have been placed on it to prevent the exchange rate falling. Special Drawing Rights (SDRs) It is a reserve asset (known as Paper Gold) created within the framework of the International Monetary Fund in an attempt to increase international liquidity, and now forming a part of countries official reserves along with gold, reserve positions in the IMF and convertible foreign currencies. Special Tax (Unit Tax) It is a tax imposed per unit of a commodity rather than on the value of the commodity compare ad-valorem. Stabilization Policy It is Government economic policy announced at reducing the cyclical and other fluctuations that take place in a market economy. Stagflation

It is a state of the economy in which economic activity is slowing down, but wages and prices continue to rise. The term is a blend of the words stagnation and inflation. Surplus Value It is the difference between the amount paid to a factor and the revenue earned by selling the output it produced. Tariff It is a tax or a duty on imports, which can be levied either on physical units, e.g., per tonne (specific), or on value (ad-valorem). Tariffs may be imposed for a variety of reasons including; to raise government revenue, to protect domestic industry from subsidized or low-wage imports, to boost domestic employment, or to ease a deficit on the balance of payments. Trade Gap It signifies the size of the deficit (or surplus) in the balance of trade i.e., the difference in value between visible imports and exports. Trade Union It is an organisation of employees who join together to further their interests. Trade Unions negotiate on behalf of their members in collective bargaining with employers, and in the event of a dispute may put pressure on employers by withdrawing labour (i.e. strike) or by some less drastic form of action (i.e. go-slow, working to rule). Transfer PaymentIt is a payment made by public authority other than one made in exchange for goods or services produced. Transfer payments are not the part of National Income. Examples includes unemployment benefit and child benefits. Vital Statistics Vital statistics refers to those data which are associated with vital events of masses like birth, death, marriage divorce etc. VAT (Value Added Tax) VAT seeks to tax the value added at every stage of manufacturing and sale, with a provision of refunding the amount of VAT already paid at the earlier stages to avoid double taxation. In other words, the tax already paid can be claimed at the next stage of value addition. Wealth Tax Wealth tax is that tax which is imposed on the value of total assets but the wealth upto a certain limit is exempted from such tax. Welfare State It refers to a nation that provides to all at least the minimum standards in respect of education, health, housing, pensions and other social benefits. Wholesale Price Index Wholesale Price Index is that index which is calculated on the basis of wholesale prices. It is calculated in a similar way to the Retail Price Index.

1. Many a times we read in financial News papers about "Share Swap" done by Big Corporates.What is a "Share Swap" ? A Share Swap is a business takeover in which acquiring company uses its own stock to pay for the acquired company. 2 On the basis of Study done by the Govt. of India,various states were divided into two groups "High Poverty States" and "Low poverty States".Which of the following is a low poverty State ? a)Andhra Padesh b)Orissa c)Gujarat d)Karnataka e)Kerala 3. As reported in the major financial newspapers,the federal Reserve had made several changes / correction in its federal fund rates.It will bring effects directly on banks located in a)Russia b)USA c)Japan d)China e)France 4.Which of the following countries has the largest checques truncatin facility in Asia a)China b)India c)Sri Lanka d)Myanmar e)None of these 5.Which of the following is the name of asset reconstruction company created by various banks in India

a)AMFI b)ARCIL c)SEBI d)HCR e)None of these 6.India doesn't have a single branch office of any of its banks in the following countries ?

a)UK b)Russia c)USA d)Nepal e)Switzerland

7. Very Often we read in Newspapers about Money Market.What are the main activities of Money Market?

a)It Provide short term funds to government and Public sector organizations. b)businessman and others get short term funds to meet their day to day neds of the working capital c)Money market is nothing but only the other name of Forex market.Only foreign currencies are purchased and sold there

(1) Only A (2) Only b (3) Only C (4) Only A & B (5) None of these 8.)Who among the following are the beneficiaries of Ways and Means Advances (WMA) facility of Reserve Bank of India? (A)Commercial Banks (B)Regional Rural banks (C)State Governments (D)Authorized Dealers of Foreign Exchanges (E)All of above 9.Which among the following is an example of Arbitrage? (A)A firm of brokers purchases certain shares at BSE and sells the same number of shares at NSE to take advantage of minor difference in rates (B)A firm of currency brokers purchases US dollars and sells the British Pounds with the same value (C)A firm buys certain number shares of a company and sells similar number of shares of another company which is doing better in the market (D)A firm buys certain currency today and undergoes a contract to sell the same currency after two months (E)None of them 8.In context with the markets, what is dematerialization of securities? (A)Shortening the time of debt repayment on securities (B)Electric holding of the shares replacing the paper securities (C)The price of a shares falling below certain point in the share market (D)The buyback of shares by a company (E)Dividend distribution

What is Bank rate?

Bank Rate is the rate at which central bank of the country (in India it is RBI) allows finance to commercial banks. Bank Rate is a tool, which central bank uses for short-term purposes. Any upward revision in Bank Rate by central bank is an indication that banks should also increase deposit rates as well as Prime Lending Rate. This any revision in the Bank rate indicates could mean more or less interest on your deposits and also an increase or decrease in your EMI.

What is Bank Rate ? (For Non Bankers) :

This is the rate at which central bank (RBI) lends money to other banks or financial institutions. If the bank rate goes up, long-term interest rates also tend to move up, and viceversa. Thus, it can said that in case bank rate is hiked, in all likelihood banks will hikes their own lending rates to ensure and they continue to make a profit. What is CRR?

The Reserve Bank of India (Amendment) Bill, 2006 has been enacted and has come into force with its gazette notification. Consequent upon amendment to sub-Section 42(1), the Reserve Bank, having regard to the needs of securing the monetary stability in the country, can prescribe Cash Reserve Ratio (CRR) for scheduled banks without any floor rate or ceiling rate. [Before the enactment of this amendment, in terms of Section 42(1) of the RBI Act, the Reserve Bank could prescribe CRR for scheduled banks between 3 per cent and 20 per cent of total of their demand and time liabilities]. RBI uses CRR either to drain excess liquidity or to release funds needed for the economy from time to time. Increase in CRR means that banks have less funds available and money is sucked out of circulation. Thus we can say that this serves duel purposes i.e. it not only ensures that a portion of bank deposits is totally risk-free, but also enables RBI to control liquidity in the system, and thereby, inflation by tying the hands of the banks in lending money. What is CRR (For Non Bankers) :

CRR means Cash Reserve Ratio. Banks in India are required to hold a certain proportion of their deposits in the form of cash. However, actually Banks dont hold these as cash with themselves, but deposit such case with Reserve Bank of India (RBI) / currency chests, which is considered as equivlanet to holding cash with themselves.. This minimum ratio (that is the part of the total deposits to be held as cash) is stipulated by the RBI and is known as the CRR

or Cash Reserve Ratio. Thus, When a banks deposits increase by Rs100, and if the cash reserve ratio is 9%, the banks will have to hold additional Rs 9 with RBI and Bank will be able to use only Rs 91 for investments and lending / credit purpose. Therefore, higher the ratio (i.e. CRR), the lower is the amount that banks will be able to use for lending and investment. This power of RBI to reduce the lendable amount by increasing the CRR, makes it an instrument in the hands of a central bank through which it can control the amount that banks lend. Thus, it is a tool used by RBI to control liquidity in the banking system. What is SLR?

Every bank is required to maintain at the close of business every day, a minimum proportion of their Net Demand and Time Liabilities as liquid assets in the form of cash, gold and unencumbered approved securities. The ratio of liquid assets to demand and time liabilities is known as Statutory Liquidity Ratio (SLR). Present SLR is 24%. (reduced w.e.f. 8/11/208, from earlier 25%) RBI is empowered to increase this ratio up to 40%. An increase in SLR also restrict the banks leverage position to pump more money into the economy. What is SLR ? (For Non Bankers) :

SLR stands for Statutory Liquidity Ratio. This term is used by bankers and indicates the minimum percentage of deposits that the bank has to maintain in form of gold, cash or other approved securities. Thus, we can say that it is ratio of cash and some other approved to liabilities (deposits) It regulates the credit growth in India. What are Repo rate and Reverse Repo rate? Repo (Repurchase) rate is the rate at which the RBI lends shot-term money to the banks. When the repo rate increases borrowing from RBI becomes more expensive. Therefore, we can say that in case, RBI wants to make it more expensive for the banks to borrow money, it increases the repo rate; similarly, if it wants to make it cheaper for banks to borrow money, it reduces the repo rate Reverse Repo rate is the rate at which banks park their short-term excess liquidity with the RBI. The RBI uses this tool when it feels there is too much money floating in the banking system. An increase in the reverse repo rate means that the RBI will borrow money from the banks at a higher rate of interest. As a result, banks would prefer to keep their money with the RBI

BANKING GENERAL AWARENESS 2011 ONE 1. Paying for facilities like issuance of demand drafts, cheque books, credit or debit cards and ATM interchange may soon become pass as public sector lender ____ waived all service charges on current and savings accounts (CASA), the first by any bank. The move is aimed at increasing the banks below-industry average CASA deposits and the bank expects to double its retail account to 1 crore in the next 12 months. a) SBI b) PNB c) Canara Bank d) IDBI Bank e) None of these 2. At present, how many families are categorized as BPL? a) 5.10 cr b) 6.52 cr c) 7.81 cr d) 10.40 cr e) None of these 3. DTC stands for a) Direct Tax Code b) Disinvestment Tax Code c) Derivative Trade Code d) Distinct Tax Code e) Delhi Tax Code 4. In a surprise move and to the disappointment of taxpayers, the government on Aug 30 deferred the implementation of the Direct Taxes Code (DTC) by a year to ___, 2012 and sough to waive the preferential treatment to woman in tax payment in the name of gender equity. a) Jan 1 b) Mar 31 c) Apr 1 d) Dec 31 e) None of these 5. Which of the following statement is/are true about Direct Taxes Code (DTC)? A) Under the Direct Taxes Code Bill, 2010 tabled in the Lok Sabha by finance minister Pranab Mukherjee and referred to the Select Committee of Parliament for scrutiny the government has sought to raise the income tax exemption limit from Rs.1.6 Lakh to Rs.2 Lakh while retaining a host of incentives for individuals. B) While senior citizens (above 65 years) will enjoy a higher exemption of Rs.2.5 Lakh, woman taxpayers will have no additional relief as they have not been categorized separately. C) As for corporate taxes, the levy will be at a flat rate of 30 per cent with no surcharges or cesses. The minimum alternate tax (MAT)will be levied on book profits at 20 per cent. a) Only A b) Only A& B c) Only B&C d) All the above e) None of these 6. Name the country with India has signed a protocol to the Double Taxation Avoidance Agreement that would the wealth illegally stashed away in banks. a) Britain b) US c) Switzerland d) France e) None of these 7. In a major step to alleviate delays in wage payments under the Mahatma Gandhi National Rural Employment Guarantee Scheme (MGNREGS), which state govt has launched an e-Shakti Financial Inclusion scheme, the first of its kind in the country? a) UP b) MP c) Bihar d) Maharashtra e) None of these 8. Name the bank whose chairman Stephan K Green will be appointed as the minister of state for trade and investment. a) HSBC Holdings b) Standard Chartered Bank c) Barclays Bank d) Royal Bank of Scotland e) None of these

\ 9. In a big boost to the disinvestment programme of the UPA-II Government, the Petroleum and Natural Gas Ministry gave its green signal to the sale of governments stake in Oil and Natural Gas Corporation (ONGC) and Indian Oil Corporation (IOC), a move aimed at raising around __________ this fiscal. a) Rs. 14,000 cr b) Rs.18,000 cr c) Rs.24,000 cr d) Rs. 30,000 cr e) None of these 10. With a net worth of US$62 billion, Mukesh Ambani would be the richest man of earth in ___________, going past the current claimant, Mexican businessman Carlos Slim, according to a forecast by Forbes. a) 2012 b) 2014 c) 2016 d) 2018 e) None of these 11. In one of his speeches Pranab Mukherjee said that the Govt had no plans to dilute the roles of market regulators. This means the role of which of the following will not be diluted? a) Life Insurance Corporation of India (LIC) b) Confederation of Indian Industry (CII) c) Federation of Chambers of Commerce & Industry (FICCI) d) Bureau of Indian Standards e) Securities & Exchange Board of India (SEBI) 12. What is the full form of ULIP , the term which was in the news recently? a) Universal Life & Investment Plan b) Unit Loan & Insurance Plan c) Universal Loan & Investment Plan d) Uniformly Loaded Investment Plan e) Unit Linked Insurance Plan 13. As per the news published in some major financial news papers, the Maharashtra Govt is planning to introduce Green Tax on Vehicles. What is the Purpose of Green Tax? a) It has been introduced with the aim of discouraging high consumption of petroleum products. b) It is a tax levied by the Govt in place of VAT as VAT is not very profitable for State Govts. c) It is just like a toll tax which will be charges from all the vehicles whenever they enter green areas of a city like hospitals, schools, and old-age homes. d) This is a tax levied on big commercial vehicles only with the aim of prohibiting them from entering residential areas and non-commercial areas. e) None of these 14. Very often we read in newspapers about 3 G Third Generation Technology. This is a set of standards used by which of the following? a) To Combat climatic changes b) Production of nuclear energy c) Film production d) Mobile telecommunications e) All of these 15. Which of the following companies is NOT in the field of insurance business? a) ICICI Prudential b) Bajaj Allianz c) Tata AIG d) Aegon Religare e) Royal Orchild 16. Many times we read that a corporate entity is in the process of raising its capital base. Why is a company required to raise money to strengthen its capital base? A) To finance its diversification Plans B) To finance its diversification plans C) To repay its loans and borrowings a) Only A b) Only B c) Only C d) Only A & B e) All A, B&C 17. Many times we read in newspapers that a company is planning to bring a public issue. What does it mean? A) Shares of the company will be issued only through public sector organizations like banks/Central financial institutions etc. B) Shares of the company will be issued to general public only through primary market. C) This means some stakeholders/ promoters are willing to leave the company. Hence they wish to sell their stock to the general public. a) Only A b) Only B c) Only C d) All A,B&C e) None of these 18. Whenever some people wish to enter into the business world, it is a must for them to approach a bank. What services do banks provide them in this regard? A) Banks act as payment agents by operating current accounts, paying cheques and receiving

payments for them. B) Maintaining account books for them for their day-to- day activities so that they are not required to appoint account/finance personnel on a regular basis. C) Lending money by way of overdraft, installment loan, credit or advance for business activities a) Only A b) Only B c) Only C d) Only A&C e) All of these 19. As we all have noticed, banks these days are giving more emphasis on Branchless Banking. What does this really mean? A) Banks will not have many branches as used to be in the good old days. Instead, the number of branches will be restricted and will conduct only a specified core business. B) Banks will launch/operate many delivery channels like ATMs, Mobile Banking/ Internet Banking etc sot that people are not required to visit a branch for their usual banking needs. C) This means banks will issue only debit or credit cards for all types of day-to-day financial transactions. Cheques/cash payments will not be allowed. a) Only A b) Only B c) Only A&B d) Only B&C e) All A, B&C 20. Which of the following is NOT a function of a bank? a) Providing project finance b) Selling Mutual Funds c) Deciding policy rates like CRR, Repo Rate / SLR etc d) Settlement of payments of behalf of the customers e) All of these are functions of a bank 21. Which of the following is a form of savings Bank popular among the poor or children? a) Core Banking b) Credit Banking c) Debit Card d) Merchant Banking e) Piggy Banking 22. Dr. Amartya Sen is a famous a) Physicist b) Chemical Engineer c) Psychologist e) Economist e) None of these 23. Which of the following nations is considered the originator of the concept of Micro Finance? a) India b) Bangladesh c) South Africa d) USA e) None of these 24. Govt normally does not announce the Minimum Support Price (MSP) of a) Wheat b) Paddy c) Sugarcane d) Jute e) All of these 25. At present Dr.D Subbarao is the a) Governor of Orissa b) Deputy Chairman of the Planning Commission c) Chairman of the FICCI d) Indias Representative in IMF e) None of these 26. Banks borrow money from the RBI on which of the following rates? a) Bank rate b) CRR c) SLR d) Reverse Repo Rate e) Repo Rate 27. Which of the following is NOT a part of Indias Money Market? a) Bill Market b) Call Money Market c) Banks d) Mutual Funds e) Indian Gold Council 28. Which of the following is/are NOT the feature(s) of Indias Foreign Trade Policy (2004 to 2009)? A) To double Indias percent age share of global trade from present 0.7 per cent to 1.5 per cent by 2009 B) Simplifying the procedures and bringing down the cost C) Make SAARC countries Indias most preferred foreign trade partners by 2009 a) Only I b) Only II c) Only III d) All I, II and III e) Only I and III 29. Which of the following is /are the measure(s) taken by the Reserve Bank of India (RBI) to ease the liquidity crunch in the country? A) Cut in Cash Reserve Ratio and Statutory Liquidity Ratio. B) Increase the flow of foreign direct investment C) Supply of additional currency notes in the market. a) Only I b) Only II c) Only III d) All I, II and III e) None of these 30. Which of the following services is NOT provided by the post offices of India? a) Savings Bank Scheme b) Retailing of Mutual Funds

c) Sale of Stamp Papers (Judicial) d) Issuance of Demand Drafts e) Life Insurance Cover 31. Which of the following is true of New Lending Rate System in Banks? a) For existing loans, the base rate would be applicable once they matured. b) For new loans, the base rate would be levied immediately. c) Banks can not charge customers any fee for such switch-over. d) All of these e) None of these 32. According to recently released RBI data on the Balance of Payment, net portfolio investments into country during April- December amounted to a) $ 23.6 billion b) $ 30.4 billion c) $ 32.8 billion] d) $ 36.6 billion e) $ 40.4 billion 33. Which of the following is NOT a Government sponsored organization? a) Small Industries Development Bank of India b) NABARD c) National Housing Bank d) ICICI Bank e) All are government sponsored 34. The Reserve Bank has constituted a working group to review the current operating procedure of monetary policy, as transmission of the policy to the rest of the system remains imperfect. The group, to be headed by RBI executive director ___________, will also review the liquidity adjustment facility (LAF), through which the central bank manages money supply in the system on a day-to-day basis. a) Usha Thorat b) Deepak Mohanty c) Rakesh Mohan d) Naresh Chandra e) None of these 35. SEBI was established in a) 1993 b) 1992 c) 1988 d) 1990 e) None of these 36. The working of SEBI includes: a) To regulate the dealings of share market b) To check the foul dealings in share market c) To control the inside trading of shares d) All of these e) None of these 37. The Ad hoc Treasury Bill a system of meeting budget deficit in India was replaced by a system which came into force on a) March 31,1997 b) April 1,1996 c) April 1,1997 d) March 31, 1996 e) None of these 38. The recommendations of the 13th Finance Commission has become/will be operational during the period a) 2009-14 b) 2010 15 c) 2011 16 d) 2012 2017 e) None of these 39. Rural Woman can avail the benefit of Mahila Samriddhi Yojana if they open their account in a) Rural Pot Offices b) Commercial Banks c) Rural Development Bank d) Any of the above e) None of these 40. The rate at which banks lend to RBI is known as a) Bank Rate b) Repo Rate c) Reverse Repo Rate d) Interest Rate e) None of these 41. The cause of deflation is a) Lack of goods and services as compared to money supply b) Lack of imports as compared to exports c) Lack of money supply as compared to supply of goods and services d) Lack of money supply and goods e) None of these 42. Which bank in India performs the duties of the central bank? a) Central Bank of India b) State Bank of India c) Reserve Bank of India d) Bank of India e) None of these 43. Mixed Economy means a) Co-existence of small and large industries b) Promoting both Agriculture and Industries in the economy c) Co-existence of public and private sectors d) Co-existence of the rich and the poor e) None of these

44. Pure Banking, Northing Else is a slogan raised by a) ICICI Bank b) HDFC Bank c) SBI d) UTI Bank e) None of these 45. Which of the following is true about Swavalamban Scheme? A) Finance minister Pranab Mukherjee will launch the Swavalamban scheme in Murshidabad district of West Bengal. B) As per the scheme, the Centre will contribute Rs.1,000 a year to each New Pension Scheme (NPS) account opened in the current year. C) The contribution will be made till 2013 14. D) To be eligible, a person will have to make a minimum contribution of Rs.1,000 and maximum contribution of Rs.12,000 per annum. a) Only A,B & C b) Only A, C & D c) Only A & D d) All the above e) None of these 46. The axe finally fell on Asias first cooperative bank, ___________, when the Reserve Bank of India (RBI) cancelled its licence and asked it not to continue any banking related transactions until further directions. a) Saraswat Co-operative Bank Ltd b) Cosmos Co-operative Bank Ltd c) Nadar Co-operative Bank Ltd d) Anyonya Cooperative Bank Ltd e) None of these 47. 13th Finance Commission was constituted under the Chairmanship of a) YSP Thorat b) Vijai L Kelkar c) TS Vijayan d) Laxmi Narayan e) None of these 48. SEBI is a a) Statutory body b) Advisory body c) Constitutional body d) Non-statutory body e) None of these 49. Bank Cash Transaction Tax (BCTT) was withdrawn with effect from a) January 1, 2009 b) March 1, 2009 c) March 31, 2009 d) April 1, 2009 e) None of these 50. Which committee recommended abolition of tax rebates under Section 88? a) Chelliah Committee b) Kelkar Committee c) Shome Committee d) Naresh Chandra Committee e) None of these

38. Tarapore Committee submitted its report on Full Convertibility on Rupee in a) Current account b) Capital account c) Both on current as well as on capital account d) Special Drawing Rights (SDRs) e) None of these 39. CENVAT is related to a) Sales Tax b) Excise Duty c) Custom Duty d) Service Tax e) None of these 45. NABARD was established on the recommendation of a) Public Accounts Committee b) Shivaraman Committee c) Narasimham Committee d) Khandelwal Committee e) None of these 46. Sampurna Gramin Rozgar Yojana has been launched from a) 1 Apr 2001 b) 25 Sep 2001 c) 30 Sep 2001 d) No scheme of such title has yet been launched e) None of these 47. VAT is imposed a) Directly on the consumer b) On the final stage of production c) On the first stage of production d) On all stages between production and final sale e) None of these 48. Kutir Jyoti scheme is associate with a) Promoting cottage industry in villages b) Promoting employment among rural unemployed youth c) Providing electricity to rural families living below the poverty line

d) All of these e) None of these 49. CAPART is related with a) Assisting and evaluating rural welfare programmes b) Computer hardware c) Consultant service of export promotion. d) Controlling pollution in big industries e) None of these 50. The note issuing dept of RBI should always possess a minimum gold stock worth a) Rs.85 crore b) Rs.115 crore c) Rs.200 crore e) Rs.215 crore e) None of these **********

ANSWERS BANKING GENERAL AWARENESS ONE 1.D 2.B 3.A 4.C 5.D 6.C 7.C 8.A 9.C 10.B 11.E 12.E 13.E 14.D 15.E 16.A 17.B 18.D 19.E 20.C 21.E 22.D 23.B 24.D 25.E 26.E 27.E 28.D 29.A 30.E 31.D 32.C 33.D 34.B 35.C 36.D 37.C 38.B 39.A 40.C 41.C 42.C 43.C 44.C 45.D 46.D 47.B 48.A 49.D 50.B ***********************

BANKING GENERAL AWARENESS - TWO 1. Tarapore Committee submitted its report on Full Convertibility on Rupee in a) Current account b) Capital account c) Both on current as well as on capital account d) Special Drawing Rights (SDRs) e) None of these 2. CENVAT is related to a) Sales Tax b) Excise Duty c) Custom Duty d) Service Tax e) None of these 3. NABARD was established on the recommendation of a) Public Accounts Committee b) Shivaraman Committee c) Narasimham Committee d) Khandelwal Committee e) None of these 4. Sampurna Gramin Rozgar Yojana has been launched from a) 1 Apr 2001 b) 25 Sep 2001 c) 30 Sep 2001 d) No scheme of such title has yet been launched e) None of these 5. VAT is imposed a) Directly on the consumer b) On the final stage of production c) On the first stage of production d) On all stages between production and final sale e) None of these 6. Kutir Jyoti scheme is associate with a) Promoting cottage industry in villages b) Promoting employment among rural unemployed youth c) Providing electricity to rural families living below the poverty line d) All of these e) None of these 7. CAPART is related with a) Assisting and evaluating rural welfare programmes b) Computer hardware c) Consultant service of export promotion.

d) Controlling pollution in big industries e) None of these 8. The note issuing dept of RBI should always possess a minimum gold stock worth a) Rs.85 crore b) Rs.115 crore c) Rs.200 crore e) Rs.215 crore e) None of these 9. We very frequently read about Europes sovereign debt crisis these days. Which of the following statements/is/are true about the same? A) In early 2010 the Euro Crisis developed in some countries like Greece, Spain and Portugal. B) This created a credit default swap between the countries of the European Union. C) SAARC countries have offered some assistance to some of the severely affected countries like Portugal, Spain and Greece. a) Only A b) Only B c) Only A&B d) All A, B&C e) None of these 10. As we al know, the RBI raised the Cash Reserve Ratio (CRR) by 25 bps in April 2010. What action will banks will have to take to implement the same? A) They will have to deposit some more money with the RBI as reserved money. B) Banks will be required to give some more loan to projects of the priority sector. C) Banks will be required to give loan equivalent to the CRR to the Govt of India for its day-to-day expenditure as and when required. a) Only A b) Only B c) Only C d) Only A&B e) All A,B&C 11. As per estimates given by the RBI, the Economic Growth during the year 2010-11 will be at which of the following levels? a) 6.5% b) 7% c) 7.5% d) 8% e) None of these 12. Which of the following is/are correct about the RBIs credit policy announced in April 2010? A) Repo Rate raised by 25 bps B) A new reporting platform was introduced for secondary market transactions in Certificates of Deposits (CDs) and Commercial Papers. (CPs). C) Core Investment Companies (CICs) having an asset size of Rs.100 crore are required to register also with the RBI. a) Only A b) Only B c) Only C d) All A,B&C e) None of these 13. As per the announcement made by the RBI, some Stock Exchanges in India are allowed to introduce Plain Vanilla Currency Options. The term Plain Vanilla Currency Options is associated with which of the following activities / operations? a) Dollar-Rupee Exchange Rate b) Floating of Commercial Papers c) Launch of new mutual funds d) Deciding the opening price of a share on a particular business day e) None of these 14. Equity schemes managed strong NAV gains, which boost their assets was a news in some financial newspapers. What is the full form of the term NAV as used in above head lines? a) Nil Accounting Variation b) Net Accounting Venture c) Net Asset Value d) New Asset Venture e) None of these 15. As per reports published in various newspapers, mutual fund companies showed 94 per cent growth in their total profits during 2009-10. This means the profits earned by these companies were 94 per cent a) in their total investments they made collectively during the year b) more than their profits during previous year c) of the total capital of the company d) less than the total expenditure of the companies e) None of these 16. As per the news published in major newspapers/journals henceforth the Credit Card information though automated interactive voice response system over the phone instead of speaking to the staff. This decision of the banks/credit card companies will provide. A) An additional hurdle to the customers as people feel comfortable in talking to the staff instead of talking to a machine. B) An additional security to the customers as this does not allow any staff to handle any transaction directly. C) Some comfort to the banks as they will be able to reduce their staff strength. a) Only A b) Only B c) Only C d) All A, B&C e) Only A&C

17. As per the reports published recently, the Govt of India has decided to create a separate agency/body to scrutinize all Foreign Direct Investments (FDIs) officially. Uptil now the security screening was being done by the a) Ministry of Foreign Affairs, Govt of India b) Reserve Bank of India c) Securities & Exchange Board of India d) Ministry of Finance, Govt of India e) Ministry of Home Affairs, Govt of India 18. As a measure to help countries hit by global financial meltdown, the RBI has purchased notes worth US $10 billion in terms of Special Drawing Rights from a World Organisation/agency. Which is this agency / organisation? a) Federal Reserve of USA b) Asian Development Bank c) World Bank d) International Monetary Fund e) None of these 19. Which of the following is NOT a public sector undertaking under the Ministry of Defence, Govt of India? a) Paragon Dock Ltd. b) Garden Reach Shipbuilders & Engineers Ltd. c) Bharat Heavy Electricals Ltd. d) Hindustan Aeronautics Ltd. e) Bharat Earth Movers Ltd. 20. Which of the following is NOT as banking related term? a) Radiation b) Outstanding amount c) Benchmark Prime Lending Rate d) Explicit Guarantee e) None of these 21. Insurance service provided by various banks is commonly known as a) Investment Banks b) Portfolio Management c) Merchant Banking d) Bancassurance e) Micro Finance 22. Who amongst the following has recently taken over as the Chairman of the 19th Law Commission? a) AR Lakshmanan b) PV Reddy c) Yashwant Sinha d) Ranjan Bharti Mittal e) None of these 23. Name the senior member on the Board of Directors of RBI who will chair the sub-committee set up by the RBI to study issues and concerns of the micro-finance sector, including interest rates charged by the lenders in this area a. Subir Gokran b. YH Malegam c. Deepak Mohanty 4. Usha Thorat e. None of these 24. The meeting of finance ministers and central bank heads of G-20 in Oct-Nov 2010 was held in a. Tokyo b. London c. Seoul d. New Delhi e. None of these 25. Which of the following is not decided by RBI? a. Bank Rate b. Base Rate c. Repo Rate d. Cash Reserve Ratio e. None of these 26. Which of the following is not nationalized bank? a. Canara Bank b. Allahabad Bank c. IDBI Bank d. PNB e. BoB 27. Which of the following is not an instrument of money market? a. Call Money b. Treasury bill c. Certificate of deposit d. Share e. Repo 28. As per the latest data on electronic transactions released by the Reserve Bank of India, which of the following banks leads in electronic transactions in India? a. ICICI Bank b. HDFC Bank c. SBI d. Citibank e. None of these 29. Which of the following has appointed the Raghuram Rajan Committee to recommend steps to modernize the financial sector? a. Reserve Bank of India b. Ministry of Finance c. Planning Commission d. Minister of Home e. None of these 30. In which year, was IDBI restricted and separated from control of the Reserve Bank of India? a. 1972 b. 1976 c. 1978 d. 1980 e. None of these

31. State owned infrastructure lender India Infrastructure Finance Company Ltd (IIDCL) has inked a pact to buy loans worth Rs.1500 crore from _____ so that the state-run bank can utilize the funds for other projects in the crucial sector. a. SBI b. PNB c. BoB d. P & SB e. None of these 32. The Reserve Bank has increased the threshold limit for electronic fund transfers using the Real Time Gross Settlement (RTGS) system from `1 Lakh to ` a. 2 Lakh b. 3 Lakh c. 4 Lakh d. 5 Lakh e. None of these 33. Which of the following banks has requested `1,500 cr capital infusion from the govt.? a. Punjab National Bank b. Union Bank of India c. Indian Overseas Bank d. Dena Bank e. None of these 34. Which of the following is not associated with banking? a. Bank rate b. Repo rate c. Morbidity rate d. Base rate e. None of these 35. Name the bank which has sought the approval of the RBI to hive off its IT (Information Technology) related activities into an independent subsidiary? a. SBI b. PNB c. Indian Overseas Bank d. Bank of Baroda e. None of these 36. The Central Government on Nov 10, 2011 informed Parliament that allowing enhanced foreign direct investment (FDI) in multi-brand retail was now under its consideration. The existing policy allows _____ FDI in single-brand retail and _____ in cash-and-carry stores, which cater only to bulk buyers and traders. a. 26%, 49% b. 49%, 74% c. 51%, 100% d. 21%, 51% e. None of these 37. Which of the following is the body/agency set up by Govt. of India to increase the flow of foreign investment in the country? a. FEMA b. FCCB c. NSDL d. FIPB 38. Which of the following is not an instrument of capital market? a) Share b) Bond c) Debenture d) Repo e) All the above 39. What is SABLA? a) Anti-dowry police force b) A Womens organisation fighting for empowerment c) A woman welfare programme launched by the Central govt d) A helpline instituted by the National Commission for Women (NCW) e) None of these 40. Which of the following is associated with economy? a) Hindu Growth Rate (HGR) b) Islamic Saving Rate (ISR) c) Sikh Deposit Rate (SDR) d) Christian Investment Ratio (CIR) d) None of these 41. A new scheme for the health improvement of expectant mothers and ensuring proper nutrition for the newborns has been named as a) Indira Gandhi Matritva Sahyog Yojana b) Rajiv Gandhi Matritva Sahyog Yojana c) Mahatma Gandhi Matritva Sahyog Yojana d) Kasturba Gandhi Matritva Sahyog Yojana e) None of these 42. The central bank in India is a) Central Bank of India b) State Bank of India c) Bank of India d) Indian Bank e) Reserve Bank of India 43. Money supply could be increased when a) RBI purchases govt securities b) RBI sells govt securities c) CRR of banks is increased d) CAR is increased e) None of these 44. Corporation tax in India is a) Tax on production by companies b) Tax on the sales income of the companies c) Tax on net income of the companies d) Tax on net savings of the companies e) None of these 45. The function of money is/are

a) Medium of exchange b) Measure of Value c) Store of Value d) All of these e) None of these 46. Which among the following is not a type of inflation? a) Creeping inflation b) Walking inflation c) Running inflation d) Flying inflation e) All the above 47. In India, the bankers, bank is a) IMF b) World Bank c) IDBI d) BIS e) None of these 48. An appropriate measure of economic development is a) rate of increase in income b) improvement in Human Development Index c) decrease in poverty rate d) increase in poverty rate d) increase in life expectancy rate 49. Disinflation means a) increase in prices in the long run b) The prices increase due to export c) The increase in prices is more than average d) Increased prices decrease e) Level of unemployment increases 50. When rate of inflation increases a) Purchasing power of money increases b) Purchasing power of money decreases c) Purchasing power of money remains unaffected d) All (1), (2) and (3) e) Foreign exchange rate increases ******************************** ANSWERS BANKING GENERAL AWARENESS TWO 1.B 2.D 3.B 4.B 5.D 6.C 7.A 8.B 9.A 10.A 11.D 12.A 13.A 14.C 15.B 16.B 17.B 18.D 19.C 20.A 21.D 22.B 23.B 24.C 25.B 26.C 27.D 28.B 29.C 30.B 31.E 32.A 33.B 34.C 35.C 36.C 37.D 38.D 39.C 40.A 41.A 42.E 43.A 44.C 45.D 46.D 47.E 48.B 49.D 50.B ********************* BANKING GENERAL AWARENESS -- THREE 1. Under the National Rural Employment Guarantee Scheme guarantee is provided for a) One person in the family for 100 days in a year b) Every person in the family for 100 days in a year c) One person in the family for the whole year d) Every person for the whole year e) None of these 2. Devaluation of money means a) decrease in purchasing power of money b) increase in purchasing power of money c) decrease in external value of money d) decrease in external reserves of the country e) None of these 3. Who has been appointed as the new Chief Information Commissioner (CIC)? a) AN Tiwari b) Satyanand Mishra c) Wajahat Habibullah d) Meera Shankar e) None of these 4. Which of the following statement is/are correct about National Rural Employment Guarantee Act (NREGA)? A) NREGA is applicable to all rural areas of the country. B) Only construction work shall be given to the workers employed under this Act C) Every worker is given an amount of 250 per day for an eight-hour shift. a) Only A b) Only B c) Only C d) Only A&B

e) All, A , B&C 5. Which of the following organizations/agencies controls the monetary policy of our country? a) SBI b) Central Bank of India c) Indian Bank d) RBI e) None of these 6. How is dual economy distinguished from other economies? It is mixture of a) Industrial sector and manufacturing sector b) Traditional agricultural sector and a modern industrial sector c) State ownership of the means of production in cooperation with foreign organizations d) Industrial sector and trading of goods obtained through imports. e) None of these 7. Which of the following is the name of the organization created specifically to develop small scale industrial sector in our country? a) NABARD b) SEBI c) SIDBI d) AMFI e) None of these 8. Who is Nandan Nilekani? a) Chairman, Unique Identification Authority of India b) Chairman, Airport Authority of India c) Member, UGO d) Member, Planning Commissioner e) None of these 9. Which of the following organization/agencies ha specifically been set up to boost overall rural development in India? 10. Which of the following organizations maintains the Rural Infrastructure Development Fund (RIDF) a) RBI b) NABARD c) SIDBI d) LIC e) None of these 11. Which of the following is / are true about the Competition Commission of India (CCI)? A) CCI is a regulatory body having quasi-judicial structure. B) It has been established to replace old Monopolies and Restrictive Trade Practices Act. C) Its main objective is to create a healthy environment in corporate world. a) Only A b) Only B c) Only C d) All A, B&C e) None of these 12. According to RBI, which one of the following is the biggest concern of the country? a) Debt crisis in Europe b) Inflation c) GDP growth d) Fiscal deficit e) None of these 13. Which of the following organization has recently launched Farmers Information Service via SMS to help farmers in Gujarat state? a) State Bank of India b) SIDBI c) GDP growth d) Fiscal deficit e) None of these 14. Which of the following is NOT a Cash Crop? a) Tobacco b) Cotton c) Coffee d) Bajara d) Tea 15. The Central Government has approved additional capital infusion of Rs.6000 cr in 10 public sector banks with an objective to raise its holding to a minimum of ----- % in all state run banks. a) 51 b) 55 c) 58 d) 61 e) none of these. 16. The government had announced infusion of Rs.15000 in the Budget 2010-11 to ensure that the capital adequacy ratio (CAR) of all the public sector banks increase to a) 7% b) 9% c) 10% d) 12% e) none of these. 17. The RBI has declared to buy Rs.12000 cr ($2.7 billion) of government bonds through ----------------- in a movie to inject liquidity into countrys banking system. a) Repo b) Open market operation (OMO) c) reverse repo d) SLR e) CRR

18. Name the institution in which RBI has recently sold its stake to the government. a) EXIM Bank b) NHB c) IDBI d) NABARD e) None of these. 19. As we know, the RBI is the apex bank of India. Similarly, the apex bank of USA is called. a) Federal Reserve b) Federal Bank of US c) Bank of America d) National Bank of US e) None of these. 20. ICRA, CRISIL and Standard and Poors (S & P) are a) credit rating agencies b) NGOs c) Financial institution d) NBFCs e) None of these. 21. Reverse repo rate is the rate a. At which the RBI lends short term funds to Banks. b. At which banks park their short-term funds with the RBI. c. At which the RBI lends short term funds to Central Government. d. At which the Central Government lends funds to the RBI. e. None of these. 22. One of the following banks has its head quarters in Kolkata: a. IOB b. SBI c. UCO d. BOB 23. The foreign banks have been restrained by a consumer court from charging an interest of more than _______ per cent a year from credit card holders. a) 10 b) 20 c) 30 d) 40 e) None of these 24. LLP stands for a) Limited Liability Partnership b) Limited Loyalty Partnership c) Liable Limited Partnership d) Lower Level Partnership e) None of these. 25. The RBI has opposed the setting up of an independent ________ to manage the debt of the government. The argument given has been that currently the govt has a high fiscal deficit, so it is not a good time to set it up. a) Debt Management Office b) Asset Reconstruction Company c) Debt Recovery Tribunal d) Debt Management tribunal e) None of these. 26. NABARD is a a) Commercial bank b) development bank c) cooperative bank d) Regional rural bank e) None of these. 27. The headquarters of the World Bank is situated at a) Washington DC b) Geneva c) New York d) Paris e) none of these. 28. Which Bank has decided to sell its retail and commercial business in India as part of the restructuring exercise, a move that follows the bank recording biggest annual loss of 24 billion pounds. a) Royal Bank of Scotland b) Citibank c) HSBC d) Yes Bank e) none of these 29. Which of the following will decrease inflation? a) Truckers strike b) Rise in the price of manufactured goods c) Cut in fuel prices d) All of the above e) none of these

30. ESMA stands for a) Essential Services Maintenance Act b) Essential Supply Maintenance Act c) Estimated Supply Maintenance Act d) Early Services Maintenance Act e) none of these. 31. SEBI was established in a) 1993 b) 1992 c) 1988 d) 1990 e) 1994 32. The working of SEBI includes a) To regulate the dealings of share market b) To check the foul dealings in share market c) To control the insider trading of shares d) All of these e) None of these. 33. The leader of the Indian money market is: a. Commercial banks b. Lead banks c. Reserve Bank of India d. SEBI. e. None of these. 34. Which of the following is referred to as the National Bank? a. Punjab National Bank b. National Housing Bank c. NABARD d. IDBI e. None of these. 35. How many public sector commercial banks are there in India at present? a. 27 b. 20 c. 22 d. 14 e. None of these. 36. Which of the following activities to be undertaken by a bank require registration under the Securities & Exchange Board of India (Bankers to Issue) Rules & Regulations, 1994? a. Bankers to public/right issues. b. Collection of allotment/call monies. c. Payment of dividend/interest/refund orders d. All of these. e. None of these. 37. In terms of section 6(1)(o) of the Banking Regulation Act, as notified by the Government of India from time to time, commercial banks are permitted to undertake the business of: a. Leasing, hire purchase and factoring, b. Insurance c. Mutual funds d. All of these. e. None of these. 38. Which of the following guidelines were issued by Reserve Bank of India in January 1993 for the entry of private sector banks in the wake of Narasimham Committee recommendations? a. The new bank, upon being granted licence under the Banking Regulation Act, by RBI, shall be registered as a public limited company under the Companies Act, 1956. b. Its inclusion in the Second Schedule to the Reserve Bank of India Act, 1934 shall be subject to Reserve Banks decision. c. Preference would be given to those banks the headquarters of which are proposed to be

located in the centre, which does not have the headquarters of any other bank. d. all of these. e. None of these. 39. As per the revised guidelines issued by RBI on 3rd January 2001: a. the minimum paid up capital requirement of a newly set up private sector bank should be at least Rs.200 crore and it should be raised to Rs.300 crore within three years of commencement of business. b. The minimum capital adequacy ratio for newly set up private sector banks has been fixed at 10 per cent on a continuous basis from the commencement of operations. c. both a) and b). d. the minimum paid up capital requirement of a newly set up private sector bank should be at least Rs.50 crore and it should be raised to Rs. 200 crore within three years of commencement of business. e. none of these. 40. Under Merchant Banking, consultancy services are rendered by banks/merchant bankers for: a. floating of new companies b. preparation, planning and execution of new projects c. giving expert guidance and managing the new floatations of new promotions of industries/enterprises. d. all of these. e. None of these. 41. Major functions undertaken by merchant bankers are: a. Issue Management b. Market maker in capital market c. capital structuring/restructuring. d. all of these. e. None of these 42. Merchant bankers also carry out functions such as: a. private placement of capital. b. short term working capital from commercial banks. c. assistance to corporate sector in raising long-term loans in foreign exchange or in rupees from the national state level financial institutions. d. all of these. e. None of these. 43. Which of the following statements about merchant banking business being conducted by banks are correct? a. SEBI guidelines prohibit blank forms bearing stamps of other brokers etc. being kept for marketing purposes within the branch premises. b. SEBI (Bankers to the Issue) Rules & Regulations, 1994 require the bank to provide efficient services to the companies as well as to the investors. c. Large float funds remain with the bank for around a month. d. all of these. e. None of these. 44. The term mutual fund refers to: a. contribution by the employees of a business enterprise in the provident fund scheme. b. the business of acquisition, holding, management, trading or disposal of securities, participation certificates or any other instruments, income or growth participation business and Unit Trust Schemes. c. fund created by banks and other financial institution for floating new shares in the market to earn a higher profit. d. all of these. d. none of these.

45. When the scheme of a mutual fund is considered as close-ended? a. the corpus is of fixed size with a redemption period. b. the stocks are listed on the stock exchanges, thus, offering easy liquidity. c. the market price is always below net asset value. d. all of these. e. None of these. 46. An offshore fund is: a. a mutual fund, which is situated at seashore. b. a fund, which is meant for development of oceans. c. a fund established outside the country. d. none of these. e. None of these.

47. The term back-end load in mutual fund parlance refers to: a. too much of load on a ship. b. load factor added to a currency rate c. a fee charged from the unit holders at the time of redemption of units. d. none of these. e. b & c only 48. Leasing is a contract between a lessor and a lessee: a. for use of specific asset. b. for a pre-determined period by the lessee. c. on a specified lease rent payable to lessor. d. all of these. e. None of these. 49. Banks are permitted to undertake financing of equipment leasing, hire purchase and factoring: a. through separately established subsidiaries. b. departmentally c. only by certain selected branches and this should be suitably publicized for the benefit of bank clientele. d. all of these. e. None of these. 50. The advantages of leasing are: a. Unlike term loans (where the borrower has to contribute a margin), it is 100 per cent finance. b. The leased assets as well as the lease obligations do not figure in the balance sheet of the lessee and thus his debt-equity ratio is not disturbed by availing such finance. c. Leasing allows the lessee the use of the asset acquired at todays cost by making payment in future out of future earnings. d. all of these. e. None of these ***************************** BANKING GENERAL AWARENESS - THREE 1.A 2.C 3.B 4.A 5.D 6.B 7.C 8.A 9.C 10.B 11.D 12.B 13.D 14.D 15.C 16.D 17.A 18.D 19.A 20.A 21.B 22.C 23.C 24.A 25.A 26.B 27.A 28.A 29.C 30.A 31.C 32.D 33.C 34.C 35.A 36.D 37.D 38.D 39.C 40.D 41.D 42.D 43.D 44.B 45.D 46.C 47.C 48.D 49.D 50.D *********************************

BANKING GENERAL AWARENESS -2001 - FOUR

1. Under lease finance business: a. The component representing the replacement cost of the assets should be carried to the balance sheet in the form of provision for depreciation. b. Banks should follow prudential accounting standard norms.] c. Full depreciation should be provided for during the primary lease period of the assets. d. all of these. e. None of these. 2. The work relating to recovery of funds locked up in credit sales falls under Factoring. Factoring is: a. a means of providing post-shipment finance to exporters. b. a type of agriculture financing for purchase of tractors. c. a means of financing traders and manufacturers by taking over the collection of their customers bills. d. Factoring is opening a new factory. e. none of these. 3. Which of the following statement about factoring are correct? a. The factor agent either buys the book debts of client or makes an advance against a charge over the debts. b. He then collects the money from his clients customers direct. c. Sometimes the factor assumes the whole of the credit risk, but in other cases he stipulates for a right of recourse against his client in case the debtors fail to pay. d. all of these. e. None of these. 4. In the case of financing of equipment leasing, hire purchase and factoring by a bank: a. 100 per cent risk weight should be given for calculation of capital to risk assets ratio. b. Income recognition, asset classification and provisioning norms apply. c. The exposure norm of not extending loans in excess of 20 per cent of the banks capital funds to an individual borrower and 50 per cent to a group of borrowers will apply. d. all the above will apply. e. none of these. 5. Portfolio Management Services provided by banks to their clients are: a. in the nature of investment consultancy/management, for a fee b. entirely at the customers risk. c. without guaranteeing, either directly or indirectly, a predetermined return. d. all of these. e. None of these 6. A company is treated as Non-Banking Financial Company: a. if its financial assets are more than 50 per cent of total assets (netted off against intangible assets). b. income from financial assets is more than 50 per cent of the gross income as evidenced from its last audited balance sheet. c. both the above.

d. none of the above. E. NONE OF THESE. 7. Non-Banking Financial Companies consist of: a. equipment leasing company, and, investment company. b. hire purchase finance company and mutual benefit financial company. c. Loan company and Housing Finance Company d. all of these. e. none of these. 8. Banks are not allowed to give bank credit to NBFC against: a. bills discounted/rediscounted by NBFCs except for rediscounting of bills discounted by NBFC arising from sale of commercial vehicles. b. investments made by NBFCs in shares, debentures of a current nature i.e. stock-in-trade except to stock broking companies. c. investments of NBFCs in and advances to subsidiaries, group companies or other entities. d. all of these. e. None of these. 9. An NBFC having a certificate of registration and otherwise entitled to accept public deposits is allowed to open its branch/office or allow its agents to operate for mobilization of public deposits: a. within the State where its registered office is situated if its net owned funds is upto Rs.50 crore. b. any where in India if its NOF is more than Rs.50 crore and its fixed deposits program has been rated by one of the approved credit rating agencies at AA or above. c. both of these. d. either a or b) . e. none of these. 10. Which of the following statements about Portfolio Management Services rendered by banks are correct? a. Funds accepted for portfolio management from their clients should not be entrusted to another bank for management. b. The bank should charge a definite fee for services rendered independent of the return to the client. c. The banks own investments and investments belonging to Portfolio Management client should be kept distinct from each other. d. all of these e. none of these. 11. The recommendations of the committee on Banking Sector Reform (popularly known as the Narasimham Committee-II), submitted in April 1998, included: a. 5 per cent weight for market risk for Government/approved securities should be provided. b. The risk weight for Government guaranteed advances to be the same as for other advances. c. Foreign exchange open position limit to carry 100 per cent risk weight. d. all of these. e. none of these. 12. Which of the following is not a recommendation of the Committee on Banking Sector Reforms (Narasimham Committee-II)? a. A minimum target of 9 per cent CAR should be achieved by the year 2000and 10 per cent by 2002. b. An asset should be classified as doubtful if it is in the sub-standard category for 18 months in the first instance and eventually for 12 months and loss if it has been so identified but not written off. c. The Debt Recovery Tribunals should have territorial jurisdiction of the High Court in the concerned state. d. The Government guarantee advances, which have turned sticky, should be classified as NPAs. e. none of these. 13. Which of the following statements are correct about banks entry into insurance sector: a. All banks entering into insurance business will be required to obtain prior approval of the

Reserve Bank. b. The Reserve Bank will give permission to banks on case-to-case basis keeping in view all relevant factors. c. Holding of equity by a promoter bank in an insurance company or the IRDA/Central Government will subject to compliance with any rules and regulations lay down participation in any form in insurance business. d. All of these. e. none of these. 14. Guidelines issued by RBI on banks entry into insurance sector include: a. Any Scheduled Commercial Bank would be permitted to undertake insurance business as agent of insurance companies on fee basis, without any risk participation. b. The subsidiaries of banks will be allowed to undertake distribution of insurance produce on agency basis. c. A subsidiary of a bank or of another bank will not normally be allowed to join the insurance company on risk participation basis. d. all of these. e. none of these. 15. Banks will be permitted to set up a joint venture company for undertaking insurance business with risk participation if: a. the maximum equity contribution of participating bank in the joint venture company does not normally exceed 50 per cent of the paid-up capital of the insurance company. b. the net worth of the bank is not less than Rs. 500 crore with the CAR at least 10 per cent as on 31st March 2000. c. the bank has recorded net profit for the last three continuous years and the level of nonperforming assets is reasonable as on 31st March 2000. d. all of these. e. none of these. 16. Under which category Regional Rural Banks are classified? a. Public sector banks. b. co-operative banks. c. private sector banks. d. Investment development bank. E. none of these. 17. Paid-up share capital of Regional Rural Bank is contributed by: a. Central Government only. b. State Government only. c. Central Government, State Government and the sponsor commercial bank in the ratio of 50:15:35 respectively. d. NABARD, the concerned State Government and the sponsor commercial bank in the ratio of 50:25:25 respectively. e. None of these. 18. Where is the headquarters of State Bank of India? a. Mumbai b. Chennai c. Delhi d. Kolkata e, none of these. 19. Name the Governor of R.B.I.:(March 12, 2011) a. Bimal Jalan b. T.N. Seshan c. Duvvuru Subba Rao d. Sam Pitroda e. None of these. 20. . According to Banking Regulation Act, RBI can fix SLR upto the ceiling of--a. 40 per cent b. 30 per cent c. 20 per cent d. 50 per cent. E. None of these. 21. . What is rate of interest allowed by Banks in Savings Bank Account?

a. 3.5% p.a. b. 4% p.a. c. 3.5% p.m. d. 4% p.m. e. none of these 22. The expansion for the BIS, in the context of the banking industry is: (A) Bank for International Settlements (B) Bank for Industrial Settlements (C) Bank for Industrial Sectors (D) Bank for International Services (e) none of these, 23. The expansion for BIFR, in the context of the Indian Industry is: (A) Board for Industrial and Financial Reconstruction (B) Bureau for Industrial and Financial Reconstruction (C) Board for Investment and Financial Reconstruction (D) Bureau for Investment and Financial Reconstruction e. None of these. 24. Cash Reserve Ratio ( CRR) and Statutory Liquidity Ratio (SLR ) are terms most closely related to which of the following industries/markets: (A) Stock Exchange (B) Banking (C) Mutual Fund (D) Income Tax e. None of these. 25. Which of the following programs aims at the promotion of savings among rural women? (A) Rashtriya Mahila Kosh (B) Mahila Samriddhi Yojna (C) Indira Mahila Yojna (D) Javhar Rozgar Yojna e. none of these. 26. Which of the following programs meets the credit needs of poor women? (A) Mahila Samriddhi Yojna (B) Rashtriya Mahila Kosh (C) Indira Mahila Yojna (D) Mahila Samakhya Programme e. None of these. 27. Which of the following deals with economic offences? (A) MISA (B) NSA (C) TADA (D) COFEPOSA e. None of these. 28. Which state has maximum branches of public sector commercial banks? a. U.P. b. Maharashtra c. Karnataka d. Gujarat e. None of these 29. Which institution is known as soft loan window of World Bank? a. IFC b. IDA c. IMF d. Indian Development Forum e. None of these. 30. Meera Seth Committee was related to --a. Development of Handlooms b. Sex-differentiation in employment c. Abolition of Child Labour

d. Welfare of working women e. None of these. 31. National Income estimates in India is prepared by --a. Planning Commission b. RBI c. Finance Ministry d. C.S.O. e. None of these. 32. Individual cannot open his account in --a. Urban CO-operative Bank b. Regional Rural Bank c. RBI d. Scheduled Commercial Banks e. None of these 33. What is NIKKEI? a. Share Price Index of Tokyo share market b. Name of Japanese Central Bank c. Japanese name of Countrys Planning Commission d. Foreign Exchange Market of Japan e. None of these 34. NABARD was established on the recommendation of --a. Public Account Committee b. Sivaraman Committee c. Narsimham Committee d. a) and b) e. None of these 35. OTCEI is --a. Atomic submarines of China b. Economic policy of USA c. An Indian Share Market d. A defence research organization e. none of these. 36. What is Hawala? a. Full details of a subject b. Illegal transactions of foreign exchange c. Illegal trading of shares d. Tax evasion e. none of these. 37. Foreign currency which has a tendency of quick migration is called --a. Scarce currency b. Soft currency c. Gold currency d. Hot currency e. None of these. 38. The Headquarter of Asian Development Bank (ADB) is at ---

a. Singapore b. Manila c. Honk Kong d. Tokyo E. None of these. 39. What is Stagflation? a. Inflation with growth b. Deflation with growth c. Inflation after deflations d. Inflation with depressions e. None of these 40. Insider Trading is related to --a. Share Market b. Horse racing c. Taxation d. Public Expenditure e. None of these. 41. Public Sector means --a. Government ownership on commerce and trade b. Capitalist ownership on commerce and trade c. Private ownership on trade d. company owned by public. E. None of these 42. What is a limited company? a. In which shareholders possess the ownership limited to their paid up capital b. In which shares are not issued c. A company of Government ownership d. A Registered company 43. Who gains in inflation? a. Saver b. Creditor c. Debtor d. Pension holder 44. Open market operation is a part of --a. Income Policy b. Fiscal Policy c. Credit Policy d. Labour Policy 45. CRISIL --a. Evaluates the credit documents of a company b. Provides financial assistance to industries c. Provides finances for rural areas d. Promotes exports 46 GIC includes various insurance companies. This number is --a. 2 b. 3 c. 4 d. 5

47 LIC of India was established in a. 1897 b. 1950 c. 1956 d. 1965 48. Consumer Protection Act (COPRA) was implemented in --a. 1985 b. 1986 c. 1987 d. 1988 49. Which direct tax gives maximum net revenue to the Government --a. Corporate Tax b. Income Tax c. Wealth Tax d. Gift Tax 50 . Scheduled Bank is that bank which is --a. Nationalized b. Not nationalized c. Based at foreign country d. Included in the second schedule of RBI ******************************** ANSWERS GENERAL AWARENESS 2011 FOUR 1.D 2.C 3.D 4.D 5.D 6.C 7.D 8.D 9.C 10.D 11.D 12.C 13.D 14.D 15.D 16.A 17.C 18.A 19.C 20.A 21.A 22.D 23.A 24.C 25.A 26.B 27.D 28.A 29.B 30.A 31.D 32.C 33.A 34.B 35.C 36.B 37.D 38.B 39.D 40.A 41.A 42.A 43.C 44.C 45.A 46.C 47.C 48.B 49.A 50.D

BANKING GENERAL AWARENESS -FIVE 1. Match the following --Unit I Unit II a. Aqua culture 1. Silk b. Floriculture 2. Grapes c. Seri culture 3. Flower d. Viti culture 4. Fisheries a. b. c. d. a. 4 3 2 1

b. 3 4 1 2 c. 3 4 2 1 d. 4 3 1 2 e. None of these. 2 . The ownership right of a company remains --a. With company Directors b. With equity shareholders c. With debenture holders d. With the government e. None of these. 3 . Which is true for Finance Commission? a. It is a statutory body b. It is constituted under article 280 of the constitution c. Mr. K.C. Pant was Chairman of 10th Finance Commission d. All of these e. None of these. 4. The Govt. of India keeps on clearing the proposal of setting up new SEZs at several places. What does the letter S represent in the SEZ ? (A) Soft (B) Small (C) Special (D) Supportive (e) none of these. 5 Various Govt. agencies/organizations are given responsibilities for implementing various policies/decisions of the Govt. of India. Policies about credit flow to the agricultural/priority sector are framed/implemented through which of the following apex bodies ? (A) ECGC (B) NABARD (C) UTI (D) IDBI Bank e. None of these. 6 . Which of the following is not a foreign bank working in India ? (A) HSBC (B) Barclays (C) Standard Chartered (D) Yes Bank e. none of these. 7 . Who amongst the following is the Chairman of the 13th Finance Commission ? (A) Bimal Jalan (B) Y. V. Reddy (C) C. Rangarajan (D) Vijay Kelkar e. none of these. 8. Names of two important persons are often visible in ewspapers/magazines/electronic media. These names are Dr. C. Rangarajan and Dr. Bimal Jalan. Which of the following facts are identical in the life of both these two persons ? (A) Both are principal economic advisors to the Prime Minister (B) Both are the Members of the Lok Sabha (C) Both are the former Governors of RBI

(D) Both were the Finance Ministers of India (e) none of these. 9. What is phishing? a) A fraudulent way of acquiring PIN numbers and bank passwords using internet (email) by claming to be a trusted brand. b) Unsolicited e-mail c) Net surfing without any specific purpose d) An ingenious way of countering the hacking mechanism 10. For opening a new branch, a Regional Rural Bank requires: a. permission of NABARD b. permission of Director, Institutional Finance. c. RBI licence. d. approval of DRDA. E. none of these. 11 Which of the following statements about RRBs are correct? a. Sponsor banks travellers cheques can be issued by RRBs. b. RRBs can enter into arrangements with the sponsor banks for providing remittance facilities to its customers. c. Where RRBs can afford the investment, they can install lockers also. d. all of these. 12. The Regulatory Authority for Regional Rural Banks is: a. Central Government. b. State Government. c. RBI and NABARD d. Sponsor Bank 13. Regional Rural Banks: a. can issue guarantee against 100% cash margin or 50% cash margin plus collateral. b. have been free from Service Area Approach. They can operate in the entire area of their jurisdiction. Commercial banks will take over the villages allotted to RRBs under SAA. c. are permitted to open Extension counters at the premises of the borrowers, if they are principal banker to the borrower. d. all of these. 14. The risks covered under the Standard Policies of E.C.G.C. are: a. cancellation of export licence or imposition of new export licensing restrictions in India (under contracts policy) b. payment of additional handling, transport or insurance charges occasioned by interruption or diversion of voyage, which cannot be recovered, from the buyer. c. any other cause of loss occurring outside India, not normally insured by commercial insurers, and beyond the control of the exporter and/or the buyer. d. all of these. 15. Commercial risks are covered by ECGC subject to: a. approval of credit limit on each buyer. b. clearance by Reserve Bank of India. c. approval of financing bankers. d. confirmation by Controller of Imports & Exports. 16. The Standard Policies issued by ECGC cover the losses arising out of: a. imposition of restriction on remittances by the government in the buyers country or any

government action which may block or delay payment to the exporter. b. war, revolution or civil disturbances in the buyers country. c. new import licensing restrictions or cancellation of valid import licence in the buyers country. d. ALL OF THESE. . 17. The Standard Policies issued by ECGC do not cover losses due to: a. commercial disputes raised by the buyer, unless the exporter obtains a decree from a competent court of law in the buyers country in his favour; b. causes inherent in the nature of goods, c. buyers failure to obtain necessary import or exchange authorization from authorities in his country d. all of these. 18. The Deposit Insurance and Credit Guarantee Corporation provides insurance cover in respect of bank deposits with a view to give a measure of protection to depositors, particularly to small depositors, from the risk of loss of their savings arising from: a. the death of the depositor b. the death of the joint party to a deposit account c. bank failures d. dispute between the survivor and the legal heirs of the deceased account holder. 19. What is the Bank Rate now? (12.03.2011) a) 6% b) 5% c) 4% d) 9% 20. What is Cash reserve ratio now? (12.03.2011) a) 5.% b) 6% c) 6.5% d) 8% 21. When did the Reserve Bank of India become a State-owned institution? a 1943 b 1947 c 1949 d 1969 e) none of these 22. Who is the Chairman of Planning commission of India? a) D V Subba Rao b) Montek Singh Ahluwalia c) Dr. Manmohan singh d) M V Narayanamurthy 23. Which of the following organizations/banks has done a commendable work in the field of micro finance and was awarded Nobel Prize also in the past ? (A) Gramin Bank of Bangladesh (B) CRY (C) ASHA (D) NABARD (E) None of these 24. Nobel Prizes are not given for the performance in the area of (A) Literature (B) Physics (C) Chemistry (D) Music (E) Medical Science 25. Which of the following is considered an informal method of getting credit/finance ? (A) Internet Banking (B) Branch visits (C) Going to money lenders (D) Tele Banking

(E) All of these 26. Which of the following is not a Public Sector Unit/Undertaking/Agency ? (A) ECGC (B) SEBI (C) SIDBI (D) Axis Bank (E) BHEL 27. Which of the following does not represent the name of a bank working in India ? (A) YES (B) HDFC (C) TATA (D) Kotak Mahindra (E) Axis 28. As per the news appeared in some major newspapers India is number two in getting PE. What is the full form of PE as used in the financial world ? (A) Professional and Ecofriendly (B) Profitable Equity (C) Profitable and Economical (D) Private and Economical (E) Private Equity 29. Many a times we read in newspapers about Financial Inclusion. What does it really mean ? [Pick up correct statement(s)] 1. Allow the merger and acquisition of banks so that only few big banks exist and continue to cater to the need of corporate sector. 2. Expending the network of banks in such a way that people from lower strata of society also get the benefit of services provided by banks. 3. Providing Insurance cover to each and every citizen so that he/she can live a healthy and long life. (A) Only 1 (B) Only 2 (C) Only 3 (D) Both 1 and 2 (E) All 1, 2 and 3 30. Which of the following organizations recently issued some guidelines related to Participatory Notes as used in financial world ? (A) RBI (B) IRDA (C) SEBI (D) AMFI (E) None of these 31. As per news items published in various newspapers, Italy has offered its help in developing SMEs in India. What is the full form of SME ? (A) Small and Monopolistic Economy (B) Small and Medium Enterprises (C) Speedy and Mechanical (D) Small and Medium Level Economy (E) None of these 32. The Reserve Bank of India (RBI) plans to engage self help groups (SHGs) and primary agriculture credit societies (PACS) as business correspondents (BCs) to ensure greater banking

penetration in the country interior rural belt. The banking regulator has decided to make use of the banking correspondent model extensively to provide banking facility to some ____________ unbanked villages of population in excess of 2,000. a) 1 lakh b) 3 lakh c) 5 lakh d) 7 lakh e) None of these 33. How many banks were nationalised in 1969? a) 10 b) 14 c) 19 d) 6 e) None of these 34. Which among the following banks was not nationalized in 1969 or 1980? a) Punjab National Bank b) Canara Bank c) Corporation Bank d) IDBI Bank e) Allahbad Bank 35. Which of the following is not a subsidiary of the SBI? a) State Bank of Patiala b) State Bank of Indore c) State Bank of Travancore d) State Bank of Hyderabad e) Bank of Maharashtra 36. In which year did Indias economic liberalization start? a) 1984 b) 1991 c) 1999 d) 2004 e) None of these 37. Which of the following is not a feature of open economy? a) Non-interference by govt b) Competition c) Demand and Supply d) Consumers benefit e) Administered price 38. Which of the following is the most advocating agency for economic globalization? a) WTO b) World Bank c) IMF d) EU e) None of these 39. Which of the following is the core of the Basel-II norms? a) Non-Performing Assets b) Capital Adequacy Ratio c) Prime Lending Rate d) Priority Sector Lending e) None of these 40. The ________, armed with a $ 20 - million grant from Bill & Melinda Gates Foundation, announced to launch a financial scheme for helping farmers hit by the economic crisis in developing nations, including India. a) IMF b) World Bank c) ADB d) SAARC e) None of these 41. Which of the following institutions is the apex body responsible for overall development, policy, planning and financial support of agriculture and rural development? a) National Bank for Agriculture and Rural Development b) National Agriculture Cooperative Marketing Federation of India c) Indian Farmers Fertilisers Cooperative Organisation. 42. The term Paper Gold refers to a) Paper currency b) gold of highest purity c) Special Drawing Rights. d) oil bonds e) yellow shining paper. 43. Who among the following is responsible for the execution of the export and import policies of the Government of India? a) Chairman, State Trading Corporation b) Director-General of Trade and Commerce c) Chief Controller of Imports and Exports d) Finance Commission e) Ministry of Commerce 44. Consider the following relationships. A) Deficit budgeting-Inflation B) Consumer Price Index-Inflation C) Export Concessions-Fiscal Policy d) Income-Standard of Living

E) Income Tax-Monetary Policy Which of the above relationship is not correct? a) A and D b) B and E c) Only C d) A and C e) B and D 45. Taxes are collected by the Government with the basic objective(s) of A) Curbing the growing of black money. B) Establishing the coercive authority of the Government. C) Financing Government plans and expenditure D) Reducing inequalities in the society. a) A and B only b) B and C only c) C and D only d) A and D only e) A, B, C and D 46. Which of the following is/are the functions of the Securities and Exchange Board of India? A) To register and regulate collective Investment plans of mutual funds. B) To regulate the business of stock exchange and other securities market. C) To safeguard the interest of investors. D) To regulate capital market with suitable measures. a) A and C only b) B and D only c) A, B and C only d) B, C and D only e) All A, B, C and D 47. Which of the following is/are the objective of the World Trade Organisation? A) To enlarge production and trade of services. B) To enlarge production and trade of goods. C) To improve standard of living of people in the member countries D) To ensure full employment and increase in effective demand a) A only b) A and B only c) A, B and C only d) B and C only d) All A, B, C and D 48. Which of the following is/are not the function(s) of the Reserve Bank of India? A) To regulate cash and credit system of India B) To keep Foreign Exchange Reserve C) To issue five rupee coins and notes D) To formulate the monetary policy. a) B only b) D only c) A and C only d) B and D only e) All A, B, C and D 49. Which of the following economies did China surpass to become the Worlds third largest economy? a) Japan b) South Korea c) The UK d) Germany e) France 50. Which of the following is a function of the National Development Council? a) To review the working of the National Plan from time to time. b) To fix the targets of growth for various economy sectors. c) To prepare a general framework of the Five Year Plan. d) To see to the implementation of the plan e) None of these ************************************* ANSWERS BANKING GENERAL AWARENESS FIVE 1.D 2.B 3.D 4.C 5.B 6.D 7.D 8.C 9.B 10.C 11.D 12.C 13.D 14.D 15.A 16.D 17.D 18.C 19.A 20.B 21.C 22.C 23.A 24.D 25.C 26.B 27.C 28.E 29.B 30.C 31.B 32.A 33.C 34.D 35.E 36.B 37.E 38.A 39.B 40.B 41.A 42.C 43.C 44.C 45.C 46.E 47.E 48.B 49.D 50.A

**************** BANKING GENERAL AWARENESS - SIX 1. Very often we see in the advertisements published by Financing Institutes/Agencies stating that their products are given high or average Ratings. These Rating Agencies classify bonds/investments in how many categories ? 1. Low Risk 2. Average Risk 3. High Risk (A) Only 1 (B) Only 2 (C) Only 3 (D) All 1, 2 and 3 (E) None of these 2. As per the reports published in various newspapers/journals etc. the economic map of the world is being redesigned. What is/are the new emerging trends of this new economic map of the world ? 1. With India and Asia at the forefront the centre of economic gravity has shifted to East. 2. Emerging markets, with India among the leaders, are growing faster than old established markets. 3. Central banks of the countries have recognized that case-by-case liquidity solutions are not the answers and a market wide and globally synchronized approach is needed to solve the present crisis. (A) Only 1 (B) Only 2 (C) Only 3 (D) All 1, 2 and 3 (E) None of these 3. As per the decision taken by the Govt. of India, the exporters of which of the following products to USA and European countries will get an incentive of 2% on their exports ? 1. Leather goods 2. Garments 3. Software (A) Only 1 (B) Only 2 (C) Only 3 (D) Both 1 and 2 only (E) All 1, 2 and 3 4. Which of the following is/are the key features of Indian Economy which were highlighted during the presentation of the Interim Budget 2009-10 ? 1. Despite global financial crisis the GDP growth rate in current financial year has been 71 per cent. 2. Indian economy was adjudged as second fastest growing economy in the world 3. A provision of Rs. 100 crores is made in the annual plan 2009-10 for setting up Unique Identification Authority of India. (A) Only 1 (B) Only 2 (C) Only 3 (D) All 1, 2 and 3

(E) None of these 5. Many a times we read a term in financial news papers SEPA. What is the full form of the term ? (A) Single Exchange Processing Agency (B) Single Euro Payments Area (C) Single Electronic Processing Agency (D) Super Electronic Purchase Agency (E) None of these 6. As per the newly launched Indira Gandhi National Widow Pension Scheme a monthly pension will be given to the widows. What will be the amount of the pension ? (A) Rs. 200 (B) Rs. 400 (C) Rs. 600 (D) Rs. 800 (E) None of these 7. As per the decision taken by the Govt. of India all the Public Sector Banks (PSBs) will be recapitalised over the next two years so that they can maintain a Capital Adequacy Ratio (CAR) of . (A) 9% (B) 12% (C) 11% (D) 22% (E) None of these 8. As per the Industrial Development Report 2009 Indias rank in Industrial Development was 54 on the Development Index. Which of the following organizations/agencies compiles Industrial Development Index of the world ? (A) International Labour Organisation (B) United Nations Industrial Development Organisation (C) World Bank (D) World Trade Organisation (E) The Economic and Social Council of UNO 9. As per the news published in major news papers/magazines the International Finance Corporation has offered a loan for providing clean drinking water to 30 lakhs people in rural India. What is the amount of the loan ? (A) 5 million US $ (B) 10 million US $ (C) 15 million US $ (D) 20 million US $ (E) None of these 10. The merger of which of the following two Indian companies took place in recent past which is termed as Largest ever merger in Indias Corporate History ? (A) Nuclear Power Corporation and National Thermal Power Corporation (B) State Bank of India and its seven associate banks (C) Hind Aluminium Company and Century Mills

(D) Tata Motors and Ashok Leyland (E) Reliance Industries and Reliance Petroleum Ltd. 11. We very frequently read about the activities of the Foreign Exchange Market in newspapers/magazines. Which of the following is/are the major functions of the same ? 1. Transfer of purchasing power from domestic to foreign market. 2. Providing credit for financing foreign trade. 3. Power to purchase gold from foreign countries as most of the nations still work on Gold Standards. (A) Only 1 (B) Only 2 (C) Only 3 (D) Both 1 and 2 only (E) Both 2 and 3 only 12. The Govt. of India recently introduced Money Laundering Bill 2009. Under the bill which of the following agencies is empowered to search the premises after the offence is committed and a case is filed ? (A) Income Tax Department (B) Vigilance Department of the Reserve Bank of India (C) Enforcement Directorate (D) Central Bureau of Investigation (E) None of these 13. Which of the following is/are the components of the Fiscal Deficit ? 1. Budgetary Deficit 2. Market Borrowings 3. Expenditure made from Pradhan Mantri Rahat Kosh (A) Only 1 (B) Only 2 (C) Only 3 (D) All 1, 2 and 3 (E) None of these

14. Which of the following is not a Welfare scheme launched by the Govt. of India ? (A) Village Grain Bank Scheme (B) Sampoorna Gramin Rozgar Yojana (C) Annapurna Scheme (D) Midday Meal Scheme (E) Bharat Nirman Yojana 15. Which of the following are the instruments of Credit Control in the hands of the RBI ? 1. Lowering or raising the discount and interest rates. 2. Raising the minimum support price of the major agro products. 3. Lowering or raising the minimum cash reserves maintained by the commercial banks. (A) Only 1 (B) Only 2 (C) Only 3 (D) Both 1 and 3 only (E) Both 2 and 3 only

16. Which of the following programmes was launched to further improve the facilities of irrigation in rural India ? (A) National Social Assistance programme (B) Construction of Kiosks for poor (C) Sampoorna Grameen Rozgar Yojana (D) Annapurna Scheme (E) National Watershed Development Programme 17. Many times we read in newspapers that a company is planning to bring a public issue. What does it mean? A) Shares of the company will be issued only through public sector organizations like banks/Central financial institutions etc. B) Shares of the company will be issued to general public only through primary market. C) This means some stakeholders/ promoters are willing to leave the company. Hence they wish to sell their stock to the general public. a) Only A b) Only B c) Only C d) All A,B&C e) None of these 18. Whenever some people wish to enter into the business world, it is a must for them to approach a bank. What services do banks provide them in this regard? A) Banks act as payment agents by operating current accounts, paying cheques and receiving payments for them. B) Maintaining account books for them for their day-to- day activities so that they are not required to appoint account/finance personnel on a regular basis. C) Lending money by way of overdraft, installment loan, credit or advance for business activities a) Only A b) Only B c) Only C d) Only A&C e) All of these 19. As we all have noticed, banks these days are giving more emphasis on Branchless Banking. What does this really mean? A) Banks will not have many branches as used to be in the good old days. Instead, the number of branches will be restricted and will conduct only a specified core business. B) Banks will launch/operate many delivery channels like ATMs, Mobile Banking/ Internet Banking etc sot that people are not required to visit a branch for their usual banking needs. C) This means banks will issue only debit or credit cards for all types of day-to-day financial transactions. Cheques/cash payments will not be allowed. a) Only A b) Only B c) Only A&B d) Only B&C e) All A, B&C 20. Which of the following is NOT a function of a bank? a) Providing project finance b) Selling Mutual Funds c) Deciding policy rates like CRR, Repo Rate / SLR etc d) Settlement of payments of behalf of the customers e) All of these are functions of a bank 21. How many banks were nationalized in 1969? (A) 16 (B) 14 (C) 15 (D) 20 (E) None of these. 22. The Reserve Bank of India was established in : (A) 1820

(B) 1920 (C) 1935 (D) 1940 (E) None of these. 23. The first Indian Bank was : (A) Traders Bank (B) Imperial Bank (C) Presidency Bank of Calcutta (D) Indian Bank (E) None of these. 24. The rupee coin was first minted in India in : (A) 1542 (B) 1601 (C) 1809 (D) 1677 (E) None of these. 25. The Export-Import (EXIM) Bank was set up in : (A) 1980 (B) 1982 (C) 1981 (D) 1989 (E) None of these. 26. Planning Commission is : (A) Advisory body (B) Executive body (C) Government body (D) Autonomous body (E) None of these. 27. The Community Development Programme was launched in : (A) 1950 (B) 1952 (C) 1956 (D) 1960 (E) None of these. 28. The highest body which approves the Five-Year Plan is the : (a) Finance Ministry (B) Lok Sabha (C) Rajya Sabha (D) National Development Council (E) None of these. 29. Which of the following commodities earn maximum foreign exchange for India? (A) Jute (B) Iron and Steel (C) Tea (D) Sugar (E) None of these. 30. The one rupee note bears the signature of :

(A) Secretary, Ministry of Finance (B) Governor, Reserve Bank of India (C) Finance Minister (D) Prime Minister (E) None of these. Computer awareness (31-50) 31. The component used in second generation computers is 1. vacuum tube 2. transistors 3. integrated circuits 4. microprocessors 5. None of these 32. Transistors were placed on 1. integrated circuits 2. microprocessors 3. silicon chips 4. vacuum tubes 5. None of these 33. The third generation computers were interfaced with. 1. machine language 2. assembly language 3. operating system 4. high-level language 5. None of these 34. Which one of the following component used in fourth generation computers? 1. Microprocessors 2. Integrated circuits 3. Transistors 4. Vacuum tubes 5. None of these 35. The fifth generation computing devices are based on . 1. internet 2. artificial intelligence 3. high-level languages 4. VLSI 5. None of these 36. The third generation computers was developed during the period . 1. 1940- 1956 2.1961-1963 3.1964-1971 4.1971-present 5. None of these 37. Thousands of integrated circuits were built on to a single silicon chip called . 1. transistors 2. integrated circuits 3. artificial intelligence 4. microprocessors 5. None of these 38. UNIVAC stands for 1 Universal Automatic Computing 2. Universal Auto Computer 3. Universal Automatic Computer 4. Union Automatic Computer 5. None of these 39. In computer, data is converted into information through.. 1. instructions 2. processing 3. data 4. facts 5. None of these 40. Integrated circuits were used in..generation of computers. 1 first 2. second 3. third 4. fourth 5. None of these 41. The first generation computers used..fro circuitry a. vacuum tube b. transistors c. integrated circuits d. microprocessors e. None of these 42. Which one of the following is defined as an un-processed collection of raw facts? a. Information b. Processing c. Instructions d. Data

e. None of these 43. Which is a collection of facts from which conclusions may drawn? a. Data b. Information c. Processing d. Instructions e. None of these 44. Information is .. a. processed facts b. in active c. technology based d. set of instructions e. None of these 45. Data are. a. processed facts b. active c. business based d. stored facts e. None of these 46. A set of instructions or procedures for solving problems is defined as a. data b. information c. algorithm d. processing e. None of these 47. Which one of the following is designed to systematically solve a problem? a. Information b. A computer program c. Algorithm d. Data e. None of these 48. A computer system has major components a. 4 b. 3 c. 2 d. 5 e. None of these 49. Which one of the following refers to all the physical components associated with the computer system? a. Hardware b. Software c. Information d. Data e. None of these 50. Software is a set of.. a. data b. raw Facts c. instructions d. information e. None of these *****************************

ANSWERS BANKING GENERAL AWARENESS SIX 1.D 2.A 3.D 4.D 5.B 6.A 7.B 8.B 9.C 10.E 11.D 12.E 13.A 14.E 15.C 16.E 17.B 18.D 19.E 20.C 21.B 22.C 23.C 24.A 25.B 26.A 27.B 28.D 29.C 30.C 31.C 32.C 33.B 34.A 35.B 36.C 37.D 38.C 39.B 40.C

41.A 42.D 43.B 44.A 45.D 46.C 47.B 48.C 49.A 50.C ****************************************************

BANKING GENERAL AWARENESS - SEVEN Questions based on latest RBI Policy 1. In the context of monitoring for verification of end use of funds for loans sanctioned by banks, which of the following is not part of RBI guidelines on the issue : a) meaningful scrutiny of the periodical progress reports and operating / financial statements of the borrowers; b) regular visits to the assisted units and inspection of securities charged / hypothecated to the banks; c) periodical scrutiny of the books of accounts of the borrowers; d) None of the above 2. In the context of monitoring for verification of end use of funds for loans sanctioned by banks,

which of the following is part of RBI guidelines on the issue: a) introduction of stock audits depending upon the extent of exposure; b) obtention of certificates from the borrowers that the funds have been utilized for the purposes approved. c) examination of all aspects of diversion of funds during internal audit / inspection of the branches and at the time of periodical reviews. d) all the above 3. A non banking financial company with carries on the business of acquisition of shares and securities and satisfies certain conditions, is called : a) Investment company b) financial company c) non bank finance company d) core investment company 4. A core investment company must hold not less than ____ of its net assets in the form of investment in equity shares, preference shares, bonds, debentures, debt or loans in group companies. a) 100% b) 90% c) 74% d) 60% 5. The investment made by core investment company in the equity shares (including instruments compulsorily convertible into equity shares within a period not exceeding 10 years from the date of issue) in group companies must constitute not less than ___ of its net assets : a) 100% b) 90% c) 74% d) 60% 6. A core investment company cannot invest in which of the following: a) bank deposits, b) money market instruments, including money market mutual funds c) govt. securities d) none of the above 7. A core investment company can undertake which of the following activity : a) investment in bonds or debentures issued by group companies; b) granting of loans to group companies c) issuing guarantees on behalf of group companies. d) all the above 8. A systemically important core investment company means a core investment company fulfilling the following conditions (a) Having total assets of not less than Rs.100 crore, either individually or in aggregate along with other Core Investment Companies in the Group; (b) Raises or holds public funds; (c) makes investment in bank deposit or govt. securities. a) a, b and c all b) a and b only c) a and c only d) b and c only 9. In the context of system of speed Clearing which of the following is correct (a) it leverages on Core Banking Solutions implemented in banks across the country (b) it facilitates realization of outstation cheques drawn on CBS enabled branches at the local centre itself (c) it obviates the need of such cheques to physically move to the outstation centre. a) a, b and c all b) a and b only c) a and c only d) b and c only 10. Speed Clearing is enabled for cheques issued by account holders with the following transaction codes: a) 10 (savings bank) only b) 11 (current account) only c) 13 (cash credit) only d) all transaction codes, other than those relating to government cheques. 11. The extended scope of Speed Clearing became operative w.e.f: a) Jan 01, 2011 b) Feb 01, 2011 c) Mar 01, 2011 d) Apr 01, 2011 12. As per RBI guidelines, the banks should not invest in Non-SLR securities of original maturity of less than one-year, except which of the following: a) certificate of deposit b) non-convertible debentures c) commercial paper d) all the above 13. As per extant guidelines of RBI on capital adequacy and market discipline, all the banks in India would continue to have the parallel run till __: a) Mar 31, 2011 b) Mar 31, 2012 c) Mar 31, 2013 d) Mar 31, 2014 14. As per extant guidelines of RBI on capital adequacy and market discipline, the banks are to ensure that their Basel II minimum capital requirement continues to be higher than the prudential

floor of ____ of the minimum capital requirement computed as per Basel I frame work for credit and market risks. a) 75% b) 80% c) 90% d) 100% 15. As per RBI directions, the preparation of calendar of SLBC/UTLBC meetings by convenor bank and intimation to all the concerned of the cut-off dates for submission of data and dates of meetings to be sent by which of the following dates: a) by 15th January b) by 31st January c) by 15th February d) by 28th February 16. What is the dead line for submission of data to SLBC convenor bank, by difference banks in a State: a) one week from close of the quarter b) 10 days from close of the quarter c) 15 from close of the quarter d) 4 weeks from close of the quarter 17. Convenor banks is required to ensure holding of SLBC meeting within: a) 30 days from close of the half year b) 45 days from close of the half year c) 30 days from close of the quarter d) 45 days from close of the quarter 18. Which time limit fixed by RBI in connection with SLBC meeting is not correctly stated: a) reminder by convenor bank to the banks regarding exact date of meeting 15 days before end of the quarter. b) forwarding the minutes of the SLBC meeting to all stakeholders within 10 days from holding the meeting. c) follow up action points emerged from the meeting within 30 days of forwarding the minutes. d) none of the above 19. Statistics on operation of pre-paid instruments is required to be furnished to RBI by banks: a) on monthly basis, within 7 days from close of the month b) on monthly basis, within 10 days from close of the month c) on quarterly basis, within 7 days from close of the quarter d) on quarterly basis, within 10 days from close of the quarter 20. All eligible current account transactions including trade transactions with which of the following ACU country should be settled in any permitted currency outside the ACU mechanism, as per RBI directions: a) Srilanka b) Iran c) Mynamar d) Nepal Recalled Questions Jan, 2011 21. An HUF cannot become partner in a Partnership Firm? This is as per provisions of: a) RBI directives b) SEBI guidelines c) Provisions of Hindus Law d) none of the above 22. Where a notice reading Customers can also tender cheques at the counter and obtain acknowledgement on the pay-in-slips is to be displayed by bank branches? a) on the counter b) on the entry door of the branch premises c) on the cheque drop-box itself d) at the place where other notices have been displayed 23. Which of the following RBI guidelines on Nomination is correct? a) on pass book, the name of nominee can be written with or without customer request. b) On FDR, the name of nominee can be written with or without customer request. c) on FDR and pass book the words nomination registered are to be indicated. d) all the above 24. Simplified KYC procedure is to be followed for the accounts with balance not exceeding ________ and advisory notice is to be sent to customer when the balance exceeds: a) Rs.50000, Rs.100000 b) Rs.50000, Rs.80000 c) Rs.40000, Rs.50000 d) Rs.50000, Rs.40000 25. Preservation period for records pertaining to identification documents obtained while opening the account is ___________ a) 10 Years after the business relationship is ended b) 10 Years after the date of opening the account c) 10 Years after the date of last transaction in the account d) 10 Years 26. Loans to __________ with investments in plant and machinery upto Rs.10 cr is classifed as

priority sector advance to _________. a) food and agro-based processing units, SME b) food and agro-based processing units, indirect agriculture c) food and agro-based processing units, indirect SME d) Fruit processing units, indirect agrilculture 27. As per recommendations of the _______, the allocation of 60% of the MSE advances to the micro enterprises is to be achived in 2010-11 and 2011-12 is 50%; 55%. a) Prime Ministers Task force b) MSME Ministry Task Force c) RBI Working Group d) Ministry of Industruy Govt. of India Task Force. 28. The limit for collateral free loans is up to Rs.10 alc, is in respect of loans to which of the following category: a) Export loans b) Direct Agriculture Loans c) MSME units d) none of these 29. In case of restructure of Suit Filed accounts under CDR system, the consent of _______ % lenders by _____ consent of ____ is required? a) 75% by value and 75% by number b) 60% by value and 75% by number c) 75% by value and 60% by number d) in suit filed loans, CDR restructuring is not permitted 30. After taking possession of the immovable property, copy of the possession notice is to be published in two local newspaper not later than ____ days. a) 7 days b) 15 days c) 30 days d) no time limit 31. As per RBI directives, the collateral security on indirect agriculture loans in priority sector, is mandatoriy waived up to ________? a) Rs.50000 b) Rs.100000 c) Rs.200000 d) Discretion of the banks 32. Cash Budget System is useful in those cases________: a) where there is wide fluction in the level of current liabilities b) where there is wide fluctuation in the level of fixed assets and long term liabilities. c) where there is wide fluctuation in the level of current assets d) all the above. 33. Which of the following group of exporters is not eligiible for interest subvention of 2% on Rupee Export Credit: a) Jute Manufacturing including Floor covering b) Leather and Leather Manufactures c) Engineering Goods d) Gems and Jewellary 34. Current Ratio is 1.25:1 I case of and 1.33:1 in case of ____ following method of lending: a) MPBF method, cash budget method b) MPBF method, turnover method c) Turnover method, cash budget method d) Turnover method, MPBF method 35. For delay, the penalty is Rs.250 per day and max Rs.25000 in respect of which and max Rs.25000 in respect of which of the following: a) violation of RBI guidelines by a bank b) violation of SEBI regulations by a company c) delay in providig information under RTI Act. d) delay in compensating a customer under Bank Ombudsman Scheme 36. Bank has to reimburse the customers within 12 day in respect of which of the folowing: a) cancellation of demand draft on request of the purchaser. b) wrongful payment of a cheque made from customers account c) amount wrongfully debited on account of failed ATM transactions d) all the above 37. Interest Subsidy of __ on Housing Loans is available upto the loan amount of Rs.10 lac where the cost of house does not exceed ____ : a) 0.5%, Rs.25 lac b) 1.0%, Rs.20 lac c) 1.5%, Rs.15 lac d) 2.0%, Rs.20 lac 38. Additional interest subvention of 2% for prompt payment of crop loan by farmers is available to Public Sector Banks on the condition that the effective rate of interest on such loans to farmers

will be ____: a) 5% b) 7% c) 8.5% d) 9% 39. Present minimum limit for RTGS transactions is ____: a) Rs.50000 b) Rs. 1 lac c) Rs. 2 lac d) Rs.5 lac 40. NEFT batches on week days and on Saturday are: a) 11 and 5, half hourly batches b) 11 and 5, hourly batches c) 11 and 5, 2-hourly batches d) 9 and 6, 2-hourly batches 41. The cause of action has arisen on Jan 22, 2010 under Consumer Protection Act. In this case, the suit can be filed by which of the following dates; a) Feb 21, 2010 b) Jan 21, 2011 c) Jan 21, 2012 d) Jan 22, 2013 42. What is the rate of commission on receipts relating to Govt. Business.. a) Rs.5 per Rs.100 of turnover b) Rs.9 per Rs.100 of turnover c) Rs.45 per transaction d) Rs.60 per transaction 43. The ANBC represents which of the following: a) outstanding loans within a bank as on Mar 31st b) net bank credit + investment in non-SLR bonds in HTM category c) net bank credit + investment in non SLR bonds in HTM category FCNR deposits. d) net bank credit export credit. 44. Subsidy in case of SGSY loans is kept in ___. a) in an interest free fixed deposit account b) in an interest free current account c) in an interest free saving bank account d) in subsidy reserve fund. 45. Maximum project cost for USEP (Women) segment projects in SJSRY is restricted to __ per borrower: a) Rs.2 lac b) Rs.1 lac c) Rs. 50000 d) there is no ceiling 46. CGTMSE amount of guarantee cover for a balance of Rs.80 lakhs in case of an eligible normal borrower in other than NE states, will be: a) Rs.55 lac b) Rs.52.50 lac c) Rs.37.50 lac d) Rs.22.50 lac 47. A bill of lading mentions that certain packets of goods are not properly packed and packing is defective. This is: a) claused bill of lading b) clean bill of lading c) defective bill of lading d) stale bill of lading 48. Which f the following are presently kept at 6% by RBI: a) CRR and Repo Rate b) Repo Rate and Bank Rate c) Bank Rate and CRR d) CRR and reverse repo rate 49. Risk weightage on Consumer Credit/Credit Cards, for capital adequacy ratio purpose is: a) 75% b) 100% c) 125% d) 150 % 50. Minimum period of issue which of the following instruments is not 7 days: a) non-convertible debenture b) certificate of deposit c) commercial paper d) bank FDR 51. Bank-A purchased an NPA account from Bank-B. Now Bank-A wants to sell this account to Bank-C. Bank-B can do so after: a) 15 months b) 15 months from date of account becoming NPA with Bank-B c) 15 months after 1st 90 days, when the Bank B classifies the account in standard category immediately on its purchase. d) 15 months from date of purchase of account from Bank-A. 52. Which of the following loan is exempted from application of Base Rate: a) loan given as export credit b) loan given by a consortium of banks c) loan given for direct agriculture purpose d) none of the above 53. As per provisions of which of the folllowing, the nominated bank gets a period of ___ days to verify the documents drawn drawn under a documentary credit: a) RBI guidelines, 7 banking days b) FEDAI rules, 5 banking days c) UCPDC, 5 calendar days d) none of these is correct

54. Which of the following is not a Financial Sector reform: a) introduction of NPA guidelines b) introduction of obtaining photograph of customers c) application of asset liability management rules d) none of the above 55. Which of the following is not correct regarding Safe Deposit Lockers guidelines of RBI: a) branches are to maintain a waitlist for allotment of lockers that should be transparent b) all locker allotment applications should be ackowledged c) all locker allotment applications should be given a wait list number d) bank can provide detailed terms and conditions of locker allotment to the customer but should not provide copy of the agreement regarding operation of locker. 56. Which of the following banking facility cannot be offered to visually challlenged persons (blind persons): a) cheque book facility, net banking and ATM facility b) Lockers c) retail loans, credit card d) none of these 57. Which of the following activity cannot be undertaken by Business Correspondents a) disbursement of small value loans b) collection of small value deposit c) recovery of principal and interest d) collection of transaction fee from customer 58. Interest subsidy on Housing Loans allowed to: a) weaker section borrowers only b) middle and low income groups c) only low income group borrowers d) all housing loan borrowers 59. TDS is deductible on which of the following account: a) all type of term deposit accounts including recurring deposit b) all saving bank accounts c) all NRO accounts including saving bank, where of the above d) none of the above. 60. Details of unimplemented awards of Ombudsman scheme pending for more than 3 months, are to be placed by the banks to which of the following: a) Dy. Governor RBI b) CMD/ED/CEO of the bank c) Customer Service Committee of Board d) Board of Directors of the bank 61. W.e.f. Jan 24, 2011, the official timing for opration of Centralised Funds Management System for Monday to Friday is: a) 11 am to 4.30 pm b) 11 am to 5.00 pm c) 10 am to 4.30 pm d) 10 am to 5.00 pm 62. W.e.f. Jan 24, 2011, the official timing for operation of Centralised Funds Management System for Saturday is: a) 11 am to 2.30 pm b) 11 am to 3.00 pm c) 10 am to 2.30 pm d) 10 am to 3.00 pm 63. W.e.f. 1.4.2011, the processing/service charges for local clearing (by clearing house from member banks) for clearing at MICR-CPCs for presenting bank and drawee bank are as under: a) Rs.1 and Rs.1 b) Rs.1 and Rs.1.50 c) Rs.1.50 and Rs.1 d) Rs.1.50 and Rs.1.50 64. W.e.f. 1.4.2011, the processing/service charges for local clearing (by clearing house from member banks) for clearing at cheque truncation, for presenting bank and drawee bank are as under: a) 100 p and 75 p b) 75 p and 75 p c) 50 p and 100 p d) 75 p and 50 p 65. W.e.f. 1.4.2011, the outstation cheque collection charges have been revised by RBI. Which of the following is not corect in the revision: a) up to Rs.5000 Rs.25 b) Rs.5001 to Rs.10000-Rs.50 c) Rs.10001 to Rs.1 lac Rs.100 d) none of the above 66. The charges for collection of outstation cheques as revised by RBI wef 1.04.2011 for cheques of above Rs.1 lac as under: a) Rs.100 b) Rs.150 c) 200 d) at discretion of bank 67. Service charges for cheque collection under speed clearing from customer by banks, wef

1.4.2011 for cheques up to Rs.1 lac are as under: a) Rs.100 b) Rs.150 c) Rs.200 d) nil 68. Service charges for cheque collection under speed clearing from customer by banks, wef 1.4.2011 for cheques above Rs.1 lac are as under: a) Rs.150 b) Rs.200 c) Rs.250 d) at discretion of the bank 69. RBI Sub-Committee of its Central Board of Directors to study Issues and concerns in the micro finance institutions (MFI) Sector, which submitted its report in Jan 2011, was headed by Sh. __: a) Dr. Chakravarty b) YH Malegam c) Deepak Mohanty d) VK Sharma 70. An accounting software developed for running in computer may be written in which of the programming languages: a) COBOL b) FOXPRO c) UNIX d) all of these ANSWERS GENERAL AWARENESS 2011 SEVEN 1.d 2.d 3.d 4.b 5.d 6.d 7.d 8.b 9.a 10.d 11.b 12.d 13.c 14.b 15.a 16.c 17.d 18.d 19.a 20.b 21.d 22.c 23.c 24.d 25.a 26.b 27.a 28.d 29.c 30.d 31.d 32.c 33.d 34.d 35.c 36.c 37.b 38.a 39.c 40.b 41.c 42.c 43.b 44.d 45.a 46.b 47.a 48.c 49.c 50.a 51.d 52.d 53.d 54.b 55.d 56.d 57.d 58.b 59.c 60.c 61.d 62.d 63.b 64.c 65.d 66.d 67.d 68.d 69.b 70. d

*****************************

GENERAL AWARNESS 2011 - EIGHT Latest Policy Based Questions 1. SLR has been reduced by RBI wef Dec 18,2010 from ______ to _______ of net demand and time liabilities : a) 40% to 25% b) 40% to 24% c) 25% to 24% d) 25% to 20% 2. The term headline inflation, represents which of the following a) wholesale price index b) consumer price index for industrial workers c) consumer price index for agriculture workers d) all the above taken together. 3. As per Prevention of Money laundering (Maintenance of Records of the Nature and Value of Transactions, the Procedure and Manner of Maintaining and Time for Furnishing Information and Verification and Maintenance of Records of the Identity of the Clients of the Banking Companies, Financial Institutions and Intermediaries) Second Amendment Rules, 2010, no banking company, financial institution and Intermediary, as the case may be, shall allow : a) the opening of or keep any anonymous account b) account in fictitious names c) account on behalf of other persons whose identity has not been disclosed or cannot be

verified d) all the above 4. The borrowers under Swarn Jayanti Gram Swairozgar Yojna have the benefit of group life insurance provided by LIC of India. In term of the scheme, in case of natural death of the borrower, the nominee will get _________ : a) Rs.5000 b) Rs.6000 c) Rs.10000 d) Rs.12000 5. The borrowers under Swarn Jayanti Gram Swairozgar Yojna have the benefit of group life insurance provided by LIC of India. In term of the scheme, in case of death of the borrower due to accident, the nominee will get. a) Rs.5000 b) Rs.6000 c) Rs.10000 d) Rs.12000 6. A customer has received inward remittance for credit to NRE account through credit push systems like RTGA, NEFT, NECS and ECS. He request for issue of foreign inward remittance certificate (FIRC): a) the FIRC cannot be issued as per RBI guidelines in such cases. b) the FIRC cannot be issued as per FEDAI Rules in such cases. c) the FIRC can be issued if the amount is below USD 10000. d) the FIRC can be issued if the amount is below USD 100000. 7. A bank branch has received an inward remittance in foreign currency for credit to customers NRE account by converting the foreign exchange in to rupees. The customer request for issue of foreign inward remittance certificate (FIRC) a) the FIRC can be issued as per RBI guidelines in such cases. b) the FIRC can be issued as per FEDAI Rules in such cases . c) the FIRC can be issued if the amount is below USD 10000 d) the FIRC can be issued if the amount is below USD 100000 8. A salaried employee working with a multi national company of repute, wants to open a saving bank account. To comply with KYC requirement he needs to submit (1) letter / certificate issued by the company that he is working with them (2) copy of pass port or driving licence (3) copy of PAN card or voter identity card: a) only 1 b) only 1 and 2 c) only 1 and 3 d) 1 and any of 2 or 3 9. Firm A is owned Mr.A and B and Firm B is also owned by Mr.A and B. Both the firms are engaged in manufacturing and have investment in plant and machinery amounting to Rs.4 cr and Rs.3 cr respectively. As per provisions of MSME Dev Act 2006: a) only one of them can be classified as small enterprise b) none of these can be classified as small enterprise c) both of these can be classified as small enterprise independently d) only firm B having lower investment can be classified as a small enterprise 10. X has a current account with the bank and he agrees to receive a cheque of Rs.10 lac from B and on the strength of this amount of Rs.10lac, he issues 5 cheques of Rs.1.90 each in the name of 5 different persons and retains the balance amount as his commission. Such type of transaction is know as: a) money agency transaction b) money pipe transaction c) money mule transaction d) money flow transaction 11. The money mule transactions can be restricted by banks if the banks follow (1) KYC guidelines of RBI (2) Anti money laundering standards (3) combating of financing of terrorism standards (4) obligations under PML Act 2002: a) 1 to 4 all b) only 1 and 4 c) only 1 and 3 d) only 1 and 2 12. Which of the following statement regarding non-convertible debentures is not correct: a) Financial Institutions (FIs) to invest in NCDs of maturity up to one year. b) NBFCs including Primary Dealers that do not maintain a working capital limit to issue NCDs of maturity up to one year; c) FIIs to invest in NCDs of maturity up to one year d) none of the above 13. All NBFCs are required to submit a certificate to RBI from Statutory Auditor with respect to the position of the company as on March 31st every year within _____ from the date of finalization of the balance sheet and in any case not later than ____ a) one month, Dec 30th of that year b) one month, June 30th of that year c) 2 months, Dec 30th of that year d) 2 months, June 30th of that year

14. As per RBI guidelines, NBFCs are required to finalise their balance sheet within a period ____ from the date to within it pertains. a) one month b) 2 months c) 3 months d) 6 months Questions on MSEs 15. An enterprise engaged in manufacturing, production or preservation of goods is a micro enterprise, where its investment in ____ is up to ____ : a) plant and Machinery, up to Rs.10 lac b) Plant and Machinery, up to Rs.25 lac c) Equipment, up to Rs.10 lac d) Equipment, up to Rs.25 lac 16. An enterprise engaged in providing or rendering of services is a micro enterprise, where its investment in ____ is up to ___ : a) plant and Machinery, up to Rs.10 lac b) Plant and Machinery, up to Rs.25 lac c) Equipment, up to Rs.10 lac d) Equipment, up to Rs.25 lac 17. An enterprise engaged in manufacturing, production or preservation of goods is a small enterprise, where its investment in _______ is up to _______: a) plant and Machinery, above Rs.10 lac up to Rs.200 lac b) Plant and Machinery, above Rs.25 lac up to Rs.500 lac c) Equipment, above Rs.10 lac up to Rs.200 lac d) Equipment, above Rs.25 lac up to Rs.500 lac. 18. An enterprise engaged in providing or rendering of services is a small enterprise, where its investment in ___is up to ____: a) plant and Machinery, above Rs.10 lac up to Rs.200 lac. b) plant and Machinery, above Rs.25 lac up to 500 lac c) Equipment, above Rs.10 lac up to Rs.200 lac d) Equipment , above Rs.25 lac up to Rs.500 lac 19. An enterprise engaged in manufacturing, production or preservation of goods is a medium enterprise, where its investment in ____ is up to ____ : a) Plant and Machinery, above Rs.10 lac up to Rs.200 lac b) plant and Machinery, above Rs.25 lac up to Rs.500 lac c) Plant and Machinery, above Rs.500 lac up to Rs.1000 lac d) Equipment, above Rs.500 lac up to Rs.1000 lac 20. An enterprise engaged in providing or rendering of services is a medium enterprise, where its investment in ______ is up to ____ a) Equipment, above Rs.10 lac up to Rs.200 lac b) Equipment, above Rs.25 lac up to Rs.500 lac c) Equipment, above Rs.200 lac up to Rs.500 lac d) Equipment, above Rs.500 lac up to Rs.1000 lac 21. The Investment criteria for Micro, Small and Medium enterprises is as per provisions of : a) RBI guidelines b) Guidelines a Ministry of Industries, Govt. of India c) MSME Development Act 2006 d) Companies Act 1956 22. Which of the following activity is also part of the micro and small enterprises (1) small road and water transport operators (2) small business (3) Retail trade (d) self employed persons a) 1 to 4 all b) 1,2 and 4 only c) 1,3 and 4 only d) 2,3 and 4 only 23. All Advances granted to units in KVI sector, irrespective of size of operations, location and amount of original investment in plant and machinery are to be included in : a) 40% sub target of advances to micro enterprises within advances to MSE b) 60% sub target of advances to micro enterprises within advances to MSE c) Overall 10% target for advances to MSE within priority sector d) such advances are non priority sector advances 24. Loans to which of the following are not part of advances to priority sector, under MSME sector : a) loans to micro enterprises b) loans to small enterprises c) loans to medium enterprises d) loans to units in KVI sector of their investment in plant and machinery is above Rs.5 cr and they are not located in rural or semi urban areas 25. Which of the following types of loans are part of indirect to MSEs in priority sector: a) Persons involved in assisting the decentralized sector in the supply of inputs and marketing of

outputs of artisans, village and cottage industries. b) Advances to cooperative of producers in the decentralized sector viz. artisans, village and cottage industries. c) Loans granted by banks to NBFCs for on lending to Micro and Small Enterprises (MSE) sector (manufacturing as well as service) d) all the above 26. In terms of recommendations of prime Ministers Task Force on MSMEs, the banks are required to ensure to_______ year on year growth of credit to _____: a) 20% micro, and small enterprises b) 20% micro, small and medium enterprises c) 10% micro and small enterprises d) 10%, micro, small and medium enterprises 27. In terms of recommendations of Prime Ministers Task Force on MSMEs, the banks are required to ensure a ____ increases in no of accounts for credit to ___ : a) 10% micro enterprises b) 10% micro and small enterprises c) 10% micro small and medium enterprises d) 10% all industry related enterprises 28. In terms of recommendations of Prime Ministers Task Force on MSMEs, the banks are to have ____% of their advances to MSE sector to micro enterprises, as on Mar 31,2011: a) 40% b) 50% c)55% d)60% 29. In terms of recommendations of Prime Ministers Task Force on MSMEs, the banks are to have 55% of their advances to MSE sector to micro enterprises, as on : a) Mar 31, 2010 b) Mar 31, 2011 c) Mar 31, 2012 d) Mar 31,2013 30. In terms of recommendations of Prime Ministers Task Force on MSMEs, the banks are to have 60% of their advances to MSE, sector to micro enterprises, which is to be done in stages. Which of the following does not match in this context: a) 45% should be achieved by 31.03.10 b) 50% should be achieved by 31.03.11 c) 55% should be achieved by 31.03.12 d) 60% should be achieved by 31.03.13 31. What %age of advances to MSE sector should be given by banks to micro enterprises with investment in plant and machinery up to Rs.5 lac or investment in equipment up to Rs.2 lac. a) 20% b)25% c) 40% d) 60% 32. 20% of advances to MSE sector should be given by banks to micro enterprises with investment in : a) plant and machinery up to Rs.5 lac or investment in equipment up to Rs.2 lac. b) plant and machinery above Rs.5 lac or investment in above Rs.2 lac. c) plant and machinery above Rs.5 lac up to Rs.10 lac or investment in equipment above Rs.2 lac up to Rs. 5 lac. d) plant and machinery above Rs.5 lac up to Rs.25 lac or investment in equipment above Rs.2 lac up to Rs.10 lac. 33. An MSME sector loan application for an amount up to Rs.5 lac is to be disposed of within: a) 2 weeks b) 4 weeks c) 6 weeks d) 8-9 weeks 34. An MSME sector loan application for an amount up to Rs.25000 is to be disposed of within : a) 2 weeks b) 4 weeks c) 6 weeks d) 8-9 weeks 35. An MSME sector loan application is to be disposed of within 8-9 weeks where the amount of loan application is : a) up to Rs.25000 b) up to Rs.5 lac c) above Rs.5 lac d) above Rs.25 lac 36. Banks have been mandated by RBI not to accept collateral security in case of MSE loans for an amount up to : a) Rs.2 lac b) Rs.5 lac c) Rs.10 lac d) Rs.25 lac 37. In case of MSE loan accounts with good track record and financial position, the banks can consider loan proposals for an amount up to Rs. ____, without a collateral security : a) Rs. 2 Lac b) Rs. 5 Lac c) Rs. 10 Lac d) Rs. 25 Lac 38. A Composite loan limit ___ can be sanctioned by banks to enable the MSE entrepreneurs to

avail of their working capital and term loan requirement through Single Window a) up to Rs.25 lac b) up to Rs.50 lac c) up to Rs. 100 lac d) up to Rs. 200 lac 39. Banks can categorise their MSME general banking branches as specialized MSE branch provided _____ of their advances in the branch are to MSME sector : a) 40% or more b) 50% or more c) 60% or more d) 80% or more 40. A unit is considered as sick MSE unit (1) when any of the borrowal account of the unit remains substandard for more than 6 months or (2) there is erosion in the net worth due to accumulated cash losses to the extent of 50% of its net worth during the previous accounting year and (3) the unit has been in commercial production for at least two years. a) 1 and 2 only b) 2 and 3 only c) 1 and 3 only d) 1 to 3 all 41. As per RBI guidelines, the rehabilitation package relating to an MSE, unit, should by fully implemented within _____ months from the date the unit is declared as potentially viable / viable. a) 2 months b) 3 months c) 4 months d) 6 months 42. Under the Credit Linked Capital Subsidy Scheme (CLSS) for Technology Upgradation of Micro and Small Enterprises, the ceiling on loan amount is restricted to : a) Rs. 25 lac b) Rs. 50 lac c) Rs.100 lac d) Rs. 500 lac 43. Under the Credit Linked Capital Subsidy Scheme (CLSS) for Technology Upgradation of Micro and Small Enterprises, the rate of subsidy is : a) 12% of the purchase price of plant and machinery b) 15% of the purchase price of plant and machinery c) 12% of the loan amount d) 15% of the loan amount 44. Which of the following committee is not associated with lending to MSE sector : a) Kapoor Committee b) Deepak Mohanty Committee c) Ganguly Committee d) Nayak Committee 45. Which of the following is to be excluded while calculating the original price of plant and machinery or equipment in case of MSE units: (i) equipment such as tools, jigs, dyes, moulds and spare parts for maintenance and the cost of consumables stores; (ii) installation of plant and machinery; (iii) research and development equipment and pollution controlled equipment (iv) power generation set and extra transformer installed by the enterprise as per regulations of the State Electricity Board; a) I to iv all b) I, ii and iv only c) ii, iii and iv only d) I, iii and iv only 46. Registration of which of the following group of enterprises, is mandatory under provisions of MSME guidelines: a) micro, small and medium enterprises b) micro and small enterprises engaged in manufacturing c) medium enterprises engaged in manufacturing, production d) medium enterprises engaged in manufacturing, production and providing service. 47. An ancillary unit in SSI is one that supplies ___% of its production to one or more industrial undertaking : a) at least 50% b) 50% or more c) max 50% d) 49% or more CGFT-MSE Guarantee 48. Which among the following borrower is not eligible for CGT MSE guarantee : a) new and existing micro & small enterprise b) IT or software industry c) select activities in case of agri clinics and agri business centres d) Rehabilitation of SSI units covered under CGF e) medium enterprises availing loans up to Rs.25 lac 49. The type of loan and the amount of loan, which is eligible for CGTMSE guarantee cover, is one of the following : a) only fund based working capital limits, up to Rs.100 lac b) only fund based working capital limits and term loans, up to Rs.100 lac c) only fund based working capital limits, term loans and bank guarantee, up to Rs.100 lac d) fund based working capital limits, term loans and non fund based limits all, up to Rs.100 lac. 50. Guarantee cover under CGTMSE scheme is available: (1) for a period as long as the bank and borrower decide to avail (2) for the term loan repayment period (3) for max 5 years where

working capital limit alone is sanctioned (4) maximum for 7 years in all cases a) only 1 is correct b) only 2 and 3 are correct c) only 4 is correct d) 2 and 4 are correct 51. The rate of guarantee fee (up front) under CGTMSE scheme in which of the following cases is not stated correctly : a) general category loans 1.5% of sanctioned limit b) loans to SC.ST 1.0% of sanctioned limit c) loans up to Rs.5 lac to micro enterprises 1.0% of sanctioned limit d) laons to units in NE States 0.75% of sanctioned limit. 52. The rate of service fee (renewal) under CGTMSE scheme in which of the following cases is not stated correctly : a) general category loans 0.75% of sanctioned limit. b) loans to SC.ST 0.75% of sanctioned limit c) loans up to Rs.5 lac to micro enterprises 0.5% of sanctioned limit d) loans to units in NE States and women enterprises 0.50% of sanctioned limit 53. The time limit for payment of guarantee fee and service fee under CGTMSE scheme is as under : a) guarantee fee within one month from date of first disbursement and service fee within 2 months from 31st Mar b) guarantee fee within 2 months from date of first disbursement and service fee within 2 months from 31st Mar c) guarantee fee within one month from date of first disbursement and service fee within 1 month from 31st Mar d) guarantee fee within 2 months from date of first disbursement and service fee within 1 month from 31st Mar. 54. The guarantee cover under CGTMSE scheme for women enterprises and loans sanctioned in NE states is ____ % of the balance on date of the account identified as NPA as per RBI guidelines (which statement is wrong) a) up to Rs.5 lac, 85% max Rs.4.25 lac b) above Rs.5 lac up to Rs.50 lac, 80% max Rs.40 lac c) above Rs.50 lac up to Rs.100 lac, 50% max Rs.25 lac d) total maximum cover amount Rs.65 lac. 55. The guarantee cover under CGTMSE scheme for micro enterprises other than the women enterprises and units is NE states is ___% of the balance on date of the account identified as NPA as per RBI guidelines (locate the false statement) a) up to Rs.5 1ac, 85% max Rs.4.25 lac b) up to Rs.50 lac, 75% max Rs.37.50 lac c) above Rs.50 lac up to Rs.100 lac, 50% max Rs.25 lac d) total maximum cover Rs.65 lac 56. The outstanding balance in a loan to a women enterprise and guaranteed under CTGMSE scheme on date of NPA is Rs.60 lac. what is the amount of claim that bank will be eligible for : a) Rs.45.00 lac b) Rs. 42.50 lac c) Rs.40.00 lac d) Rs.37.50 lac 57. The outstanding balance in a loan to a micro enterprises and guaranteed under CTGMSE scheme on date of NPA is Rs. 60 lac. What is the amount of claim that bank will be eligible for : a) Rs.45.00 lac b) Rs.42.50 lac c) Rs.40.00 lac d) Rs.37.50 lac 58. In a loan guaranteed under CGTMSE schemes, the bank can claim the amount if the following conditions are satisfied (1) guarantee cover is in force (2) account is classified as NPA (3) loan has been recalled and recovery proceedings have been initiated (4) loan has been written off, a) 1 to 4 all b) 1 to 3 only c) 1,2 and 4 only d) 2,3 and 4 only 59. Under CGTMSE guarantee scheme the claim can be lodged (1) within 6 months from date of account becoming NPA (2) within 1 year from date of account becoming NPA (3) within 6 months from date of completion of 18 months lock in period (4) within 12 months from date of completion of 18 months lock in period

a) 1 and 4 are correct b) 1 and 3 are correct c) 2 and 3 are correct d) 2 and 4 are correct 60. Under the CGTMSE guarantee scheme, the payment of eligible claims is made as under by the guarantee fund: a) 100% amount immediately within 30 days b) 75% amount within 30 days and balance 25% on conclusion of recovery proceedings c) 75% amount within 45 days and balance 25% on conclusion of recovery proceedings d) 50% amount within 30 days and balance 50% on conclusion of recovery proceedings. ********** ANSWERSGENERAL AWARENESS EIGHT 1.c 2.a 3.d 4.b 5.d 6.a 7.a 8.d 9.c 10.c 11.a 12.d 13.a 14.c 15.b 16.c 17.b 18.c 19.c 20.c 21.c 22.a 23.b 24.c 25.d 26.a 27.a 28.b 29.b 30.a 31.c 32.d 33.b 34.a 35.c 36.c 37.d 38.c 39.c 40.d 41.d 42.c 43.b 44.b 45.a 46.c 47.a 48.e 49.d 50.b 51.b 52.d 53.a 54.b 55.d 56.a 57.b 58.b 59. 60.b **

BANKING GENERAL AWARENESS - SIX 1. Very often we see in the advertisements published by Financing Institutes/Agencies stating that their products are given high or average Ratings. These Rating Agencies classify bonds/investments in how many categories ? 1. Low Risk 2. Average Risk 3. High Risk (A) Only 1 (B) Only 2 (C) Only 3 (D) All 1, 2 and 3 (E) None of these 2. As per the reports published in various newspapers/journals etc. the economic map of the world is being redesigned. What is/are the new emerging trends of this new economic map of the world ? 1. With India and Asia at the forefront the centre of economic gravity has shifted to East. 2. Emerging markets, with India among the leaders, are growing faster than old established markets. 3. Central banks of the countries have recognized that case-by-case liquidity solutions are not the answers and a market wide and globally synchronized approach is needed to solve the present crisis. (A) Only 1 (B) Only 2 (C) Only 3 (D) All 1, 2 and 3

(E) None of these 3. As per the decision taken by the Govt. of India, the exporters of which of the following products to USA and European countries will get an incentive of 2% on their exports ? 1. Leather goods 2. Garments 3. Software (A) Only 1 (B) Only 2 (C) Only 3 (D) Both 1 and 2 only (E) All 1, 2 and 3 4. Which of the following is/are the key features of Indian Economy which were highlighted during the presentation of the Interim Budget 2009-10 ? 1. Despite global financial crisis the GDP growth rate in current financial year has been 71 per cent. 2. Indian economy was adjudged as second fastest growing economy in the world 3. A provision of Rs. 100 crores is made in the annual plan 2009-10 for setting up Unique Identification Authority of India. (A) Only 1 (B) Only 2 (C) Only 3 (D) All 1, 2 and 3 (E) None of these 5. Many a times we read a term in financial news papers SEPA. What is the full form of the term ? (A) Single Exchange Processing Agency (B) Single Euro Payments Area (C) Single Electronic Processing Agency (D) Super Electronic Purchase Agency (E) None of these 6. As per the newly launched Indira Gandhi National Widow Pension Scheme a monthly pension will be given to the widows. What will be the amount of the pension ? (A) Rs. 200 (B) Rs. 400 (C) Rs. 600 (D) Rs. 800 (E) None of these 7. As per the decision taken by the Govt. of India all the Public Sector Banks (PSBs) will be recapitalised over the next two years so that they can maintain a Capital Adequacy Ratio (CAR) of . (A) 9% (B) 12% (C) 11% (D) 22% (E) None of these

8. As per the Industrial Development Report 2009 Indias rank in Industrial Development was 54 on the Development Index. Which of the following organizations/agencies compiles Industrial Development Index of the world ? (A) International Labour Organisation (B) United Nations Industrial Development Organisation (C) World Bank (D) World Trade Organisation (E) The Economic and Social Council of UNO 9. As per the news published in major news papers/magazines the International Finance Corporation has offered a loan for providing clean drinking water to 30 lakhs people in rural India. What is the amount of the loan ? (A) 5 million US $ (B) 10 million US $ (C) 15 million US $ (D) 20 million US $ (E) None of these 10. The merger of which of the following two Indian companies took place in recent past which is termed as Largest ever merger in Indias Corporate History ? (A) Nuclear Power Corporation and National Thermal Power Corporation (B) State Bank of India and its seven associate banks (C) Hind Aluminium Company and Century Mills (D) Tata Motors and Ashok Leyland (E) Reliance Industries and Reliance Petroleum Ltd. 11. We very frequently read about the activities of the Foreign Exchange Market in newspapers/magazines. Which of the following is/are the major functions of the same ? 1. Transfer of purchasing power from domestic to foreign market. 2. Providing credit for financing foreign trade. 3. Power to purchase gold from foreign countries as most of the nations still work on Gold Standards. (A) Only 1 (B) Only 2 (C) Only 3 (D) Both 1 and 2 only (E) Both 2 and 3 only 12. The Govt. of India recently introduced Money Laundering Bill 2009. Under the bill which of the following agencies is empowered to search the premises after the offence is committed and a case is filed ? (A) Income Tax Department (B) Vigilance Department of the Reserve Bank of India (C) Enforcement Directorate (D) Central Bureau of Investigation (E) None of these 13. Which of the following is/are the components of the Fiscal Deficit ? 1. Budgetary Deficit 2. Market Borrowings 3. Expenditure made from Pradhan Mantri Rahat Kosh (A) Only 1

(B) Only 2 (C) Only 3 (D) All 1, 2 and 3 (E) None of these

14. Which of the following is not a Welfare scheme launched by the Govt. of India ? (A) Village Grain Bank Scheme (B) Sampoorna Gramin Rozgar Yojana (C) Annapurna Scheme (D) Midday Meal Scheme (E) Bharat Nirman Yojana 15. Which of the following are the instruments of Credit Control in the hands of the RBI ? 1. Lowering or raising the discount and interest rates. 2. Raising the minimum support price of the major agro products. 3. Lowering or raising the minimum cash reserves maintained by the commercial banks. (A) Only 1 (B) Only 2 (C) Only 3 (D) Both 1 and 3 only (E) Both 2 and 3 only

16. Which of the following programmes was launched to further improve the facilities of irrigation in rural India ? (A) National Social Assistance programme (B) Construction of Kiosks for poor (C) Sampoorna Grameen Rozgar Yojana (D) Annapurna Scheme (E) National Watershed Development Programme 17. Many times we read in newspapers that a company is planning to bring a public issue. What does it mean? A) Shares of the company will be issued only through public sector organizations like banks/Central financial institutions etc. B) Shares of the company will be issued to general public only through primary market. C) This means some stakeholders/ promoters are willing to leave the company. Hence they wish to sell their stock to the general public. a) Only A b) Only B c) Only C d) All A,B&C e) None of these 18. Whenever some people wish to enter into the business world, it is a must for them to approach a bank. What services do banks provide them in this regard? A) Banks act as payment agents by operating current accounts, paying cheques and receiving payments for them. B) Maintaining account books for them for their day-to- day activities so that they are not required to appoint account/finance personnel on a regular basis. C) Lending money by way of overdraft, installment loan, credit or advance for business activities a) Only A b) Only B c) Only C d) Only A&C e) All of these 19. As we all have noticed, banks these days are giving more emphasis on Branchless Banking. What does this really mean?

A) Banks will not have many branches as used to be in the good old days. Instead, the number of branches will be restricted and will conduct only a specified core business. B) Banks will launch/operate many delivery channels like ATMs, Mobile Banking/ Internet Banking etc sot that people are not required to visit a branch for their usual banking needs. C) This means banks will issue only debit or credit cards for all types of day-to-day financial transactions. Cheques/cash payments will not be allowed. a) Only A b) Only B c) Only A&B d) Only B&C e) All A, B&C 20. Which of the following is NOT a function of a bank? a) Providing project finance b) Selling Mutual Funds c) Deciding policy rates like CRR, Repo Rate / SLR etc d) Settlement of payments of behalf of the customers e) All of these are functions of a bank 21. How many banks were nationalized in 1969? (A) 16 (B) 14 (C) 15 (D) 20 (E) None of these. 22. The Reserve Bank of India was established in : (A) 1820 (B) 1920 (C) 1935 (D) 1940 (E) None of these. 23. The first Indian Bank was : (A) Traders Bank (B) Imperial Bank (C) Presidency Bank of Calcutta (D) Indian Bank (E) None of these. 24. The rupee coin was first minted in India in : (A) 1542 (B) 1601 (C) 1809 (D) 1677 (E) None of these. 25. The Export-Import (EXIM) Bank was set up in : (A) 1980 (B) 1982 (C) 1981 (D) 1989 (E) None of these. 26. Planning Commission is : (A) Advisory body (B) Executive body (C) Government body (D) Autonomous body

(E) None of these. 27. The Community Development Programme was launched in : (A) 1950 (B) 1952 (C) 1956 (D) 1960 (E) None of these. 28. The highest body which approves the Five-Year Plan is the : (a) Finance Ministry (B) Lok Sabha (C) Rajya Sabha (D) National Development Council (E) None of these. 29. Which of the following commodities earn maximum foreign exchange for India? (A) Jute (B) Iron and Steel (C) Tea (D) Sugar (E) None of these. 30. The one rupee note bears the signature of : (A) Secretary, Ministry of Finance (B) Governor, Reserve Bank of India (C) Finance Minister (D) Prime Minister (E) None of these. Computer awareness (31-50) 31. The component used in second generation computers is 1. vacuum tube 2. transistors 3. integrated circuits 4. microprocessors 5. None of these 32. Transistors were placed on 1. integrated circuits 2. microprocessors 3. silicon chips 4. vacuum tubes 5. None of these 33. The third generation computers were interfaced with. 1. machine language 2. assembly language 3. operating system 4. high-level language 5. None of these 34. Which one of the following component used in fourth generation computers? 1. Microprocessors 2. Integrated circuits 3. Transistors 4. Vacuum tubes 5. None of these 35. The fifth generation computing devices are based on . 1. internet 2. artificial intelligence 3. high-level languages 4. VLSI 5. None of these 36. The third generation computers was developed during the period . 1. 1940- 1956 2.1961-1963 3.1964-1971 4.1971-present 5. None of these

37. Thousands of integrated circuits were built on to a single silicon chip called . 1. transistors 2. integrated circuits 3. artificial intelligence 4. microprocessors 5. None of these 38. UNIVAC stands for 1 Universal Automatic Computing 2. Universal Auto Computer 3. Universal Automatic Computer 4. Union Automatic Computer 5. None of these 39. In computer, data is converted into information through.. 1. instructions 2. processing 3. data 4. facts 5. None of these 40. Integrated circuits were used in..generation of computers. 1 first 2. second 3. third 4. fourth 5. None of these 41. The first generation computers used..fro circuitry a. vacuum tube b. transistors c. integrated circuits d. microprocessors e. None of these 42. Which one of the following is defined as an un-processed collection of raw facts? a. Information b. Processing c. Instructions d. Data e. None of these 43. Which is a collection of facts from which conclusions may drawn? a. Data b. Information c. Processing d. Instructions e. None of these 44. Information is .. a. processed facts b. in active c. technology based d. set of instructions e. None of these 45. Data are. a. processed facts b. active c. business based d. stored facts e. None of these 46. A set of instructions or procedures for solving problems is defined as a. data b. information c. algorithm d. processing e. None of these 47. Which one of the following is designed to systematically solve a problem? a. Information b. A computer program c. Algorithm d. Data e. None of these 48. A computer system has major components a. 4 b. 3 c. 2 d. 5 e. None of these 49. Which one of the following refers to all the physical components associated with the computer system? a. Hardware b. Software c. Information d. Data e. None of these

50. Software is a set of.. a. data b. raw Facts c. instructions d. information e. None of these *****************************

ANSWERS BANKING GENERAL AWARENESS SIX 1.D 2.A 3.D 4.D 5.B 6.A 7.B 8.B 9.C 10.E 11.D 12.E 13.A 14.E 15.C 16.E 17.B 18.D 19.E 20.C 21.B 22.C 23.C 24.A 25.B 26.A 27.B 28.D 29.C 30.C 31.C 32.C 33.B 34.A 35.B 36.C 37.D 38.C 39.B 40.C 41.A 42.D 43.B 44.A 45.D 46.C 47.B 48.C 49.A 50.C ****************************************************

BANKING GENERAL AWARENESS - SEVEN Questions based on latest RBI Policy 1. In the context of monitoring for verification of end use of funds for loans sanctioned by banks, which of the following is not part of RBI guidelines on the issue : a) meaningful scrutiny of the periodical progress reports and operating / financial statements of the borrowers; b) regular visits to the assisted units and inspection of securities charged / hypothecated to the banks; c) periodical scrutiny of the books of accounts of the borrowers; d) None of the above 2. In the context of monitoring for verification of end use of funds for loans sanctioned by banks, which of the following is part of RBI guidelines on the issue: a) introduction of stock audits depending upon the extent of exposure; b) obtention of certificates from the borrowers that the funds have been utilized for the purposes approved. c) examination of all aspects of diversion of funds during internal audit / inspection of the branches and at the time of periodical reviews. d) all the above 3. A non banking financial company with carries on the business of acquisition of shares and securities and satisfies certain conditions, is called : a) Investment company b) financial company c) non bank finance company d) core investment company 4. A core investment company must hold not less than ____ of its net assets in the form of investment in equity shares, preference shares, bonds, debentures, debt or loans in group companies. a) 100% b) 90% c) 74% d) 60% 5. The investment made by core investment company in the equity shares (including instruments compulsorily convertible into equity shares within a period not exceeding 10 years from the date of issue) in group companies must constitute not less than ___ of its net assets : a) 100% b) 90% c) 74% d) 60% 6. A core investment company cannot invest in which of the following: a) bank deposits, b) money market instruments, including money market mutual funds c) govt. securities d) none of the above 7. A core investment company can undertake which of the following activity : a) investment in bonds or debentures issued by group companies; b) granting of loans to group companies c) issuing guarantees on behalf of group companies. d) all the above 8. A systemically important core investment company means a core investment company fulfilling the following conditions (a) Having total assets of not less than Rs.100 crore, either individually or in aggregate along with other Core Investment Companies in the Group; (b) Raises or holds public funds; (c) makes investment in bank deposit or govt. securities.

a) a, b and c all b) a and b only c) a and c only d) b and c only 9. In the context of system of speed Clearing which of the following is correct (a) it leverages on Core Banking Solutions implemented in banks across the country (b) it facilitates realization of outstation cheques drawn on CBS enabled branches at the local centre itself (c) it obviates the need of such cheques to physically move to the outstation centre. a) a, b and c all b) a and b only c) a and c only d) b and c only 10. Speed Clearing is enabled for cheques issued by account holders with the following transaction codes: a) 10 (savings bank) only b) 11 (current account) only c) 13 (cash credit) only d) all transaction codes, other than those relating to government cheques. 11. The extended scope of Speed Clearing became operative w.e.f: a) Jan 01, 2011 b) Feb 01, 2011 c) Mar 01, 2011 d) Apr 01, 2011 12. As per RBI guidelines, the banks should not invest in Non-SLR securities of original maturity of less than one-year, except which of the following: a) certificate of deposit b) non-convertible debentures c) commercial paper d) all the above 13. As per extant guidelines of RBI on capital adequacy and market discipline, all the banks in India would continue to have the parallel run till __: a) Mar 31, 2011 b) Mar 31, 2012 c) Mar 31, 2013 d) Mar 31, 2014 14. As per extant guidelines of RBI on capital adequacy and market discipline, the banks are to ensure that their Basel II minimum capital requirement continues to be higher than the prudential floor of ____ of the minimum capital requirement computed as per Basel I frame work for credit and market risks. a) 75% b) 80% c) 90% d) 100% 15. As per RBI directions, the preparation of calendar of SLBC/UTLBC meetings by convenor bank and intimation to all the concerned of the cut-off dates for submission of data and dates of meetings to be sent by which of the following dates: a) by 15th January b) by 31st January c) by 15th February d) by 28th February 16. What is the dead line for submission of data to SLBC convenor bank, by difference banks in a State: a) one week from close of the quarter b) 10 days from close of the quarter c) 15 from close of the quarter d) 4 weeks from close of the quarter 17. Convenor banks is required to ensure holding of SLBC meeting within: a) 30 days from close of the half year b) 45 days from close of the half year c) 30 days from close of the quarter d) 45 days from close of the quarter 18. Which time limit fixed by RBI in connection with SLBC meeting is not correctly stated: a) reminder by convenor bank to the banks regarding exact date of meeting 15 days before end of the quarter. b) forwarding the minutes of the SLBC meeting to all stakeholders within 10 days from holding the meeting. c) follow up action points emerged from the meeting within 30 days of forwarding the minutes. d) none of the above 19. Statistics on operation of pre-paid instruments is required to be furnished to RBI by banks: a) on monthly basis, within 7 days from close of the month b) on monthly basis, within 10 days from close of the month c) on quarterly basis, within 7 days from close of the quarter d) on quarterly basis, within 10 days from close of the quarter 20. All eligible current account transactions including trade transactions with which of the following ACU country should be settled in any permitted currency outside the ACU mechanism, as per RBI directions: a) Srilanka b) Iran c) Mynamar d) Nepal

Recalled Questions Jan, 2011 21. An HUF cannot become partner in a Partnership Firm? This is as per provisions of: a) RBI directives b) SEBI guidelines c) Provisions of Hindus Law d) none of the above 22. Where a notice reading Customers can also tender cheques at the counter and obtain acknowledgement on the pay-in-slips is to be displayed by bank branches? a) on the counter b) on the entry door of the branch premises c) on the cheque drop-box itself d) at the place where other notices have been displayed 23. Which of the following RBI guidelines on Nomination is correct? a) on pass book, the name of nominee can be written with or without customer request. b) On FDR, the name of nominee can be written with or without customer request. c) on FDR and pass book the words nomination registered are to be indicated. d) all the above 24. Simplified KYC procedure is to be followed for the accounts with balance not exceeding ________ and advisory notice is to be sent to customer when the balance exceeds: a) Rs.50000, Rs.100000 b) Rs.50000, Rs.80000 c) Rs.40000, Rs.50000 d) Rs.50000, Rs.40000 25. Preservation period for records pertaining to identification documents obtained while opening the account is ___________ a) 10 Years after the business relationship is ended b) 10 Years after the date of opening the account c) 10 Years after the date of last transaction in the account d) 10 Years 26. Loans to __________ with investments in plant and machinery upto Rs.10 cr is classifed as priority sector advance to _________. a) food and agro-based processing units, SME b) food and agro-based processing units, indirect agriculture c) food and agro-based processing units, indirect SME d) Fruit processing units, indirect agrilculture 27. As per recommendations of the _______, the allocation of 60% of the MSE advances to the micro enterprises is to be achived in 2010-11 and 2011-12 is 50%; 55%. a) Prime Ministers Task force b) MSME Ministry Task Force c) RBI Working Group d) Ministry of Industruy Govt. of India Task Force. 28. The limit for collateral free loans is up to Rs.10 alc, is in respect of loans to which of the following category: a) Export loans b) Direct Agriculture Loans c) MSME units d) none of these 29. In case of restructure of Suit Filed accounts under CDR system, the consent of _______ % lenders by _____ consent of ____ is required? a) 75% by value and 75% by number b) 60% by value and 75% by number c) 75% by value and 60% by number d) in suit filed loans, CDR restructuring is not permitted 30. After taking possession of the immovable property, copy of the possession notice is to be published in two local newspaper not later than ____ days. a) 7 days b) 15 days c) 30 days d) no time limit 31. As per RBI directives, the collateral security on indirect agriculture loans in priority sector, is mandatoriy waived up to ________? a) Rs.50000 b) Rs.100000 c) Rs.200000 d) Discretion of the banks 32. Cash Budget System is useful in those cases________: a) where there is wide fluction in the level of current liabilities b) where there is wide fluctuation in the level of fixed assets and long term liabilities. c) where there is wide fluctuation in the level of current assets d) all the above. 33. Which of the following group of exporters is not eligiible for interest subvention of 2% on

Rupee Export Credit: a) Jute Manufacturing including Floor covering b) Leather and Leather Manufactures c) Engineering Goods d) Gems and Jewellary 34. Current Ratio is 1.25:1 I case of and 1.33:1 in case of ____ following method of lending: a) MPBF method, cash budget method b) MPBF method, turnover method c) Turnover method, cash budget method d) Turnover method, MPBF method 35. For delay, the penalty is Rs.250 per day and max Rs.25000 in respect of which and max Rs.25000 in respect of which of the following: a) violation of RBI guidelines by a bank b) violation of SEBI regulations by a company c) delay in providig information under RTI Act. d) delay in compensating a customer under Bank Ombudsman Scheme 36. Bank has to reimburse the customers within 12 day in respect of which of the folowing: a) cancellation of demand draft on request of the purchaser. b) wrongful payment of a cheque made from customers account c) amount wrongfully debited on account of failed ATM transactions d) all the above 37. Interest Subsidy of __ on Housing Loans is available upto the loan amount of Rs.10 lac where the cost of house does not exceed ____ : a) 0.5%, Rs.25 lac b) 1.0%, Rs.20 lac c) 1.5%, Rs.15 lac d) 2.0%, Rs.20 lac 38. Additional interest subvention of 2% for prompt payment of crop loan by farmers is available to Public Sector Banks on the condition that the effective rate of interest on such loans to farmers will be ____: a) 5% b) 7% c) 8.5% d) 9% 39. Present minimum limit for RTGS transactions is ____: a) Rs.50000 b) Rs. 1 lac c) Rs. 2 lac d) Rs.5 lac 40. NEFT batches on week days and on Saturday are: a) 11 and 5, half hourly batches b) 11 and 5, hourly batches c) 11 and 5, 2-hourly batches d) 9 and 6, 2-hourly batches 41. The cause of action has arisen on Jan 22, 2010 under Consumer Protection Act. In this case, the suit can be filed by which of the following dates; a) Feb 21, 2010 b) Jan 21, 2011 c) Jan 21, 2012 d) Jan 22, 2013 42. What is the rate of commission on receipts relating to Govt. Business.. a) Rs.5 per Rs.100 of turnover b) Rs.9 per Rs.100 of turnover c) Rs.45 per transaction d) Rs.60 per transaction 43. The ANBC represents which of the following: a) outstanding loans within a bank as on Mar 31st b) net bank credit + investment in non-SLR bonds in HTM category c) net bank credit + investment in non SLR bonds in HTM category FCNR deposits. d) net bank credit export credit. 44. Subsidy in case of SGSY loans is kept in ___. a) in an interest free fixed deposit account b) in an interest free current account c) in an interest free saving bank account d) in subsidy reserve fund. 45. Maximum project cost for USEP (Women) segment projects in SJSRY is restricted to __ per borrower: a) Rs.2 lac b) Rs.1 lac c) Rs. 50000 d) there is no ceiling 46. CGTMSE amount of guarantee cover for a balance of Rs.80 lakhs in case of an eligible normal borrower in other than NE states, will be: a) Rs.55 lac b) Rs.52.50 lac c) Rs.37.50 lac d) Rs.22.50 lac 47. A bill of lading mentions that certain packets of goods are not properly packed and packing is

defective. This is: a) claused bill of lading b) clean bill of lading c) defective bill of lading d) stale bill of lading 48. Which f the following are presently kept at 6% by RBI: a) CRR and Repo Rate b) Repo Rate and Bank Rate c) Bank Rate and CRR d) CRR and reverse repo rate 49. Risk weightage on Consumer Credit/Credit Cards, for capital adequacy ratio purpose is: a) 75% b) 100% c) 125% d) 150 % 50. Minimum period of issue which of the following instruments is not 7 days: a) non-convertible debenture b) certificate of deposit c) commercial paper d) bank FDR 51. Bank-A purchased an NPA account from Bank-B. Now Bank-A wants to sell this account to Bank-C. Bank-B can do so after: a) 15 months b) 15 months from date of account becoming NPA with Bank-B c) 15 months after 1st 90 days, when the Bank B classifies the account in standard category immediately on its purchase. d) 15 months from date of purchase of account from Bank-A. 52. Which of the following loan is exempted from application of Base Rate: a) loan given as export credit b) loan given by a consortium of banks c) loan given for direct agriculture purpose d) none of the above 53. As per provisions of which of the folllowing, the nominated bank gets a period of ___ days to verify the documents drawn drawn under a documentary credit: a) RBI guidelines, 7 banking days b) FEDAI rules, 5 banking days c) UCPDC, 5 calendar days d) none of these is correct 54. Which of the following is not a Financial Sector reform: a) introduction of NPA guidelines b) introduction of obtaining photograph of customers c) application of asset liability management rules d) none of the above 55. Which of the following is not correct regarding Safe Deposit Lockers guidelines of RBI: a) branches are to maintain a waitlist for allotment of lockers that should be transparent b) all locker allotment applications should be ackowledged c) all locker allotment applications should be given a wait list number d) bank can provide detailed terms and conditions of locker allotment to the customer but should not provide copy of the agreement regarding operation of locker. 56. Which of the following banking facility cannot be offered to visually challlenged persons (blind persons): a) cheque book facility, net banking and ATM facility b) Lockers c) retail loans, credit card d) none of these 57. Which of the following activity cannot be undertaken by Business Correspondents a) disbursement of small value loans b) collection of small value deposit c) recovery of principal and interest d) collection of transaction fee from customer 58. Interest subsidy on Housing Loans allowed to: a) weaker section borrowers only b) middle and low income groups c) only low income group borrowers d) all housing loan borrowers 59. TDS is deductible on which of the following account: a) all type of term deposit accounts including recurring deposit b) all saving bank accounts c) all NRO accounts including saving bank, where of the above d) none of the above. 60. Details of unimplemented awards of Ombudsman scheme pending for more than 3 months, are to be placed by the banks to which of the following: a) Dy. Governor RBI b) CMD/ED/CEO of the bank c) Customer Service Committee of Board d) Board of Directors of the bank

61. W.e.f. Jan 24, 2011, the official timing for opration of Centralised Funds Management System for Monday to Friday is: a) 11 am to 4.30 pm b) 11 am to 5.00 pm c) 10 am to 4.30 pm d) 10 am to 5.00 pm 62. W.e.f. Jan 24, 2011, the official timing for operation of Centralised Funds Management System for Saturday is: a) 11 am to 2.30 pm b) 11 am to 3.00 pm c) 10 am to 2.30 pm d) 10 am to 3.00 pm 63. W.e.f. 1.4.2011, the processing/service charges for local clearing (by clearing house from member banks) for clearing at MICR-CPCs for presenting bank and drawee bank are as under: a) Rs.1 and Rs.1 b) Rs.1 and Rs.1.50 c) Rs.1.50 and Rs.1 d) Rs.1.50 and Rs.1.50 64. W.e.f. 1.4.2011, the processing/service charges for local clearing (by clearing house from member banks) for clearing at cheque truncation, for presenting bank and drawee bank are as under: a) 100 p and 75 p b) 75 p and 75 p c) 50 p and 100 p d) 75 p and 50 p 65. W.e.f. 1.4.2011, the outstation cheque collection charges have been revised by RBI. Which of the following is not corect in the revision: a) up to Rs.5000 Rs.25 b) Rs.5001 to Rs.10000-Rs.50 c) Rs.10001 to Rs.1 lac Rs.100 d) none of the above 66. The charges for collection of outstation cheques as revised by RBI wef 1.04.2011 for cheques of above Rs.1 lac as under: a) Rs.100 b) Rs.150 c) 200 d) at discretion of bank 67. Service charges for cheque collection under speed clearing from customer by banks, wef 1.4.2011 for cheques up to Rs.1 lac are as under: a) Rs.100 b) Rs.150 c) Rs.200 d) nil 68. Service charges for cheque collection under speed clearing from customer by banks, wef 1.4.2011 for cheques above Rs.1 lac are as under: a) Rs.150 b) Rs.200 c) Rs.250 d) at discretion of the bank 69. RBI Sub-Committee of its Central Board of Directors to study Issues and concerns in the micro finance institutions (MFI) Sector, which submitted its report in Jan 2011, was headed by Sh. __: a) Dr. Chakravarty b) YH Malegam c) Deepak Mohanty d) VK Sharma 70. An accounting software developed for running in computer may be written in which of the programming languages: a) COBOL b) FOXPRO c) UNIX d) all of these ANSWERS GENERAL AWARENESS 2011 SEVEN 1.d 2.d 3.d 4.b 5.d 6.d 7.d 8.b 9.a 10.d 11.b 12.d 13.c 14.b 15.a 16.c 17.d 18.d 19.a 20.b 21.d 22.c 23.c 24.d 25.a 26.b 27.a 28.d 29.c 30.d 31.d 32.c 33.d 34.d 35.c 36.c 37.b 38.a 39.c 40.b 41.c 42.c 43.b 44.d 45.a 46.b 47.a 48.c 49.c 50.a 51.d 52.d 53.d 54.b 55.d 56.d 57.d 58.b 59.c 60.c 61.d 62.d 63.b 64.c 65.d 66.d 67.d 68.d 69.b 70. d

*****************************

GENERAL AWARNESS 2011 - EIGHT Latest Policy Based Questions 1. SLR has been reduced by RBI wef Dec 18,2010 from ______ to _______ of net demand and time liabilities : a) 40% to 25% b) 40% to 24% c) 25% to 24% d) 25% to 20% 2. The term headline inflation, represents which of the following a) wholesale price index b) consumer price index for industrial workers c) consumer price index for agriculture workers d) all the above taken together. 3. As per Prevention of Money laundering (Maintenance of Records of the Nature and Value of Transactions, the Procedure and Manner of Maintaining and Time for Furnishing Information and Verification and Maintenance of Records of the Identity of the Clients of the Banking Companies, Financial Institutions and Intermediaries) Second Amendment Rules, 2010, no banking company, financial institution and Intermediary, as the case may be, shall allow : a) the opening of or keep any anonymous account b) account in fictitious names c) account on behalf of other persons whose identity has not been disclosed or cannot be verified d) all the above 4. The borrowers under Swarn Jayanti Gram Swairozgar Yojna have the benefit of group life insurance provided by LIC of India. In term of the scheme, in case of natural death of the borrower, the nominee will get _________ : a) Rs.5000 b) Rs.6000 c) Rs.10000 d) Rs.12000 5. The borrowers under Swarn Jayanti Gram Swairozgar Yojna have the benefit of group life insurance provided by LIC of India. In term of the scheme, in case of death of the borrower due to accident, the nominee will get. a) Rs.5000 b) Rs.6000 c) Rs.10000 d) Rs.12000 6. A customer has received inward remittance for credit to NRE account through credit push systems like RTGA, NEFT, NECS and ECS. He request for issue of foreign inward remittance certificate (FIRC): a) the FIRC cannot be issued as per RBI guidelines in such cases. b) the FIRC cannot be issued as per FEDAI Rules in such cases. c) the FIRC can be issued if the amount is below USD 10000. d) the FIRC can be issued if the amount is below USD 100000. 7. A bank branch has received an inward remittance in foreign currency for credit to customers NRE account by converting the foreign exchange in to rupees. The customer request for issue of foreign inward remittance certificate (FIRC) a) the FIRC can be issued as per RBI guidelines in such cases. b) the FIRC can be issued as per FEDAI Rules in such cases . c) the FIRC can be issued if the amount is below USD 10000 d) the FIRC can be issued if the amount is below USD 100000 8. A salaried employee working with a multi national company of repute, wants to open a saving bank account. To comply with KYC requirement he needs to submit (1) letter / certificate issued by the company that he is working with them (2) copy of pass port or driving licence (3) copy of PAN card or voter identity card: a) only 1 b) only 1 and 2 c) only 1 and 3 d) 1 and any of 2 or 3 9. Firm A is owned Mr.A and B and Firm B is also owned by Mr.A and B. Both the firms are engaged in manufacturing and have investment in plant and machinery amounting to Rs.4 cr and

Rs.3 cr respectively. As per provisions of MSME Dev Act 2006: a) only one of them can be classified as small enterprise b) none of these can be classified as small enterprise c) both of these can be classified as small enterprise independently d) only firm B having lower investment can be classified as a small enterprise 10. X has a current account with the bank and he agrees to receive a cheque of Rs.10 lac from B and on the strength of this amount of Rs.10lac, he issues 5 cheques of Rs.1.90 each in the name of 5 different persons and retains the balance amount as his commission. Such type of transaction is know as: a) money agency transaction b) money pipe transaction c) money mule transaction d) money flow transaction 11. The money mule transactions can be restricted by banks if the banks follow (1) KYC guidelines of RBI (2) Anti money laundering standards (3) combating of financing of terrorism standards (4) obligations under PML Act 2002: a) 1 to 4 all b) only 1 and 4 c) only 1 and 3 d) only 1 and 2 12. Which of the following statement regarding non-convertible debentures is not correct: a) Financial Institutions (FIs) to invest in NCDs of maturity up to one year. b) NBFCs including Primary Dealers that do not maintain a working capital limit to issue NCDs of maturity up to one year; c) FIIs to invest in NCDs of maturity up to one year d) none of the above 13. All NBFCs are required to submit a certificate to RBI from Statutory Auditor with respect to the position of the company as on March 31st every year within _____ from the date of finalization of the balance sheet and in any case not later than ____ a) one month, Dec 30th of that year b) one month, June 30th of that year c) 2 months, Dec 30th of that year d) 2 months, June 30th of that year 14. As per RBI guidelines, NBFCs are required to finalise their balance sheet within a period ____ from the date to within it pertains. a) one month b) 2 months c) 3 months d) 6 months Questions on MSEs 15. An enterprise engaged in manufacturing, production or preservation of goods is a micro enterprise, where its investment in ____ is up to ____ : a) plant and Machinery, up to Rs.10 lac b) Plant and Machinery, up to Rs.25 lac c) Equipment, up to Rs.10 lac d) Equipment, up to Rs.25 lac 16. An enterprise engaged in providing or rendering of services is a micro enterprise, where its investment in ____ is up to ___ : a) plant and Machinery, up to Rs.10 lac b) Plant and Machinery, up to Rs.25 lac c) Equipment, up to Rs.10 lac d) Equipment, up to Rs.25 lac 17. An enterprise engaged in manufacturing, production or preservation of goods is a small enterprise, where its investment in _______ is up to _______: a) plant and Machinery, above Rs.10 lac up to Rs.200 lac b) Plant and Machinery, above Rs.25 lac up to Rs.500 lac c) Equipment, above Rs.10 lac up to Rs.200 lac d) Equipment, above Rs.25 lac up to Rs.500 lac. 18. An enterprise engaged in providing or rendering of services is a small enterprise, where its investment in ___is up to ____: a) plant and Machinery, above Rs.10 lac up to Rs.200 lac. b) plant and Machinery, above Rs.25 lac up to 500 lac c) Equipment, above Rs.10 lac up to Rs.200 lac d) Equipment , above Rs.25 lac up to Rs.500 lac 19. An enterprise engaged in manufacturing, production or preservation of goods is a medium enterprise, where its investment in ____ is up to ____ : a) Plant and Machinery, above Rs.10 lac up to Rs.200 lac b) plant and Machinery, above Rs.25 lac up to Rs.500 lac c) Plant and Machinery, above Rs.500 lac up to Rs.1000 lac d) Equipment, above Rs.500 lac up to Rs.1000 lac 20. An enterprise engaged in providing or rendering of services is a medium enterprise, where its

investment in ______ is up to ____ a) Equipment, above Rs.10 lac up to Rs.200 lac b) Equipment, above Rs.25 lac up to Rs.500 lac c) Equipment, above Rs.200 lac up to Rs.500 lac d) Equipment, above Rs.500 lac up to Rs.1000 lac 21. The Investment criteria for Micro, Small and Medium enterprises is as per provisions of : a) RBI guidelines b) Guidelines a Ministry of Industries, Govt. of India c) MSME Development Act 2006 d) Companies Act 1956 22. Which of the following activity is also part of the micro and small enterprises (1) small road and water transport operators (2) small business (3) Retail trade (d) self employed persons a) 1 to 4 all b) 1,2 and 4 only c) 1,3 and 4 only d) 2,3 and 4 only 23. All Advances granted to units in KVI sector, irrespective of size of operations, location and amount of original investment in plant and machinery are to be included in : a) 40% sub target of advances to micro enterprises within advances to MSE b) 60% sub target of advances to micro enterprises within advances to MSE c) Overall 10% target for advances to MSE within priority sector d) such advances are non priority sector advances 24. Loans to which of the following are not part of advances to priority sector, under MSME sector : a) loans to micro enterprises b) loans to small enterprises c) loans to medium enterprises d) loans to units in KVI sector of their investment in plant and machinery is above Rs.5 cr and they are not located in rural or semi urban areas 25. Which of the following types of loans are part of indirect to MSEs in priority sector: a) Persons involved in assisting the decentralized sector in the supply of inputs and marketing of outputs of artisans, village and cottage industries. b) Advances to cooperative of producers in the decentralized sector viz. artisans, village and cottage industries. c) Loans granted by banks to NBFCs for on lending to Micro and Small Enterprises (MSE) sector (manufacturing as well as service) d) all the above 26. In terms of recommendations of prime Ministers Task Force on MSMEs, the banks are required to ensure to_______ year on year growth of credit to _____: a) 20% micro, and small enterprises b) 20% micro, small and medium enterprises c) 10% micro and small enterprises d) 10%, micro, small and medium enterprises 27. In terms of recommendations of Prime Ministers Task Force on MSMEs, the banks are required to ensure a ____ increases in no of accounts for credit to ___ : a) 10% micro enterprises b) 10% micro and small enterprises c) 10% micro small and medium enterprises d) 10% all industry related enterprises 28. In terms of recommendations of Prime Ministers Task Force on MSMEs, the banks are to have ____% of their advances to MSE sector to micro enterprises, as on Mar 31,2011: a) 40% b) 50% c)55% d)60% 29. In terms of recommendations of Prime Ministers Task Force on MSMEs, the banks are to have 55% of their advances to MSE sector to micro enterprises, as on : a) Mar 31, 2010 b) Mar 31, 2011 c) Mar 31, 2012 d) Mar 31,2013 30. In terms of recommendations of Prime Ministers Task Force on MSMEs, the banks are to have 60% of their advances to MSE, sector to micro enterprises, which is to be done in stages. Which of the following does not match in this context: a) 45% should be achieved by 31.03.10 b) 50% should be achieved by 31.03.11 c) 55% should be achieved by 31.03.12 d) 60% should be achieved by 31.03.13 31. What %age of advances to MSE sector should be given by banks to micro enterprises with

investment in plant and machinery up to Rs.5 lac or investment in equipment up to Rs.2 lac. a) 20% b)25% c) 40% d) 60% 32. 20% of advances to MSE sector should be given by banks to micro enterprises with investment in : a) plant and machinery up to Rs.5 lac or investment in equipment up to Rs.2 lac. b) plant and machinery above Rs.5 lac or investment in above Rs.2 lac. c) plant and machinery above Rs.5 lac up to Rs.10 lac or investment in equipment above Rs.2 lac up to Rs. 5 lac. d) plant and machinery above Rs.5 lac up to Rs.25 lac or investment in equipment above Rs.2 lac up to Rs.10 lac. 33. An MSME sector loan application for an amount up to Rs.5 lac is to be disposed of within: a) 2 weeks b) 4 weeks c) 6 weeks d) 8-9 weeks 34. An MSME sector loan application for an amount up to Rs.25000 is to be disposed of within : a) 2 weeks b) 4 weeks c) 6 weeks d) 8-9 weeks 35. An MSME sector loan application is to be disposed of within 8-9 weeks where the amount of loan application is : a) up to Rs.25000 b) up to Rs.5 lac c) above Rs.5 lac d) above Rs.25 lac 36. Banks have been mandated by RBI not to accept collateral security in case of MSE loans for an amount up to : a) Rs.2 lac b) Rs.5 lac c) Rs.10 lac d) Rs.25 lac 37. In case of MSE loan accounts with good track record and financial position, the banks can consider loan proposals for an amount up to Rs. ____, without a collateral security : a) Rs. 2 Lac b) Rs. 5 Lac c) Rs. 10 Lac d) Rs. 25 Lac 38. A Composite loan limit ___ can be sanctioned by banks to enable the MSE entrepreneurs to avail of their working capital and term loan requirement through Single Window a) up to Rs.25 lac b) up to Rs.50 lac c) up to Rs. 100 lac d) up to Rs. 200 lac 39. Banks can categorise their MSME general banking branches as specialized MSE branch provided _____ of their advances in the branch are to MSME sector : a) 40% or more b) 50% or more c) 60% or more d) 80% or more 40. A unit is considered as sick MSE unit (1) when any of the borrowal account of the unit remains substandard for more than 6 months or (2) there is erosion in the net worth due to accumulated cash losses to the extent of 50% of its net worth during the previous accounting year and (3) the unit has been in commercial production for at least two years. a) 1 and 2 only b) 2 and 3 only c) 1 and 3 only d) 1 to 3 all 41. As per RBI guidelines, the rehabilitation package relating to an MSE, unit, should by fully implemented within _____ months from the date the unit is declared as potentially viable / viable. a) 2 months b) 3 months c) 4 months d) 6 months 42. Under the Credit Linked Capital Subsidy Scheme (CLSS) for Technology Upgradation of Micro and Small Enterprises, the ceiling on loan amount is restricted to : a) Rs. 25 lac b) Rs. 50 lac c) Rs.100 lac d) Rs. 500 lac 43. Under the Credit Linked Capital Subsidy Scheme (CLSS) for Technology Upgradation of Micro and Small Enterprises, the rate of subsidy is : a) 12% of the purchase price of plant and machinery b) 15% of the purchase price of plant and machinery c) 12% of the loan amount d) 15% of the loan amount 44. Which of the following committee is not associated with lending to MSE sector : a) Kapoor Committee b) Deepak Mohanty Committee c) Ganguly Committee d) Nayak Committee 45. Which of the following is to be excluded while calculating the original price of plant and machinery or equipment in case of MSE units: (i) equipment such as tools, jigs, dyes, moulds and spare parts for maintenance and the cost of consumables stores; (ii) installation of plant and machinery; (iii) research and development equipment and pollution controlled equipment (iv) power generation set and extra transformer installed by the enterprise as per regulations of the

State Electricity Board; a) I to iv all b) I, ii and iv only c) ii, iii and iv only d) I, iii and iv only 46. Registration of which of the following group of enterprises, is mandatory under provisions of MSME guidelines: a) micro, small and medium enterprises b) micro and small enterprises engaged in manufacturing c) medium enterprises engaged in manufacturing, production d) medium enterprises engaged in manufacturing, production and providing service. 47. An ancillary unit in SSI is one that supplies ___% of its production to one or more industrial undertaking : a) at least 50% b) 50% or more c) max 50% d) 49% or more CGFT-MSE Guarantee 48. Which among the following borrower is not eligible for CGT MSE guarantee : a) new and existing micro & small enterprise b) IT or software industry c) select activities in case of agri clinics and agri business centres d) Rehabilitation of SSI units covered under CGF e) medium enterprises availing loans up to Rs.25 lac 49. The type of loan and the amount of loan, which is eligible for CGTMSE guarantee cover, is one of the following : a) only fund based working capital limits, up to Rs.100 lac b) only fund based working capital limits and term loans, up to Rs.100 lac c) only fund based working capital limits, term loans and bank guarantee, up to Rs.100 lac d) fund based working capital limits, term loans and non fund based limits all, up to Rs.100 lac. 50. Guarantee cover under CGTMSE scheme is available: (1) for a period as long as the bank and borrower decide to avail (2) for the term loan repayment period (3) for max 5 years where working capital limit alone is sanctioned (4) maximum for 7 years in all cases a) only 1 is correct b) only 2 and 3 are correct c) only 4 is correct d) 2 and 4 are correct 51. The rate of guarantee fee (up front) under CGTMSE scheme in which of the following cases is not stated correctly : a) general category loans 1.5% of sanctioned limit b) loans to SC.ST 1.0% of sanctioned limit c) loans up to Rs.5 lac to micro enterprises 1.0% of sanctioned limit d) laons to units in NE States 0.75% of sanctioned limit. 52. The rate of service fee (renewal) under CGTMSE scheme in which of the following cases is not stated correctly : a) general category loans 0.75% of sanctioned limit. b) loans to SC.ST 0.75% of sanctioned limit c) loans up to Rs.5 lac to micro enterprises 0.5% of sanctioned limit d) loans to units in NE States and women enterprises 0.50% of sanctioned limit 53. The time limit for payment of guarantee fee and service fee under CGTMSE scheme is as under : a) guarantee fee within one month from date of first disbursement and service fee within 2 months from 31st Mar b) guarantee fee within 2 months from date of first disbursement and service fee within 2 months from 31st Mar c) guarantee fee within one month from date of first disbursement and service fee within 1 month from 31st Mar d) guarantee fee within 2 months from date of first disbursement and service fee within 1 month from 31st Mar. 54. The guarantee cover under CGTMSE scheme for women enterprises and loans sanctioned in NE states is ____ % of the balance on date of the account identified as NPA as per RBI guidelines (which statement is wrong) a) up to Rs.5 lac, 85% max Rs.4.25 lac b) above Rs.5 lac up to Rs.50 lac, 80% max Rs.40 lac c) above Rs.50 lac up to Rs.100 lac, 50% max Rs.25 lac

d) total maximum cover amount Rs.65 lac. 55. The guarantee cover under CGTMSE scheme for micro enterprises other than the women enterprises and units is NE states is ___% of the balance on date of the account identified as NPA as per RBI guidelines (locate the false statement) a) up to Rs.5 1ac, 85% max Rs.4.25 lac b) up to Rs.50 lac, 75% max Rs.37.50 lac c) above Rs.50 lac up to Rs.100 lac, 50% max Rs.25 lac d) total maximum cover Rs.65 lac 56. The outstanding balance in a loan to a women enterprise and guaranteed under CTGMSE scheme on date of NPA is Rs.60 lac. what is the amount of claim that bank will be eligible for : a) Rs.45.00 lac b) Rs. 42.50 lac c) Rs.40.00 lac d) Rs.37.50 lac 57. The outstanding balance in a loan to a micro enterprises and guaranteed under CTGMSE scheme on date of NPA is Rs. 60 lac. What is the amount of claim that bank will be eligible for : a) Rs.45.00 lac b) Rs.42.50 lac c) Rs.40.00 lac d) Rs.37.50 lac 58. In a loan guaranteed under CGTMSE schemes, the bank can claim the amount if the following conditions are satisfied (1) guarantee cover is in force (2) account is classified as NPA (3) loan has been recalled and recovery proceedings have been initiated (4) loan has been written off, a) 1 to 4 all b) 1 to 3 only c) 1,2 and 4 only d) 2,3 and 4 only 59. Under CGTMSE guarantee scheme the claim can be lodged (1) within 6 months from date of account becoming NPA (2) within 1 year from date of account becoming NPA (3) within 6 months from date of completion of 18 months lock in period (4) within 12 months from date of completion of 18 months lock in period a) 1 and 4 are correct b) 1 and 3 are correct c) 2 and 3 are correct d) 2 and 4 are correct 60. Under the CGTMSE guarantee scheme, the payment of eligible claims is made as under by the guarantee fund: a) 100% amount immediately within 30 days b) 75% amount within 30 days and balance 25% on conclusion of recovery proceedings c) 75% amount within 45 days and balance 25% on conclusion of recovery proceedings d) 50% amount within 30 days and balance 50% on conclusion of recovery proceedings. ********** ANSWERSGENERAL AWARENESS EIGHT 1.c 2.a 3.d 4.b 5.d 6.a 7.a 8.d 9.c 10.c 11.a 12.d 13.a 14.c 15.b 16.c 17.b 18.c 19.c 20.c 21.c 22.a 23.b 24.c 25.d 26.a 27.a 28.b 29.b 30.a 31.c 32.d 33.b 34.a 35.c 36.c 37.d 38.c 39.c 40.d 41.d 42.c 43.b 44.b 45.a 46.c 47.a 48.e 49.d 50.b 51.b 52.d 53.a 54.b 55.d 56.a 57.b 58.b 59. 60.b **********************

41. Q. What is a Balance Sheet of a company? Ans. Balance sheet : It is a statement of accounts, generally of a business concern,prepared at the end of a year,showing debits and credits under broad heads, to find out the profit and loss position. 42. Q. What is a Bankers Cheque? Ans. Banker's Cheque : A cheque by one bank on another. 43. Q. What is meant by Bank Rate? Ans. Bank Rate : It is the rate of interest charged by the Reserve Bank of India for lending money to commercial banks. 44. Q. What is Black Money? Ans. Black Money : It means unaccounted money. 45. Q. Define Bear in stock exchanges: Ans. Bear : A speculator in the stock market who believes that prices will go down. 46. Q. What is a bearer in the context of cheques? Ans. Bearer : This term on cheques and bills denotes that any person holding the same has the same right in respect of it, as the person who issued it. 47. Q. What is a Bonus in the context of shares of companies? Ans. Bonus : It is in addition to normal payment of dividend to shareholders by a company, or an extra gratuity paid to workers by the employer. 48. Q. What is a Budget? Ans. Budget : An estimate of expected revenues and expenditure for a given period,usually a year, item by item. 49. Q. What is meant by Budget deficit? Ans. Budget deficit : When the expenditure of the government exceeds the revenue, the balance between the two is the budget deficit. 50. Q. What do you understand by the term Bulls in the context of stock market? Ans. Bulls : Speculators in the stock markets who buy goods, in some cases without money to pay with, anticipating that prices will go up. 51. Q. What is a Buyers Market situation? Ans. Buyer's Market : An area in which the supply of certain goods exceeds the demands so that purchases can drive hard bargains. 52.Q. What is a carat? Ans. Carat : Measure or weight for precious stones. 24 carat gold is the purest gold, thus 22 carat gold means a piece of gold in which 22 parts of pure gold and 2 parts of an alloy, usually copper. 53. Q. What is a cartel in the context of business? Ans. Cartel : It is a combination of business, generally in the same trade formed with a view to controlling price and enjoying monopoly. 54. Q. What is call money? Ans. Call money : Loan made for a very short period in the finance market mostly among banks. It carries rate of interest depending on the demand/supply position from time to time.

55. Q. What is a commercial bank? Ans. Commercial Banks : Financial institutions that create credit accept deposits, give loans and perform other financial functions. They create credit by creation deposits on the basis of their cash reserve ratio. 56. Q. What is meant by deflation? Ans. Deflation : It is a state in monetary market when money in circulation has decreased and is characterised by low prices, unemployment,etc. 57. Q. What is depreciation? Ans. Depreciation : Reduction in the value of a fixed assets due to wear and tear. 58. Q. What is devaluation of currency? Ans. Devaluation : Official reduction in the foreign value of domestic currency. It is done to encourage the country's exports and discourage imports. 59. Q. What is dumping? Ans. Dumping : Sale of a commodity at different prices in different markets, lower price being charged in a market where demand is relatively elastic. 60. Q. What do you understand by the term Exchange rate? Ans. Exchange Rate : The rate at which central banks will exchange one country's currency for another.
Posted by Sambasivan.S at 10:26 AM 0 comments Labels: BANKING RELATED ECONOMICS QUIZ MONDAY, AUGUST 15, 2011 Links to this post

BANKING RELATED ECONOMICS QUIZ -2


HAPPY INDEPENDENCE DAY I am posting Economics quiz -- This will help u in written examinations and also in bank interviews. If u have many questions like this please send them to samba.ssivan@gmail.com so that I can post them and it will be of use for many friends. Please go through other economics objective questions posted herein. also search other subjects relating to banking examinations, download and save in ur computers. U may use these in other forums. Please read Economics Quiz -1 also posted yesterday. 21. Q. Marginal utility, which a customer derives from a good, is A. The change in his total utility as a result of adding one unit to his stock of a good. 22. Q. The "Marginal Productivity Theory of Wages" states that A. The demand for labour is determined by the marginal physical product of labour multiplied by its marginal revenue. 23. Q. Which bank was merged with Oriental Bank of Commerce on RBI's directions? A. Global Trust Bank 24. Q. The difference between the monetary policy and the fiscal policy is

A. Fiscal policy tries to control aggregate demand through budgetary means whereas monetary policy controls the same through the quantity of money and the rate of interest. 25. Q. The major field research of the Nobel laureate, Amartya Sen, is A. Welfare Economics 26. Q. "Smart Money" is a term used for A. Credit Card 27. Q. What is "Dumping"? A. It is the sale of the products and services by foreign nations and firms at very low prices so that they could find firm markets in India. 28. Q. The Five Year Plans, in the context of Indian economy, have A. Worked but the benefits have not reached the masses. 29. Q. India has her only OTCEI in A. Mumbai 30. Q. WTO is dedicated to A. Removal of all the trade protection procedures by all the member nations. 31. Q. The basic object of SIDBI is to A. Grant loans to agricultural farmers for export of their commodities. 32. Q. The major function of the Food Corporation of India is: A. To Check the fluctuation in food grain price by controlling supply 33. Q. The major task of CRISIL is to A. Give credit rating to the firms that want to collect funds from the stock markets through shares and debentures. 34. Q. During the bullish market trends, A. The prices off shares go up 35. Q. NNP (Net National Product) or National Income is the money value of A. Final goods and services produced annually in the economy 36. Q. Marginal Revenue would be negative if the elasticity of demand is A. Inelastic 37. Q. According to JM Keynes, the relationship of National Income(Y), Investment (I) and Consumption (C) is as follows:

A. Y = C + I 38. Q. What is Annuity Payments? Ans. Annuity : A fixed amount paid once a year or at interval of a stipulated period. 39. Q. What is Arbitration? Ans. Arbitration : A method for solving disputes, generally of an industrial nature, between the employer and his employees. 40. Q. What is Balance of Trade? Ans. Balance of Trade(or Payment) : The differance between the visible exports and visible imports of two countries in trade with each other is called balance of payment.If the difference is positive the balance of payment(BOP)is called favourable and if negative it is called unfavourable.
Posted by Sambasivan.S at 12:29 PM 0 comments Links to this post Labels: BANKING RELATED ECONOMICS QUIZ, CWE PO SUNDAY, AUGUST 14, 2011

BANKING RELATED ECONOMICS QUIZ -1


I am posting Economics quiz -- This will help u in written examinations and also in bank interviews. If u have many questions like this please send them to samba.ssivan@gmail.com so that I can post them and it will be of use for many friends. Please go through other economics objective questions posted herein. also search other subjects relating to banking examinations, download and save in ur computers. U may use these in other forums. ECONOMICS 1. Q. The term paper-gold is associated with A. Special Drawing Rights (SDRs) from the IMF. 2. Q. TRAI refers to A. Telecommunication Research Authority of India. 3.Q. Disguised Unemployment means A. More persons employed for a job that only a few could accomplish 4. Q. BIFR stands for A. Board of Industrial and Financial Reconstruction 5. Q. National Remote Sensing Agency (NRSA) is located at A. Hyderabad 6. Q. Professor Amartya Sen is the _____ Indian to win a Noble Prize A. Sixth 7. Q. Operation Flood-II, the prestigious milk products revolution of India was initiated on A. October 2, 1979 8. Q. Operation Flood is related to _____ whereas development of fisheries is related to _____ A. White revolution, Blue Revolution 9. Q. Mixed Farming refers to A. Agriculture and dairy farming clubbed together 10. Q. According to Census-2001 the density of population in India is around A. 324 11. Q. NPA stands for A. Non Performing Assets 12. Q. What is common in IRDP, JRY, DDP, DPAP? A. These are the schemes initiated by the central government for eliminating poverty and unemployment in the rural areas 13. Q. The World Bank, also known as the International Bank of Reconstruction and

Development is located at A. Washington 14. Q. Perfect Competition is said to exist if A. Every firm in the industry accepts the market price because it has to 15. Q. A firm would be optimum size if A. Average total cost is minimum 16. Q. The Two gaps theory refers to A. Savings gap and exchange gap 17. Q. The costs, which vary with the output, are termed as A. Overhead costs 18. Q. If the demand and supply schedules for a commodity increases by an equal amount, the market price would tend to A. Remain unchanged 19. Q. Customs duties, export duties, corporation taxes, taxes on capital value of assets (excluding agricultural land of individuals and companies) are A. Taxes and duties which accrue wholly to the central government. 20. Q. Deflation means A. A contraction in the volume of money or credit that results in a decline of price level.
Posted by Sambasivan.S at 6:59 AM 0 comments

61. Q. What is meant by Excise duty? Excise Duty : Tax imposed on the manufacture, sale to the consumption of various commodities, such as taxes on textiles, cloth, liquor, etc. 62. Q. What is meant by Fiscal Policy? Ans. Fiscal Policy : Governments expenditure and Tax policy. 63. Q. What is meant by Foreign Exchange? Ans. Foreign Exchange : Claims on a country by another, held in the form of currency of that country. Foreign exchange system enables one currency to be exchanged for another, thus facilitating trade between countries. 64. Q. What is Gross Domestic Product? Ans. Gross Domestic Product (GDP) : A measure of the total flow of goods and services produced by the economy over a specific time period, normally a year. It is obtained by valuing output of goods and services at market prices and then aggregating. 65. Q. What are Indirect Taxes? Ans. Indirect Taxes : Taxes levied on goods purchased by the consumer for which the tax payer's liabilities varies in proportion to the quantity of particular goods purchased or sold.

66. Q. What do you understand by Laissez-faire? Ans. Laissez-faire : The principle of non-intervention of government in economic affairs. 67. Q. What is meant by mixed economy? Ans. Mixed Economy : Total economy in which there is a unique blend of public sector and private sector co-exist. The perfect example is India. 68.Q. What is National Income? Ans. National Income : Total of all incomes earned or input to factors of production, used in economic literature to represent the output or income of an economy in a simple fashion. 69. Q. What is per capital income? Ans. Per Capita Income : Total GNP of a country divided by the total population. 70. Q. What are Patents? Ans. Patents : It is an exclusive right granted under the patents Act to the inventor for a new invention. 71. Q. What are preference shares? Ans. Preference Shares : These are the shares entitled to a fixed dividend before any distribution of profits can be made amongst the holders of ordinary shares or stock. 72. Q. What is public sector? Ans. Public Sector : A term which is generally applied to state enterprises, ie, those companies which are nationalised and run by the government. 73. Q. What is recession? Ans. Recession : It happens when there is excess of production over demand. 74. Q. What is ad valorem tariff? Ans. Tariff (ad valorem) : A fixed percentage tax on the value of an imported commodity, levied at the point of entry into the importing country 75. Q. What is meant by Value Added Tax? Value Added Tax (VAT) : A tax levied on the values that are added to goods and services turned out by the producers during stages of production and distribution. 76. Q. What is Zero Based Budgeting? Zero Based Budgeting (ZBB) : The practice of justifying the utility in cost benefit terms of each government expenditure on projects. The ZBB Technique involves a critical review of every scheme before a budgetary provision is made in its favour. If ZBB is properly implemented it could help to reverse the trend of large deficits on the revenue account of the Union Government. 77. Q. What do the terms CRR and SLR mean? CRR, or cash reserve ratio, refers to a portion of deposits (as cash) which banks have to keep/maintain with the RBI. This serves two purposes. It ensures that a portion of bank deposits is totally risk-free and secondly it enables that RBI control liquidity in the system, and thereby, inflation. Besides the CRR, banks are required to invest a portion of their deposits in government securities as a part of their statutory liquidity ratio (SLR) requirements. The government securities (also known as gilt-edged securities or gilts) are bonds issued by the Central government to meet its revenue requirements. Although the bonds are long-term in nature, they are liquid as they can be traded in the secondary market. Since 1991, as the economy has recovered and sector reforms increased, the CRR has fallen from 15 per cent in March 1991 to 5.0 per cent in March 2009. The SLR has fallen from 38.5 per

cent to 24 per cent over the past few years. (as on 5.3.2009) 78. Q. What impact does a cut in CRR have on interest rates? From time to time, RBI prescribes a CRR or the minimum amount of cash that banks have to maintain with it. A cut in CRR will result in more money available in the hands of banker and relatively their need for funds will be less. Hence banks will be offering lower rate of interest on their deposits. 79. Q. How does the Monetary Policy affect the domestic industry and exporters in particular? Exporters look forward to the monetary policy since the central bank always makes an announcement on export refinance, or the rate at which the RBI will lend to banks which have advanced pre-shipment credit to exporters. A lowering of these rates would mean lower borrowing costs for the exporter. 80. Q. The stock markets and money move similarly, in some ways. Why? Most people attribute the link between the amount of money in the economy and movements in stock markets to the amount of liquidity in the system. This is not entirely true. The factor connecting money and stocks is interest rates. People save to get returns on their savings. In true market conditions, this made bank deposits or bonds (whose returns are linked to interest rates) and stocks (whose returns are linked to capital gains), competitors for people's savings. A hike in interest rates would tend to suck money out of shares into bonds or deposits; a fall would have the opposite effect. This argument has survived econometric tests and practical experience.
Posted by Sambasivan.S at 9:45 AM 0 co

TEST II MARKETING AND COMPUTER AWARENESS 41. A coloured or crooked perception of marketing and short-sightedness about business is a) marketing myopia b) marketing mix c) product concept d) both a) and b) e) None of these. 42. Money market is market for a) Long term loans b) short loans c) medium term loans d) both a) and b) e) None of these. 43. Market with one buyer and one seller a) Monopoly b) bi monopoly c) bilateral d) both a) and b) e) None of these. 44. Marketing management is a term ________ than sales management a) Wider b) narrower c) equal d) both a) and b) e) None of these. 45. Promoting mix includes a) Advertising, publicity, personal selling and promotion b) Physical attributes, branch package and product life cycle c) Brand name, product benefits, credit delivery d) both a) and b) e) None of these. 46. The systematic gathering and analyzing of data on a particular marketing problem is a) Promotional research b) marketing information c) Marketing research d) both a) and b) e) None of these.

47. A pricing strategy aimed at paying current or past due bills is a) Survival pricing b) brand pricing c) status quo pricing d) both a) and b) e) None of these. 48. Marketing provides maximum satisfaction of a) Profits b) human wants c)soci al wants d) both a) and b) e) None of these. 49. Moral advertising is also known as a) Outdoor advertising b) direct mail advertising c)open advg. d) both a) and b) e) None of these. 50. SBI Life Insurance agent is an example of a) Two level marketing b) Three level marketing c) direct marketing d) both a) and b) e) None of these. 51. Marketing manager may be classified in the category of a) Top level management b) middle level management c) Lower level management d) both a) and b) e) None of these. 52. Selling costs are found under a) Monopoly b) monopolistic competition c) oligopoly d) both a) and b) e) None of these. 53. Product differentiation is an important characteristic of a) Perfect market b) imperfect market c) monopolistic competition d) both a) and b) e) None of these. 54. Many firm producing homogeneous products can be generally treated as a) Perfect competition b) bilateral monopoly c) oligopoly d) both a) and b) e) None of these. 55. ________ means giving a name to a product so as to distinguish it from others a) Grading b) branding c) promotion d) both a) and b) e) None of these. 56. Who coined the expression of marketing mix? a) Adam Smith b) James Colleton c) Peter F Drucker d) both a) and b) e) None of these. 57. With you all the Way is the slogan of a) Bank of India b) Union bank of India c) State Bank of India d) IOB e) None of these. 58. In which promotion is necessary a) introduction b) growth c) decline d) both a) and b) e) None of these. 59. MRP stands for a) management Resource planning b) Marketing research planning c) Manufacturing resource planning d) both a) and b) e) None of these. 60. Customer satisfying process is the view of a) selling b) marketing c) production d) both a) and b) e) None of these.

61. Database redesign is not terribly difficult if the .. a. database is structured b. database is well designed c. database ins relatively small d. database has no data e. database is relatively large 62. Which of the following are normally used to initialize a computer systems hardware? a. Bootstrap memory b. Volatile memory c. External mass memory d. Static memory e. Random access memory 63. Which of the following is not true about two-tier client-server database architecture? a. SQL statements are processed on the server b. SQL statements may be processed on some of the clients c. Business login is mostly processed on clients d. Business logic may be processed on the server e. None of these 64. The process by which the structure of the database is modified to eliminate hidden dependencies and repeating groups is .. a. Enforcing data integrity b. Enforcing referential integrity c. Indexing d. Normalization e. None of these 65. Which of the following functions is not performed by TCP? a. Flow control b. Sequencing c. Error checking d. Subnetting e. None of these 66. In Oracle, who owns the data dictionary a. Oracle b. SYS c. The DBA d. SYSTEM e. None of these 67. The reason the data outputs of most ROM ICs are tri-state outputs is to .. a. Allow for three separate data input lines b. Allow the bidirectional flow of data between the bus line and the ROM registers. c. Permit the connection of many ROM chips to a common data bus d. Isolate the registers from the data bus during read operations e. None of these 68. To drop a column that is used as a foreign key, first .. a. drop the primary key b. drop the table c. drop the foreign key constraint d. all of these e. None of these 69. In the straight CGI approach to database connectivity on the internet .. a. The external program is located between the client and the web server b. The external program is located between the database server and the database c. The external program is located between the web server and the database server d. There is no external program e. None of these 70. What is the name of the protocol used to eliminate loops? a. Switching b. ISL c. Frame Tagging d. Spanning Tree Protocol e. Scanning 71. The effect of the ROLLBACK command in a transaction is the following .. a. Undo all changes to the database resulting from the execution of the transaction b. Undo the effects of the last UPDATE command c. Restore the content of the database to its state at the end of the previous day d. Make sure that all changes to the database are in effect e. None of these 72. In Oracle, what is the default number of transactions that MAXTRANS is set to if not specified? a. 512 b. 10 c. 40 d. 1 e. 255 73. To which pin on the RAM chip does the address decoder connect in order to signal which memory chip is being accessed? a. The address input b. The output enable c. The chip enable d. The data input e. The data output 74. Which of the following switching methods provides the greatest frame throughout? a. Store-and forward switching b. Frame-tag switching c. Cut-through switching d. ISL switching e. Store-and-backward switching 75. Which kind of lock includes a keypad that can be used to control access into areas? a. Cipher b. Warded c. device d. Tumbler e. Typelock 76. A gateway is .. a. A device that enables users to send information simultaneously over a network without slowing

down transmission b. An electronic device that connects several computing devices to a network c. A point in one network that is an entrance point to another network d. A device that links two or more segments of a network and helps direct traffic e. None of these 77. Which of the following is considered a vulnerability of SNMP? a. Clear text community strings b. Its use of TCP c. The fact that it is on by default in Windows 2000 server d. The fact that it is on by default in Windows XP Professional e. None of these 78. Network routing information distributed among routers is stored in which of the following a. Flash memory b. Route table c. Metric table d. NVRAM e. Router memory 79. If the destination did not receive a segment how will the TCP host know to resent the information? a. The ACK received will not include the segment number that was not received b. The ACK received will include the segment number that was not received c. The sending host will send a PACK to verify segment receipt d. The destination host will send a YACK message back to the sending host e. None of these 80. What are the effects of mixing RAM modules with different speed ratings? a. The system suns at the speed of the slowest RAM stick b. The systems run normally c. The system runs at the speed of the memory bus d. The system may not run or it crashes periodically e. None of these

For SBI Associate PO & CWE -Economics One


ECONOMICS--ONE Note: If none of the answers is correct then mark E (None of these) as your answer. 1. Indravati Hydroelectric Project is the multipurpose project of --a. Maharashtra State b. Gujarat State c. Orissa State d. Tamil Nadu 2. What is a limited company? a. In which shareholders possess the ownership limited to their paid up capital b. In which shares are not issued c. A company of Government ownership d. A Registered company 3. Who gains in inflation? a. Saver b. Creditor c. Debtor d. Pension holder 4. Open market operation is a part of ---

a. Income Policy b. Fiscal Policy c. Credit Policy d. Labour Policy 5. CRISIL --a. Evaluates the credit documents of a company b. Provides financial assistance to industries c. Provides finances for rural areas d. Promotes exports 6. Which among the following states is a leading industrialized state? a. West Bengal b. Orissa c. Maharashtra d. Madhya Pradesh 7. On the recommendation of Jankiraman Committee, a committee was constituted for improving transaction method of public sector bonds and mutual funds. This committee was --a. Nadkarni committee b. Nayak committee c. Goiporia committee d. Tikku committee 8. GIC includes various insurance companies. This number is --a. 2 b. 3 c. 4 d. 5 9. LIC of India was established in a. 1897 b. 1950 c. 1956 d. 1965 10. Consumer Protection Act (COPRA) was implemented in --a. 1985 b. 1986 c. 1987 d. 1988 11. Which direct tax gives maximum net revenue to the Government --a. Corporate Tax b. Income Tax c. Wealth Tax d. Gift Tax 12. The main food grains of India is a. Rice b. Wheat c. Sugarcane d. Maize 13. Which rank India attains in tobacco production in the world? a. First b. Second c. Sixth d. Fourth 14. Scheduled Bank is that bank which is --a. Nationalized

b. Not nationalized c. Based at foreign country d. Included in the second schedule of RBI 15. Match the following --Unit I Unit II a. Aqua culture 1. Silk b. Floriculture 2. Grapes c. Seri culture 3. Flower d. Viti culture 4. Fisheries a. b. c. d. a. 4 3 2 1 b. 3 4 1 2 c. 3 4 2 1 d. 4 3 1 2 16. Which state has the lowest women literary rate in India (2011 census)? a. Bihar b. Jharkhand c. U.P. d. Rajasthan 17. Which country is the largest oil producing country of the world? a. Saudi Arabia b. Russia c. America d. None of these 18. The ownership right of a company remains --a. With company Directors b. With equity shareholders c. With debenture holders d. With the government 19. TRIPS Trade Related Intellectual Property Rights and TRIMS Trade Related Investment Measures are associated with --a. Preston Proposal b. Dunkel Proposal c. Chelliah Committee d. None of these 20. Which is true for Finance Commission? a. It is a statutory body b. It is constituted under article 280 of the constitution c. Mr. K.C. Pant was Chairman of 10th Finance Commission d. All of these 21. According to Banking Regulation Act, RBI can fix SLR up to the ceiling of --a. 40% b. 50% c. 30% d. 45% 22. What was the field of Sunderrajan Committee? a. Petroleum b. Small Industry c. Education d. Power

23. Corporate Tax is imposed by --a. State Government b. Local Government c. Central Government d. Both Centre and State Government 24. Marginal Propensity to Consume (MPC) means When income rises consumption? a. Remains constant b. Increases in similar proportion c. Increases in higher proportion d. Increases in lesser proportion 25. What is needed for creating demand? a. Production b. Price c. Income d. Import 26. IMF is the result of --a. Hawana conference b. Rome conference c. Brettonwood conference d. Geneva conference 27. Primary gold is a gold of --a. 20 carat b. 22 carat c. 23 carat d. 24 carat 28. Who is not the member of National Development Council? a. Governors of the State b. Chief Ministers of the State c. Administrators of Union Territory d. All ministers of Union Cabinet 29. The demand for agriculture product mainly depends upon --a. Increase in agriculture area b. Increase in population c. Increase in income d. Fall in price 30. The Headquarter of ILO is at --a. Washington b. Geneva c. The Hague d. Manila 31. According to 2011 census which State had the highest literacy rate? a. Kerala b.Mizoram c. Goa d. H.P. 32. According to 2011 census what is the percentage of India's population to that of world population? a. 25 .5 b.17.5% c.30.5 d. 12.5 33. What is the total population of Indi as per 2011 census? a. 131.02 crore b. 101.02 crore c. 121.02 crore d. 111.02 crore 34. What is the total population of China? a. 114.10 b.124.10 c.104.10 d.134.10

35. Which country stands third place in total population ? a. Indonesia b. USA c. India c. Canada 36. According to 2011 census which district in India has highest density? a. North East Delhi b. Kolkata c. Chennai d. Mumbai 37. According to 2011 census Child sex ratio number of female children per 1000 male children is: a. 814 b.1014 c.914 d. 714 38 First census of India was taken in which year?. a. 1972 b. 1772 c. 1902 d. 1872 39. The Census of India done in 2011 is ....... census. a. 10th b. 15th c. 20th d. 18th 40. Which state had a dip in literacy rate in 2011 census compared to 2001 census? a. Andhra pradesh b. Punjab c . Orissa d. Goa Q.41 Industrial sickness is growing at an alarming rate and is causing concern to the (a) Financial Institutions (b) Banks (c) Government (d) All the above Q.42 Industrial Sickness may be caused by (a) Unfavourable external environment (b) Managerial deficiencies (c) Any of the above (d) All the above Q.43 Industrial sickness occurs (a) Overnight (b) Develops gradually over time Both (a) and (b) (d) All of thesee Q.44 In a leasing agreement, lessee gets a right to (a) Use the asset (b) Sell the asset Both the above (d) All of these Q.45 A leasing company can grow profitably over time, provided it offers (a) Competitive rates (b) Earns a satisfactory rate of return on its investments (c) Both (a) and (b) (d) All of these Q.46 For assessing the financial health of a company, one requires an understanding of (a) Future industry economies and structure (b) Competitive and operating characteristics of business (c) Long term goals and plans of management (d) All the above Q.47 Projects costing upto Rs. 5 crore are financed by

(a) State level financial institutions and banks (b) All India Financial Institutions (c) Both (a) and (b) (d) All of these. Q.48 Credit worthiness of a potential customer relating to his capital variable, can be judged by (a) Quick Ratio (b) Current Ratio (c) Net Worth (d) Cash Flow Q.49 Fund Flow Statement on working capital (CA: CL) basis presents the (a) Source of working capital (b) Uses of working capital (c) Net change in working capital (d) All the above Q.50 Fund Flow Statement on Cash basis shows the (a) Sources of cash (b) Users of cash (c) Net change in Cash (d) All the above ECONOMICS --ONE 1.C 2.A 3.C 4.C 5.A 6.C 7.A 8.C 9.C 10.B 11.A 12.A 13.B 14.D 15.D 16.A 17.C 18.B 19.B 20.D 21.A 22.A 23.C 24.A 25.C 26.C 27.D 28.A 29.B 30.B 31.A 32.B 33.C 34.D 35.B 36.A 37.C 38.D 39.B 40.A 41.D 42.C 43.B 44.C 45.C 46.D 47.A 48.A 49.F 50.D

BANK OF BARODA PO EXAM SOLVED QUESTION PAPER


Bank of Baroda Probationary Officers Exam., 2008 (Held on 5-10-2008) General Socio-Economic & Banking Awareness: Solved Paper 1. RBIs open market operation transactions are carried out with a view to regulate (A) Liquidity in the economy (B) Prices of essential commodities (C) Inflation (D) Borrowing power of the banks (E) All the above 2. When more than one banks are allowing credit facilities to one party in coordination with each other under a formal arrangement, the arrangement is generally known as (A) Participation

(B) Consortium (C) Syndication (D) Multiple banking (E) None of these 3. Open market operations, one of the measures taken by RBI in order to control credit expansion in the economy means (A) Sale or purchase of Govt. securities (B) Issuance of different types of bonds (C) Auction of gold (D) To make available direct finance to borrowers (E) None of these 4. The bank rate means (A) Rate of interest charged by commercial banks from borrowers (B) Rate of interest at which commercial banks discounted bills of their borrowers (C) Rate of interest allowed by commercial banks on their deposits (D) Rate at which RBI purchases or rediscounts bills of exchange of commercial banks (E) None of these 5. What is an Indian Depository Receipt ? (A) A deposit account with a Public Sector Bank (B) A depository account with any of Depositories in India (C) An instrument in the form of depository receipt created by an Indian depository against underlying equity shares of the issuing company (D) An instrument in the form of deposit receipt issued by Indian depositories (E) None of these 6. An instrument that derives its value from a specified underlying (currency, gold, stocks etc.) is known as (A) Derivative (B) Securitisation Receipts (C) Hedge Fund (D) Factoring (E) Venture Capital Funding 7. Fiscal deficit is (A) total income less Govt. borrowing (B) total payments less total receipts (C) total payments less capital receipts (D) total expenditure less total receipts excluding borrowing (E) None of these 8. In the Capital Market, the term arbitrage is used with reference to (A) purchase of securities to cover the sale (B) sale of securities to reduce the loss on purchase (C) simultaneous purchase and sale of securities to make profits from price (D) variation in different markets (E) Any of the above

9. Reverse repo means (A) Injecting liquidity by the Central Bank of a country through purchase of Govt. securities (B) Absorption of liquidity from the market by sale of Govt. securities (C) Balancing liquidity with a view to enhancing economic growth rate (D) Improving the position of availability of the securities in the market (E) Any of the above 10. The stance of RBI monetary policy is (A) inflation control with adequate liquidity for growth (B) improving credit quality of the Banks (C) strengthening credit delivery mechanism (D) supporting investment demand in the economy (E) Any of the above 11. Currency Swap is an instrument to manage (A) Currency risk (B) interest rate risk (C) currency and interest rate risk (D) cash flows in different currencies (E) All of the above 12. Sub-prime refers to (A) lending done by banks at rates below PLR (B) funds raised by the banks at sub-Libor rates (C) Group of banks which are not rated as prime banks as per Bankers Almanac (D) lending done by financing institutions including banks to customers not meeting with normally required credit appraisal standards (E) All of the above 13. Euro Bond is an instrument (A) issued in the European market (B) issued in Euro Currency (C) issued in a country other than the country of the currency of the Bond (D) All of the above (E) None of these 14. Money Laundering normally involves (A) placement of funds (B) layering of funds (C) integration of funds (D) All of (A), (B) and (C) (E) None of (A), (B) and (C) 15. The IMF and the World Bank were conceived as institutions to (A) strengthen international economic co-operation and to help create a more stable and prosperous global economy (B) IMF promotes international monetary cooperation (C) The World Bank promotes long term economic development and poverty reduction

(D) All of (A), (B) and (C) (E) None of (A), (B) and (C) 16. Capital Market Regulator is (A) RBI (B) IRDA (C) NSE (D) BSE (E) SEBI 17. In the term BRIC, R stands for (A) Romania (B) Rajithan (C) Russia (D) Regulation (E) None of these 18. FDI refers to (A) Fixed Deposit Interest (B) Fixed Deposit Investment (C) Foreign Direct Investment (D) Future Derivative Investment (E) None of these 19. What is Call Money ? (A) Money borrowed or lent for a day or over night (B) Money borrowed for more than one day but upto 3 days (C) Money borrowed for more than one day but upto 7 days (D) Money borrowed for more than one day but upto 14 days (E) None of these 20. Which is the first Indian company to be listed in NASDAQ ? (A) Reliance (B) TCS (C) HCL (D) Infosys (E) None of these 21. Which of the following is the Regulator of the credit rating agencies in India ? (A) RBI (B) SBI (C) SIDBI (D) SEBI (E) None of these 22. Who is Brand Endorsing Personality of Bank of Baroda ? (A) Juhi Chawla (B) Kiran Bedi (C) Amitabh Bachchan (D) Kapil Dev (E) None of these

23. The branding line of Bank of Baroda is (A) International Bank of India (B) Indias International Bank (C) Indias Multinational Bank (D) Worlds local Bank (E) None of these 24. The logo of Bank of Baroda is known as (A) Sun of Bank of Baroda (B) Baroda Sun (C) Bank of Barodas Rays (D) Sunlight of Bank of Baroda (E) None of these 25. Which of the following statements(s) is/are True about the exports of China which is a close competitor of India ? (i) Chinas economic success is basically on the fact that it exports cheaper goods to rich nations like the USA, etc. (ii) In the year 2007 Chinas exports became almost 40% of its GDP. (iii) When compared to India Chinas share in the World Exports is more than 30% whereas Indias share is mere 6% of the global exports. (A) Only (i) (B) Only (ii) (C) Both (i) and (ii) (D) All (i), (ii) and (iii) (E) None of these 26. One of the major challenges banking industry is facing these days is money laundering. Which of the following acts/norms are launched by the banks to prevent money laundering in general ? (A) Know Your Customer Norms (B) Banking Regulation Act (C) Negotiable Instrument Act (D) Narcotics and Psychotropic Substance Act (E) None of these 27. Lot of Banks in India these days are offering M-Banking Facility to their customers. What is the full form of M in M-Banking ? (A) Money (B) Marginal (C) Message (D) Mutual Fund (E) Mobile Phone 28. Which of the following is/are true about the Sub-Prime Crisis ? (The term was very much in news recently.) (i) It is a mortgage crisis referring to credit default by the borrowers. (ii) Sub-Prime borrowers were those borrowers who were rated low and were high risk

borrowers. (iii) This crisis originated because of negligence in credit rating of the borrowers. (A) Only (i) (B) Only (ii) (C) Only (iii) (D) All (i), (ii) and (iii) (E) None of these 29. Which of the following is not the part of the structure of the Financial System in India ? (A) Industrial Finance (B) Agricultural Finance (C) Government Finance (D) Development Finance (E) Personal Finance 30. Which of the following is not the part of the scheduled banking structure in India ? (A) Money Lenders (B) Public Sector Banks (C) Private Sector Banks (D) Regional Rural Banks (E) State Co-operative Banks 31. As we all know Govt. of India collects tax revenue on various activities in the country. Which of the following is a part of the tax revenue of the Govt. ? (i) Tax on Income (ii) Tax on Expenditure (iii) Tax on Property or Capital Asset (iv) Tax on Goods and Services (A) Both (i) and (iii) only (B) Both (ii) and (iv) only (C) All (i), (ii), (iii) and (iv) (D) Only (ii), (iii) and (iv) (E) None of these 32. We very frequently read about Special Economic Zones (SEZs) in newspapers. These SEZs were established with which of the following objectives ? (i) To attract foreign investment directly. (ii) To protect domestic market from direct competition from multinationals. (iii) To provide more capital to agricultural and allied activities. (A) Only (i) (B) Only (ii) (C) Only (iii) (D) All (i), (ii) and (iii) (E) None of these 33. Which of the following groups of countries has almost 50% share in global emission of carbon every year ? (A) US, China, India, South Africa

(B) India, China, Russia, Britain (C) South Africa, Nepal, Myanmar (D) US, Russia, China & India (E) None of these 34. Which of the following correctly describes the concept of Nuclear Bank floated by International Atomic Energy Agency ? (i) It is a nuclear fuel bank to be shared by all the nations jointly. (ii) It is a facility to help nations in enrichment of uranium. (iii) It is an agency which will keep a close vigil on the nuclear programme of all the nations. (A) Only (i) (B) Only (ii) (C) Both (i) and (iii) only (D) Only (iii) (E) Both (i) and (ii) only 35. Many times we read about Future Trading in newspapers. What is Future Trading ? (i) It is nothing but a trade between any two stock exchanges wherein it is decided to purchase the stocks of each other on a fixed price throughout the year. (ii) It is an agreement between two parties to buy or sell an underlying asset in the future at a predetermined price. (iii) It is an agreement between stock exchanges that they will not trade the stocks of each other under any circumstances in future or for a given period of time. (A) Only (i) (B) Only (ii) (C) Only (iii) (D) All (i), (ii) and (iii) (E) None of these 36. Inflation in India is measured on which of the following indexes/indicators ? (A) Cost of Living Index (COLI) (B) Consumer Price Index (CPI) (C) Gross Domestic Product (D) Wholesale Price Index (WPI) (E) None of these 37. As per the reports published in the newspapers a section of society staged a demonstration at the venue of the G-8 Summit recently. What was/were the issues towards which these demonstrators were trying to draw the attention of G-8 leaders ? (i) Food shortage which has taken 50 million people in its grip. (ii) Inflation which has gone up substantially across the Globe. (iii) USAs consistent presence in Iraq. (A) Only (i)

(B) Only (ii) (C) Only (iii) (D) Both (i) and (ii) only (E) None of these 38. Hillary Clinton formally suspended her campaign to ensure election of who amongst the following for the next President of USA ? (A) George Bush (B) Barack Obama (C) John McCain (D) Bill Clinton (E) None of these 39. Hugo Chavez whose name was recently in news is the (A) President of Congo (B) Prime Minister of Uganda (C) President of Venezuela (D) Prime Minister of Brazil (E) None of these 40. The Govt. of India has raised the amount of the Loan Waiver to the farmers by 20%. Now the amount is nearly (A) Rs. 60,000 crore (B) Rs. 65,000 crore (C) Rs. 72,000 crore (D) Rs. 76,000 crore (E) Rs. 80,000 crore 41. Delimitation Commission has made a recommendation that next Census should be Panchayat-wise. When is the next Census due ? (A) 2010 (B) 2011 (C) 2012 (D) 2013 (E) 2015 42. The World Health Organisation has urged that advertisements of which of the following should be banned to protect youth from bad effects of the same ? (A) Tobacco (B) Alcoholic drinks (C) Junk Food (D) Soft drinks with chemical preservatives (E) None of these 43. Which of the following countries has allocated a huge amount of US $ 10 billion to provide relief to its earthquake victims ? (A) Japan (B) South Korea (C) China (D) South Africa (E) None of these

44. India and Nepal have many agreements on sharing of the water of various rivers. Which of the following rivers is not covered under these agreements ? (A) Kosi (B) Gandak (C) Ganga (D) Mahakali (E) All these rivers are covered 45. Which of the following names is not closely associated with space programme of India or any other country ? (A) CARTOSAT (B) NLS 5 (C) RUBIN 8 (D) GSLV (E) SCOPE 46. Vijay Hazare Trophy is associated with the game of (A) Hockey (B) Cricket (C) Badminton (D) Football (E) Golf 47. Which of the following was the theme of the Olympic Torch ? (A) Journey of Harmony (B) Green World Clean World (C) Journey of Peace (D) Journey for Hunger-free World (E) None of these 48. Which of the following schemes is not a social development Scheme ? (A) Indira Awas Yojana (B) Mid Day Meal (C) Bharat Nirman Yojana (D) Sarva Shiksha Abhiyan (E) All are social schemes 49. Which of the following is not a member of the ASEAN ? (A) Malaysia (B) Indonesia (C) Vietnam (D) Britain (E) Singapore 50. Which of the following Awards are given for excellence in the field of Sports ? (A) Kalinga Prize (B) Shanti Swarup Bhatnagar Award (C) Arjun Award (D) Pulitzer Prize (E) None of these

Answers : 1. (E) 2. (B) 3. (A) 4. (D) 5. (C) 6. (C) 7. (D) 8. (C) 9. (A) 10. (E) 11. (D) 12. (D) 13. (C) 14. (D) 15. (D) 16. (E) 17. (C) 18. (C) 19. (A) 20. (D) 21. (D) 22. (E) 23. (B) 24. (B) 25. (C) 26. (E) 27. (E) 28. (D) 29. (E) 30. (A) 31. (C) 32. (A) 33. (D) 34. (B) 35. (B) 36. (D) 37. (B) 38. (B) 39. (C) 40. (A) 41. (B) 42. (A) 43. (C) 44. (C) 45. (E) 46. (B) 47. (A) 48. (C) 49. (D) 50. (C

COMPUTER
Questions on Computer Awareness have recently been introduced in most Bank Recruitment exams. To help you prepare better for the Bank exams we present to you the revision notes on Computer Awareness. 1. The earlier computers, which were massive in size, were based onvacuum tubes. 2. Early computing machines, like the ENIAC, were actually meant to assist the armed forces. 3. The printers in pre-1950s were punch cards. 4. An improvement on the ENIAC, which pioneered 'stored program', was made possible with the help of the mathematician John von Neumann. 5. Before the 1950s, computers were mostly owned by universities and research labs. 6. The B-programming language was developed by Ken Thompson. 7. Famous people, associated with the ENIAC, EDVAC, UNIVAC computers are Eckert & Mauchly. 8. The 1st commercially produced and sold computer (1951) was UNIVAC. 9. IBM was provided software for PCs by Microsoft. 10. Time-sharing, teletyping, were associated with mainframe computers. 11. The transformation from heavy computers to PCs was made possible using microprocessors. 12. The first microprocessor was developed in 1971 by Intel. 13. The term 'micro' (extremely small) denotes 10-6m. 14. The Harvard student, who chose to write computer programs and dropped studies was Bill Gates. 15. A pentium 4 (P-4) employs roughly 40 million transistors. 16. Mark-1, Apple-1, and collossus were initial desktop computers. 17. Binary digits are briefed as bit. 18. A collection of bits is called byte. 19. C++, is a computer language. 20. The process of eliminating programming faults is called debugging. 21. Starting up on operating system is called booting. 22. A program used to browse the web is called browser.

23. An error in software designing which can even cause a computer to crash is called bug. 24. Click and double-click are achieved using the mouse. 25. Java, C, ForTran, Pascal and BASIC are computer programming languages. 26. The device which sends computer data using a phone line is calledMODEM. 27. 'Worm' and 'virus' are actually programs. 28. A 'file' is a unit of information. 29. A megabyte has 106(million) bytes. 30. A small, single-site network is called LAN. 31. A processor that collects several data and sends them over a single line is called bridge. 32. 'Nano' stands for one billionth part. 33. The number of bit patterns using an n-bit code is 2n. 34. The part of a computer that works with the data/programs is called CPU. 35. To convert a binary number to a decimal, we have to express it in power of 2. 36. www stands for world wide web. 37. Mathematics employed in computers is called Boolean algebra. 38. A collection of 8 bits is called byte. 39. The first home computer (1977), which was sold in millions of units wasApple II. 40. 'PARAM' is a supercomputer. 41. A website containing periodic posts is called blog. 42. While cutting and pasting, the cutitem is temporarily stored in theclipboard. 43. http stands for hyper text transfer protocol. 44. The unwanted or non-requested emails are called "spam". 45. A computer framed to give various network services is called server.

Allahabad Bank Probationary Officer Exam Computer General Awareness

1. Programs from the same developer , sold bundled together , that provide better integration and share common features , toolbars and menus are known as . (A) software suites (B) integrated software packages (C) software processing packages (D) personal information managers (E) none of these

2. A data warehouse is which of the following ? (A) Can be updated by the end users (B) Contains numerous naming conventions and formats (C) Organized around important subject areas (D) Contains only current data (E) None of these 3. ______________ servers store and manages files for network users. (A) Authentication (B) Main (C) Web (D) File (E) None of these 4. All of the following are examples of real security and privacy risks Except _____ (A) hackers (B) spam (C) viruses (D) identify theft (E) None of these 5. ___________ terminals (formerly known as cash registers) are often connected to complex inventory and sales computer systems . (A) Data (B) Sales (C) Query (D) (Point-of-sale (POS) (E) None of these 6. The OSI model is divided into _______ processes called layers. (A) five (B) six (C) seven (D) eight (E) None of these

7. System software is the set of programs that enables your computers hardware devices and ______ software to work together. (A) management (B) processing (C) utility (D) application (E) None of these 8. ________ are specially designed computer chips reside inside other devices, such as your car or your electronic thermostat. (A) Servers (B) Embedded computers (C) Robotic computers (D) Mainframes (E) None of these 9. The following are all computing devices , except (A) notebook computers (B) cellular telephones (C) digital scanners (D) personal digital assistants (E) None of these 10. in a ring topology , the the computer in possession of the ______ can trasmit data (A) packet (B) data (C) access method (D) token (E) None of these 11. This part of operating system manages the essential peripherals, such as the keyboard, screen , disk drives, and parallel and serial ports ____ (A) basic input/output system (B) secondary input/output system (C) peripheral input/output system (D) marginal input/output system (E) None of these

12. ________ Viruses are often transmitted by a floppy disk left in the floppy drive (A) Trojan horse (B) Boot sector (C) Script (D) Logic bomb (E) None of these 13. ________ controls the way in which the computer system functions and provides a means by which users can interact with the computer (A) The platform (B) Application software (C) Operating system (D) The motherboard (E) None of these 14. Servers are computers that provide resources to other computers connected to a ___________ (A) mainframe (B) network (C) supercomputer (D) client (E) None of these 15. A goal of data mining includes which of the following? (A) To explain some observed event or condition (B) To confirm that data exists (C) To analyze data for expected relationships (D) To create a new data warehouse (E) None of these 16. URL stands for .. (A) Universal Research List (B) Universal Resource List (C) Uniform Resource Locator (D) Uniform Research Locator (E) None of these

17. A database management system (DBMS) is a (A) hardware system used to create , maintain and provide controlled access to a database (B) hardware system used to create, maintain, and provide uncontrolled access to a database. (C) software system used to create, maintain, and provide uncontrolled access to a database. (D) software system used to create, maintain, and provide controlled access to a database. (E) None of these 19. A Proxy server is used for which of the following? (A) To provide security against unauthorized users (B) To process client requests for web pages (C) To process client requests for database access (D) To provide TCP/IP (E) None of these 20. When data changes in multiple lists and all lists are not updated, this causes: (A) data redundancy (B) information overload (C) duplicate data (D) data inconsistency (E) None of these 21. _____________ are words that a programming language has set aside for its own use. (A) Control worlds (B) Reserved words (C) Control structures (D) Reserved keys (E) None of these 23. ______ describe what is database fields. (A) Structures (B) Field markers (C) Field definition (D) Field names (E) None of these

24. You must install a (n) ____________ on a network if you want to share a broadband Internet connection. (A) router (B) modem (C) node (D) cable (E) None of these 25. A goal of normalization is to __________ (A) minimize the number of relationships (B) minimize the number of entities (C) minimize the number of tables (D) minimize the number of redundancy (E) None of these 26. Granting an outside organization access to internet web pages is often implemented using a (n) ____ (A) extranet (B) intranet (C) internet (D) hacker (E) None of these 27. Which term identifies a specific computer on the web and the main page of the entire site (A) URL (B) Web site address (C) Hyperlink (D) Domain name (E) None of these 28. The code that relational database management systems use to perform their database task is referred to as .. (A) QBE (B) SQL (C) OLAP (D) Sequel Server (E) None of these

29. Chip is a common nickname for a (n): (A) transistor (B) resistor (C) integrated circuit (D) semiconductor (E) None of these 30. The purpose of the primary key in a database is to: (A) unlock the database (B) provide a map of the data (C) uniquely identify a record (D) establish constraints on database operations. (E) None of these 31. The design of the network is called the network: (A) architecture (B) server (C) transmission (D) type (E) None of these 32. The most frequently used instructions of a computer program are likely to be fetched from: (A) the hard disk (B) cache memory (C) RAM (D) registers (E) None of these 33. The ____________ contains data descriptions and defines the name, data type, and length of each field in the database. (A) data dictionary (B) data table (C) data record (D) data field (E) None of these

34. Personal logs or journal entries posted on the Web are known as: (A) listservs (B) Webcasts (C) blogs (D) subject directories (E) None of these 35. A(n) _______ uses pictures (called icons) and menus displayed on the screen to send commands to the computer system. (A) command based user interface (B) GUI (C) system utility (D) API (E) None of these 36. Which of the following is NOT a type of broadband Internet connection? (A) Cable (B) DSL (C) Dial-up (D) Satellite (E) None of these 37. Software, such as viruses, worms and Trojan horses, that has a malicious intent, is known as: (A) spyware (B) adware (C) spam (D) malware (E) None of these 38. Making a field ______ means that it cannot be left blank. (A) numeric (B) required (C) calculated (D) validated (E) None of these

39. ____________ are viruses that are triggered by the passage of time or on a certain date. (A) Boot-sector viruses (B) Macro viruses (C) Time bombs (D) Worms (E) None of these 40. Linux is a (n) ____________ operating system. (A) open-source (B) Microsoft (C) Windows (D) Mac (E) None of these 41. What is a backup? (A) Restoring the information backup (B) An exact copy of a systems information (C) The ability to get a system up and running in the event of a system crash or failure (D) All of these (E) None of these 42. Which of the following places the common data elements in order from smallest to largest (A) character, file, record, field, database (B) character, record, field, database, file (C) character, field, record, file, database (D) Bit, byte, character, record, field, file, database, (E) None of these 43. The internet is (A) a large network of networks (B) an internal communication system for a business (C) a communication system for the Indian government (D) All of these (E) None of these 44. ____________ are lists of commands that appear on the screen. (A) GUIs

(B) Icons (C) Menus (D) Windows (E) None of these 45. Which of the following statements is FALSE concerning file names? (A) Files may share the same name or the same extension but not both (B) Every file in the same folder must have a unique name (C) File extension is another name for file type (D) The file extension comes before the dot (.) followed by the file name (E) None of these 46. Verification of a login name and password is known as: (A) configuration (B) accessibility (C) authentication (D) logging in (E) None of these 47. Which of the following would most likely NOT be a symptom of a virus? (A) Existing program files and icons disappear. (B) The CD-ROM stops functioning. (C) The Web browser opens to an unusual home page. (D) Odd messages or images are displayed on the screen. (E) None of these

Anda mungkin juga menyukai